OB midterm chapters 1,2,3,10,11,12,19,20,21,13,14

अब Quizwiz के साथ अपने होमवर्क और परीक्षाओं को एस करें!

By the end of the second stage of labor, the nurse would expect which of the following events? The A. cervix is fully dilated and effaced B. placenta is detached and expelled C. fetus is born and on mother's chest D. woman to request pain medication

"C" because the second stage of labor is defined as beginning with complete dilation of the cervix (10 cm) and ending with the expulsion of the fetus. - Response "A" is incorrect because the cervix is fully dilated at the start of stage 2, not at the end of it. Response "

A nurse is teaching a pregnant woman at risk for preterm labor about what to do if she experiences signs and symptoms. The nurse determines that the teaching was successful when the woman states that if she experiences any symptoms, she will do which of the following? A) "I'll sit down to rest for 30 minutes." B) "I'll try to move my bowels." C) "I'll lie down with my legs raised." D) "I'll drink several glasses of water."

"I'll drink several glasses of water." If the woman experiences any signs and symptoms of preterm labor, she should stop what she is doing and rest for 1 hour, empty her bladder, lie down on her side, drink two to three glasses of water, feel her abdomen and note the hardness of the contraction, and call her health care provider and describe the contraction.

A woman is to undergo an amnioinfusion. Which statement would be most appropriate to include when teaching the woman about this procedure? A) "You'll need to stay in bed while you're having this procedure." B) "We'll give you an analgesic to help reduce the pain." C) "After the infusion, you'll be scheduled for a cesarean birth." D) "A suction cup is placed on your baby's head to help bring it out."

"You'll need to stay in bed while you're having this procedure." An amnioinfusion involves the instillation of a volume of warmed, sterile normal saline or Ringer's lactate into the uterus via an intrauterine pressure catheter. The client must remain in bed during the procedure. The use of analgesia is unrelated to this procedure. A cesarean birth is necessary only if the FHR does not improve after the amnioinfusion. Application of a suction cup to the head of the fetus refers to a vacuum-assisted birth.

Leopold's maneuvers steps

- Maneuver 1: What fetal part (head or buttocks) is located in the fundus (top of the uterus)? - Maneuver 2: On which maternal side is the fetal back located? (Fetal heart tones are best auscultated through the back of the fetus.) - Maneuver 3: What is the presenting part? - Maneuver 4: Is the fetal head flexed and engaged in the pelvis?

When does the perineal phase of the second stage occurs

- Period of active pushing - with complete cervical dilation and effacement, contractions occurring every 2 to 3 minutes and lasting 60 to 90 seconds, and a tremendous urge to push by the mother.

A nurse is assigned the task of educating a pregnant client about childbirth. Which of the following nursing interventions should the nurse perform as a part of prenatal education for the client to ensure a positive childbirth experience?

- Provide the client clear information on procedures involved - Encourage the client to have a sense of mastery and self-control - Encourage the client to have a positive reaction to pregancy

- Signs that the placenta is separating include:

- a firmly contracting uterus - a change in uterine shape from discoid to globular ovoid - a sudden gush of dark blood from the vaginal opening - and lengthening of the umbilical cord protruding from the vagina.

The extent of the laceration is defined by depth:

- a first-degree laceration extends through the skin - a second-degree laceration extends through the muscles of the perineal body - a third-degree laceration continues through the anal sphincter muscle - a fourth-degree laceration also involves the anterior rectal wall.

Flexion and extension are terms used to describe fetal ____________ . Cephalic is a term used to describe fetal _________ .

- attitude - presentation

The active phase of the first stage of labor is characterized by

- cervical dilation of 4 to 7 cm - effacement of 40% to 80% - contractions occurring every 2 to 5 minutes lasting 45 to 60 seconds.

The transition phase of the first stage of labor is characterized by

- cervical dilation of 8 to 10 cm - effacement of 80% to 100% - contractions occurring every 1 to 2 minutes lasting 60 to 90 seconds.

what is amniotic fluid? also, how is it made?

- fluid serving as a cushion to fetus + umbilical cord (trauma/compression prevention). good for allowing musculoskeletal development via free-movement, + maintaining fetal temperature. composed 98% water, 2% organic materials (bilirubin, urea, uric acid, creatinine, etc) - the inner-layer amnion (from ectoderm of blastocyst) expands until it touches the outer-layer chorion (from the trophoblast)

A nurse is caring for a pregnant client during labor. Which of the following methods should the nurse use to provide comfort to the pregnant client? Select all

- hand holding - massaging - acupuncture

During labor, the mother experiences various physiologic responses including:

- increase in heart rate by 10 to 20 bpm - a rise in blood pressure by up to 35 mm Hg during a contraction - increase in white blood cell count to 25,000 to 30,000 cells/mm3, perhaps as a result of tissue trauma - an increase in respiratory rate with greater oxygen consumption due to the increase in metabolism.

A client in labor is administered lorazepam to help her relax enough so that she can participate effectively during her labor process rather than fighting against it. For which adverse effect of the drug should the nurse monitor? A. Increased sedation B. Newborn respiratory depression C. Nervous system depression D. Decreased alertness

- increase sedation is an adverse effect of lorezapam - diazepam and midazolam cause CNS depression - opiods cause newborn respiration depression

Latent phase of the first stage of labor

- involves cervical dilation of 0 to 3 cm - cervical effacement of 0% to 40%, - contractions every 5 to 10 minutes lasting 30 to 45 seconds.

Latent phase of labor:

- is characterized by mild contractions every 5 to 10 minutes, -cervical dilation of 0 to 3 cm - effacement of 0% to 40%, - excitement and frequent talking by the mother.

Active phase of labor:

- moderate to strong contractions every 2 to 5 minutes - cervical dilation of 4 to 7 cm - effacement of 40% to 80% - with the mother becoming intense and inwardly focused.

what are the three stages of pregnancy, and when do they occur?

- preembryonic stage: fertilization - 2nd week - embryonic stage: end of 2nd week - 8th week - fetal stage - end of 8th week - birth (longest period)

morula:

- the resulting ball of cells from the mitosis of the zygote. after fertilization, the zygote makes it's way to the uterus (72 hrs), while on its way it goes through mitosis (or cleavage) x4, where it ends up a ball of 16 cells known as a morula.

A pregnant woman admitted to the labor and birth suite undergoes rapid HIV testing and is found to be HIV-positive. Which of the following would the nurse expect to include when developing a plan of care for this women? (Select all that apply.) A) Administration of penicillin G at the onset of labor B) Avoidance of scalp electrodes for fetal monitoring C) Refraining from obtaining fetal scalp blood for pH testing D) Administering zidovudine at the onset of labor. E) Electing for the use of forceps-assisted delivery

-Avoidance of scalp electrodes for fetal monitoring -Refraining from obtaining fetal scalp blood for pH testing -Administering zidovudine at the onset of labor

__________ labor is characterized by contractions that are irregular and weak, often slowing down with walking or a position change. ________ labor contractions begin in the back and radiate around toward the front of the abdomen. They are regular and become stronger over time; the woman may find it extremely difficult if not impossible to have a conversation during a contraction.

-False -True

The nurse is providing home care for a 6-year-old girl with multiple medical challenges. Which of the following activities would be considered the tertiary level of prevention? A) Arranging for a physical therapy session B) Teaching parents to administer albuterol C) Reminding parent to give a full course of antibiotics D) Giving a DTaP vaccination at the proper interval

-The tertiary level of prevention involves restorative, rehabilitative, or quality-of-life care, such as arranging for a physical therapy session. - Teaching the parents to administer albuterol and reminding them to give the full course of antibiotics as prescribed are part of the secondary level of prevention, which focuses on diagnosis and treatment of illness. - Giving a DTaP vaccination at proper intervals is an example of the primary level of prevention, which centers on health promotion and illness prevention.

Which position would be most appropriate for the nurse to suggest as a comfort measure to a woman who is in the first stage of labor? (Select all that apply.) A) Walking with partner support B) Straddling with forward leaning over a chair C) Closed knee-chest position D) Rocking back and forth with foot on chair E) Supine with legs raised at a 90-degree angle

-Walking with partner support -Straddling with forward leaning over a chair -Rocking back and forth with foot on chair

Symptoms of preterm labor

-increase vaginal discharge - pushing down sensation - low dull pain - menstrual- like cramps - uterine contraction without pain

Positioning during the first stage of labor includes:

-walking with support from the partner - side-lying with pillows between the knees - leaning forward by straddling a chair, table, or bed or kneeling over a birthing ball -lunging by rocking weight back and forth with a foot up on a chair or birthing ball or an open knee-chest position.

For continuous internal electronic fetal monitoring, four criteria must be met:

1. ruptured membranes 2. cervical dilation of at least 2 cm 3. fetal presenting part low enough to allow placement of the electrode 4. skilled practitioner available to insert the electrode.

The nurse is providing care to several pregnant women who may be scheduled for labor induction. The nurse identifies the woman with which Bishop score as having the best chance for a successful induction and vaginal birth? A) 11 B) 8 C) 6 D) 3

11 The Bishop score helps identify women who would be most likely to achieve a successful induction. The duration of labor is inversely correlated with the Bishop score: a score over 8 indicates a successful vaginal birth. Therefore the woman with a Bishop score of 11 would have the greatest chance for success.

Contractions should occur every

2 to 3 minutes, lasting 40 to 60 seconds.

A uterine resting tone greater than ____ mm Hg would require intervention.

20

The nurse is monitoring a pregnant client admitted to a health care center who is in the latent phase of labor. The nurse demonstrates appropriate nursing care by monitoring the fetal heart rate with the Dopplar at least how often?

30 to 60 min

The woman's temperature is typically assessed every ____ hours during the first stage of labor and every ____ hours after ruptured membranes.

4 2

A 29-week-gestation client is admitted with moderate vaginal discharge. The nurse performs a nitrazine test to determine if the membranes have ruptured. The nitrazine tape remains yellow to olive green, with pH between 5 and 6. What should the nurse do NEXT? A. Prepare the client for birth B. Assess the client's cervical status C. notify the health care provider D. perform leopold's maneuver

5 to 6 pH means acidic environment with presence of vaginal fluid and less blood. notify provider

A pregnant woman in her 39th week of pregnancy presents to the clinic with a vaginal infection. She tests positive for chlamydia. What would this disease make her infant at risk for? A. blindness B. neonatal laryngeal papillomas C.deafness D. chicken pox

A R:A pregnant woman who contracts chlamydia is at increased risk for spontaneous abortion (miscarriage), preterm rupture of membranes, and preterm labor. The postpartum woman is at higher risk for endometritis (Fletcher & Ball, 2006). The fetus can encounter bacteria in the vagina during the birth process. If this happens, the newborn can develop pneumonia or conjunctivitis that can lead to blindness.

A nurse is conducting a class on gestational diabetes for a group of pregnant women who are at risk for the condition. The nurse determines that additional teaching is needed when the class identifies which complication as affecting the neonate? A. hyperglycemia B. birth trauma C. hypoglycemia D. macrosomia

A R:Gestational diabetes is associated with either neonatal complications such as macrosomia, hypoglycemia, and birth trauma or maternal complications such as preeclampsia and cesarean birth.

A woman with diabetes is in labor. To reduce the likelihood of neonatal hypoglycemia, the nurse monitors the client's blood glucose level closely with the goal to maintain which level? A. below 110 mg/dL B. below 115 mg/dL C. below 105 mg/dL D. below 120 mg/dL

A R: For the laboring woman with diabetes, the blood glucose levels are monitored every 1 to 2 hours with the goal to maintain the levels below 110 mg/dL throughout the labor to reduce the likelihood of neonatal hypoglycemia. If necessary, an infusion of regular insulin may be given to maintain this level.

A pregnant client with diabetes in the hospital reports waking up with shakiness and diaphoresis. Which action should the nurse prioritize after discovering the client's fasting blood sugar is 60 mg/dL? A. Provide the client some milk to drink. B. Withhold her insulin, and notify the health care provider. C. Stay with her, and ask another nurse to bring her insulin. D. Recheck her blood sugar for accuracy.

A R: The client is hypoglycemic when awakening in the morning. The nurse should provide glucose in the form of carbohydrate, such as crackers, and milk, and be prepared to reassess. The nurse should not recheck at this point, since the client is symptomatic. She does not need insulin, and she will have her morning dose adjusted after breakfast.

25. A nurse is describing how the fetus moves through the birth canal. Which of the following would the nurse identify as being most important in allowing the fetal head to move through the pelvis? A) Sutures B) Fontanelles C) Frontal bones D) Biparietal diameter

A Sutures are important because they allow the cranial bones to overlap in order for the head to adjust in shape (elongate) when pressure is exerted on it by uterine contractions or the maternal bony pelvis. * Fontanelles are the intersections formed by the sutures.

8. Assessment of a woman in labor reveals cervical dilation of 3 cm, cervical effacement of 30%, and contractions occurring every 7 to 8 minutes, lasting about 40 seconds. The nurse determines that this client is in: A) Latent phase of the first stage B) Active phase of the first stage C) Transition phase of the first stage D) Perineal phase of the second stage

A The latent phase of the first stage of labor involves cervical dilation of 0 to 3 cm, cervical effacement of 0% to 40%, and contractions every 5 to 10 minutes lasting 30 to 45 seconds.

A pregnant woman in her 39th week of pregnancy presents to the clinic with a vaginal infection. She tests positive for chlamydia. What would this disease make her infant at risk for? A. blindness B. neonatal laryngeal papillomas C.deafness D. chicken pox

A A pregnant woman who contracts chlamydia is at increased risk for spontaneous abortion (miscarriage), preterm rupture of membranes, and preterm labor. The postpartum woman is at higher risk for endometritis (Fletcher & Ball, 2006). The fetus can encounter bacteria in the vagina during the birth process. If this happens, the newborn can develop pneumonia or conjunctivitis that can lead to blindness.

The nurse notes persistent early decelerations on the fetal monitoring strip. Which of the following would the nurse do next? A) Continue to monitor the FHR because this pattern is benign. B) Perform a vaginal exam to assess cervical dilation and effacement. C) Stay with the client while reporting the finding to the physician. D) Administer oxygen after turning the client on her left side.

A Early decelerations are not indicative of fetal distress and do not require intervention. Therefore, the nurse would continue to monitor the fetal heart rate pattern. They are most often seen during the active stage of any normal labor, during pushing, crowning, or vacuum extraction. They are thought to be a result of fetal head compression that results in a reflex vagal response with a resultant slowing of the FHR during uterine contractions.

trisomy

A genetic disorder in which a person has three copies of a chromosome instead of two. ex. down syndrome

blastocyst

A thin-walled hollow structure in early embryonic development that contains a cluster of cells called the inner cell mass from which the embryo arises. pg. 336

After teaching nursing students about the basic concepts of family-centered care, the instructor determines that the teaching was successful when the students state which of the following? A) "Childbirth affects the entire family, and relationships will change." B) "Families are not capable of making health care decisions for themselves." C) "Mothers are the family members affected by childbirth." D) "Childbirth is a medical procedure."

A) "Childbirth affects the entire family, and relationships will change."

After teaching a group of students about the concept of maternal mortality, the instructor determines that additional teaching is needed when the students state which of the following? A) "The rate includes accidental causes for deaths." B) "It addresses pregnancy-related causes." C) "The duration of the pregnancy is not a concern." D) "The time frame is typically for a specified year."

A) "The rate includes accidental causes for deaths."

The nurse is trying to get consent to care for an 11-year-old boy with diabetic ketoacidosis. His parents are out of town on vacation, and the child is staying with a neighbor. Which action would be the priority? A) Getting telephone consent with two people listening to the verbal consent B) Providing emergency care without parental consent C) Contacting the child's aunt or uncle to obtain their consent D) Advocating for termination of parental rights for this situation

A) Getting telephone consent with two people listening to the verbal consent

A group of nursing students are reviewing information about factors affecting maternal, newborn, and women's health. The students demonstrate understanding of the information when they identify which of the following deficiencies as being associated with poverty? Select all that apply. A) Literacy B) Employment opportunities C) Mobility D) Political representation E) Skills

A) Literacy B) Employment opportunities C) Mobility D) Political representation E) Skills

After teaching a group of nursing students about the issue of informed consent. Which of the following, if identified by the student, would indicate an understanding of a violation of informed consent? A) Performing a procedure on a 15-year-old without consent B) Serving as a witness to the signature process C) Asking whether the client understands what she is signing D) Getting verbal consent over the phone for emergency procedures

A) Performing a procedure on a 15-year-old without consent

A nurse is working to develop a health education program for a local community to address breast cancer awareness. Which of the following would the nurse expect to include when describing this problem to the group? Select all that apply. A) White women have higher rates of breast cancer than African American women. B) African American women are more likely to die from breast cancer at any age. C) Survival at any stage is worse among white women. D) Women living in South America have the highest rates of breast cancer. E) Breast cancer is the leading cause of cancer mortality in women.

A) White women have higher rates of breast cancer than African American women. B) African American women are more likely to die from breast cancer at any age.

A pregnant woman in her second trimester tells the nurse, "I've been passing a lot of gas and feel bloated." Which of the following suggestions would be helpful for the woman? A) "Watch how much beans and onions you eat." B) "Limit the amount of fluid you drink with meals" C) "Try exercising a little more." D) "Some say that eating mints can help." E) "Cut down on your intake of cheeses."

A) "Watch how much beans and onions you eat." C) "Try exercising a little more." D) "Some say that eating mints can help."

The experienced labor and birth nurse knows to evaluate progress in active labor by using which simple rule? A) 1 cm/hour for cervical dilation B) 2 cm/hour for cervical dilation C) 1/4 cm/hour for cervical dilation D) 1/2 cm/hour for cervical dilation

A) 1 cm/hour for cervical dilation In evaluating the progress in active labor, the nurse uses the simple rule of 1 cm/hour for cervical dilation.

A nursing instructor is describing trends in maternal and newborn health care. The instructor addresses the length of stay for vaginal births during the past decade, citing that which of the following denotes the average stay? A) 24-48 hours or less B) 72-96 hours or less C) 48-72 hours or less D) 96-120 hours or less

A) 24-48 hours or less

A nurse is examining a female client and tests the client's vaginal pH. Which finding would the nurse interpret as normal? A) 4.5 B) 7 C) 8.5 D) 10

A) 4.5

A biophysical profile has been completed on a pregnant woman. The nurse interprets which score as normal? A) 9 B) 7 C) 5 D) 3

A) 9

A pregnant woman with gestational diabetes comes to the clinic for a fasting blood glucose level. When reviewing the results, the nurse determines that which result indicates good glucose control? A) 90 mg/dL B) 100 mg/dL C) 110 mg /dL D) 120 mg/dL

A) 90 mg/dL

A nursing instructor is preparing a class discussion on case management in maternal and newborn health care. Which of the following would the instructor include as a key component? Select all that apply. A) Advocacy B) Coordination C) Communication D) Resource management E) Event managed care

A) Advocacy B) Coordination C) Communication D) Resource management

A pregnant woman asks the nurse about giving birth in a birthing center. She says, "I'm thinking about using one but I'm not sure." Which of the following would the nurse need to integrate into the explanation about this birth setting? (Select all that apply.) A) An alternative for women who are uncomfortable with a home birth. B) The longer length of stay needed when compared to hospital births C) Focus on supporting women through labor instead of managing labor D) View of labor and birth as a normal process requiring no intervention E) Care provided primarily by obstetricians withmidwives as backup care

A) An alternative for women who are uncomfortable with a home birth. C) Focus on supporting women through labor instead of managing labor D) View of labor and birth as a normal process requiring no intervention

After teaching a pregnant woman with iron deficiency anemia about nutrition, the nurse determines that the teaching was successful when the woman identifies which of the following as being good sources of iron in her diet? (Select all that apply.) A) Dried fruits B) Peanut butter C) Meats D) Milk E) White bread

A) Dried fruits B) Peanut butter C) Meats

A nurse is preparing a teaching program for a group of pregnant women about preventing infections during pregnancy. When describing measures for preventing cytomegalovirus infection, which of the following would the nurse most likely include? A) Frequent handwashing B) Immunization C) Prenatal screening D) Antibody titer screening

A) Frequent handwashing

A pregnant woman tests positive for HBV. Which of the following would the nurse expect to administer? A) HBV immune globulin B) HBV vaccine C) Acylcovir D) Valacyclovir

A) HBV immune globulin

After teaching a group of students about the changes in health care delivery and funding, which of the following, if identified by the group as a current trend seen in the maternal and child health care settings, would indicate that the teaching was successful? A) Increase in community settings for care B) Decrease in family poverty level C) Increase in hospitalization of children D) Decrease in managed care

A) Increase in community settings for care

A woman in the 34th week of pregnancy says to the nurse, "I still feel like having intercourse with my husband." The woman's pregnancy has been uneventful. The nurse responds based on the understanding that: A) It is safe to have intercourse at this time. B) Intercourse at this time is likely to cause rupture of membranes. C) There are other ways that the couple can satisfy their needs. D) Intercourse at this time is likely to result in premature labor.

A) It is safe to have intercourse at this time.

When describing the ovarian cycle to a group of students, which phase would the instructor include? A) Luteal phase B) Proliferative phase C) Menstrual phase D) Secretory phase

A) Luteal phase

A client with a 28-day cycle reports that she ovulated on May 10. The nurse would expect the client's next menses to begin on: A) May 24 B) May 26 C) May 30 D) June 1

A) May 24

A nurse is conducting a class for a group of teenage girls about female reproductive anatomy and physiology. Which of the following would the nurse include as an external female reproductive organ? Select all that apply. A) Mons pubis B) Labia C) Vagina D) Clitoris E) Uterus

A) Mons pubis B) Labia D) Clitoris

A nurse has been invited to speak at a local high school about adolescent pregnancy. When developing the presentation, the nurse would incorporate information related to which of the following? (Select all that apply.) A) Peer pressure to become sexually active B) Rise in teen birth rates over the years. C) Latinas as having the highest teen birth rate D) Loss of self-esteem as a major impact E) Majority of teen pregnancies in the 1517-year-old age group

A) Peer pressure to become sexually active C) Latinas as having the highest teen birth rate D) Loss of self-esteem as a major impact

A pregnant woman is scheduled to undergo percutaneous umbilical blood sampling. When discussing this test with the woman, the nurse reviews what can be evaluated with the specimens collected. Which of the following would the nurse include? (Select all that apply.) A) Rh incompatibility B) Fetal acid-base status C) Sex-linked disorders D) Enzyme deficiencies E) Coagulation studies

A) Rh incompatibility B) Fetal acid-base status E) Coagulation studies

The nurse is caring for a 2-week-old newborn girl with a metabolic disorder. Which of the following activities would deviate from the characteristics of family-centered care? A) Softening unpleasant information or prognoses B) Evaluating and changing the nursing plan of care C) Collaborating with the child and family as equals D) Showing respect for the family's beliefs and wishes

A) Softening unpleasant information or prognoses

A nurse is preparing a teaching plan for a woman who is pregnant for the first time. Which of the following would the nurse incorporate into the teaching plan to foster the client's learning? (Select all that apply.) A) Teach "survival skills" first B) Use simple, nonmedical language C) Refrain from using a hands-on approach D) Avoid repeating information E)Use visual materials such as photos and videos

A) Teach "survival skills" first B) Use simple, nonmedical language E)Use visual materials such as photos and videos

After teaching a group of students about the different levels of prevention, the instructor determines a need for additional teaching when the students identify which of the following as a secondary prevention level activity in community-based health care? A) Teaching women to take folic acid supplements to prevent neural tube defects B) Working with women who are victims of domestic violence C) Working with clients at an HIV clinic to provide nutritional and CAM therapies D) Teaching hypertensive clients to monitor blood pressure

A) Teaching women to take folic acid supplements to prevent neural tube defects

The nurse is teaching a health education class on male reproductive anatomy and asks the students to identify the site of sperm production. Which structure, if identified by the group, would indicate to the nurse that the teaching was successful? A) Testes B) Seminal vesicles C) Scrotum D) Prostate gland

A) Testes

A nurse is considering a change in employment from the acute care setting to community-based nursing. The nurse is focusing her job search on ambulatory care settings. Which of the following would the nurse most likely find as a possible setting? Select all that apply. A) Urgent care center B) Hospice care C) Immunization clinic D) Physician's office E) Day surgery center F) Nursing home

A) Urgent care center D) Physician's office E) Day surgery center

After teaching a group of students about the discomforts of pregnancy, the students demonstrate understanding of the information when they identify which as common during the first trimester? (Select all that apply.) A) Urinary frequency B) Breast tenderness C) Cravings D) Backache E) Leg cramps

A) Urinary frequency B) Breast tenderness C) Cravings

While a nurse is obtaining a health history, the client tells the nurse that she practices aromatherapy. The nurse interprets this as which of the following? A) Use of essential oils to stimulate the sense of smell to balance the mind and body B) Application of pressure to specific points to allow self-healing C) Use of deep massage of areas on the foot or hand to rebalance body parts D) Participation in chanting and praying to promote healing.

A) Use of essential oils to stimulate the sense of smell to balance the mind and body

A client is giving birth when shoulder dystocia occurs in the fetus. The nurse recognizes that which condition in the client is likely to increase the risk for shoulder dystocia? A) diabetes B) pendulous abdomen C) nullipara D) preterm birth

A) diabetes Shoulder dystocia is most apt to occur in women with diabetes, in multiparas, and in postdate pregnancies.

A 10-week pregnant woman with diabetes has a glycosylated hemoglobin (HbA1C. level of 13%. At this time the nurse should be most concerned about which of the following possible fetal outcomes? A)Congenital anomalies B)Incompetent cervix C)Placenta previa D)Abruptio placentae

A)Congenital anomalies

When teaching a class of pregnant women about the effects of substance abuse during pregnancy, which of the following would the nurse most likely include? A)Low-birth-weight infants B)Excessive weight gain C)Higher pain tolerance D)Longer gestational periods

A)Low-birth-weight infants

Because a pregnant clients diabetes has been poorly controlled throughout her pregnancy, the nurse would be alert for which of the following in the neonate at birth? A)Macrosomia B)Hyperglycemia C)Low birth weight D)Hypobilirubinemia

A)Macrosomia

After teaching a group of students about the use of antiretroviral agents in pregnant women who are HIV- positive, the instructor determines that the teaching was successful when the group identifies which of the following as the underlying rationale? A)Reduction in viral loads in the blood B)Treatment of opportunistic infections C)Adjunct therapy to radiation and chemotherapy D)Can cure acute HIV/AIDS infections

A)Reduction in viral loads in the blood

15. After teaching a group of students about the factors affecting the labor process, the instructor determines that the teaching was successful when the group identifies which of the following as a component of the true pelvis? (Select all that apply.) A) Pelvic inlet B) Cervix C) Mid pelvis D) Pelvic outlet E) Vagina F) Pelvic floor muscles

A, C, D The true pelvis is made up of three planes: the pelvic inlet, mid pelvis, and pelvic outlet. The cervix, vagina, and pelvic floor muscles are the soft tissues of the passageway.

23. A nurse is preparing a presentation for a group of pregnant women about the labor experience. Which of the following would the nurse most likely include when discussing measures to promote coping for a positive labor experience? (Select all that apply.) A) Presence of a support partner B) View of birth as a stressor C) Low anxiety level D) Fear of loss of control E) Participation in a pregnancy exercise program

A, C, E

A nurse is preparing an inservice education program for a group of nurses about dystocia involving problems with the passenger. Which of the following would the nurse most likely include as the most common problem? A) Macrosomia B) Breech presentation C) Persistent occiput posterior position D) Multifetal pregnancy

A,B,C

The nurse is teaching a pregnant woman about how to prevent contracting cytomegalovirus (CMV) during pregnancy. What tips would the nurse share with this client? Select all that apply. A. Do not share food or drinks with young children, especially if they are in daycare. B. If you develop any flu-like symptoms, notify your physician immediately to be evaluated for CMV. C. Wash your hands thoroughly with soap and water after touching saliva or urine. D. If you have CMV, it is suggested that you not breast-feed your infant. E. If you contract CMV, your doctor will give you some oral medicine to treat it.

A,B,C R: Cytomegalovirus (CMV) is a mild infection and women may not know they have contracted it. The problem arises when a pregnant woman contracts it during the first 20 weeks of gestation. Prevention is the key, so the nurse would reinforce handwashing, not eating or drinking from a container after a small child has done so, and notifying the physician if the client develops mild flu-like symptoms so she can be tested to rule out CMV.

A woman with known cardiac disease from childhood presents at the obstetrician's office 6 weeks' pregnant. What recommendations would the nurse make to the client to address the known cardiac problems for this pregnancy? Select all that apply. A. Plan periods of rest into the workday. B. Receive pneumococcal and influenza vaccines. Continue taking the scheduled warfarin. C. Let the physician know if you become short of breath or have a nighttime cough. D. Increase the amount of sodium in your diet to compensate for the expanding fluid needs of the fetus.

A,B,C R: Women with known heart conditions need to be closely followed by both the obstetrician and a cardiologist. Recommendations would include rest periods, reduction of stress, getting immunizations, and monitoring for heart failure as demonstrated by a nighttime cough and shortness of breath. Consuming more sodium in the diet is not recommended due of the potential of developing hypertension. Warfarin is contraindicated during pregnancy since it crosses the placental barrier and can cause spontaneous abortion, stillbirth or preterm birth.

A pregnant client has been diagnosed with gestational diabetes. Which are risk factors for developing gestational diabetes? Select all that apply. A. previous large for gestational age (LGA) infant B. hypertension C. maternal age less than 18 years D. obesity E. genitourinary tract abnormalities

A,B,D R: Obesity, hypertension, and a previous infant weighing more than 9 lb (4 kg) are risk factors for developing gestational diabetes. Maternal age less than 18 years and genitourinary tract abnormalities do not increase the risk of developing gestational diabetes.

The nurse is caring for a pregnant client who indicates that she is fond of meat, works with children, and has a pet cat. Which instructions should the nurse give this client to prevent toxoplasmosis? Select all that apply. A. Eat meat cooked to 160° F (71° C). B. Avoid cleaning the cat's litter box. C. Avoid contact with children when they have a cold. D. Avoid outdoor activities such as gardening. E. Keep the cat outdoors at all times.

A,B,D R:To minimize risk of toxoplasmosis, the nurse should instruct the client to eat meat that has been cooked to an internal temperature of 160° F (71° C) throughout and to avoid cleaning the cat's litter box or performing activities such as gardening. Avoiding children with colds is unreasonable when working with children, and contact with children with colds is not a cause of toxoplasmosis. The cat should be kept indoors to prevent it from hunting and eating birds or rodents.

The nurse is educating a client with type 1 diabetes about the complications associated with diabetes and pregnancy. Which problems would the nurse include in her teaching? Select all that apply. A. Decreased birth weight B. Polyhydramnios C. Increased risk of spontaneous abortion D. Hypertension E. Cystic fibrosis

A,B,D R:Women with pregestational diabetes, which is type 1 diabetes, are at a higher risk of having an infant with complications during the pregnancy and at delivery. Spontaneous abortion is higher in women who have pregestational diabetes. Also, they run a higher risk of having a pregnancy with polyhydramnios, and of developing maternal hypertension. The birth weight of an infant born to a mother with diabetes is increased, not decreased. Cystic fibrosis is not associated with maternal diabetes.

A woman who immigrated here from a third world country presents to the clinic to find out if she is pregnant. Which signs and/or symptoms would the nurse assess as possible indicators that she might have an active case of tuberculosis as well? Select all that apply. A. anorexia B. hemoptysis C. weight gain D. night sweats E. fatigue

A,B,D,E R: Women emigrating from developing countries are at high risk for tuberculosis. Clinical manifestations include fatigue, fever or night sweats, nonproductive cough, weakness, slow weight loss, anemia, hemoptysis, and anorexia.

A woman who immigrated here from a third world country presents to the clinic to find out if she is pregnant. Which signs and/or symptoms would the nurse assess as possible indicators that she might have an active case of tuberculosis as well? Select all that apply. A. anorexia B. hemoptysis C. weight gain D. night sweats E. fatigue

A,B,D,E Women emigrating from developing countries are at high risk for tuberculosis. Clinical manifestations include fatigue, fever or night sweats, nonproductive cough, weakness, slow weight loss, anemia, hemoptysis, and anorexia.

A nurse is caring for a client with cardiovascular disease who has just given birth. What nursing interventions should the nurse perform when caring for this client? Select all that apply. A. Assess for shortness of breath. B. Assess for edema and note any pitting. C. Monitor the client's hemoglobin and hematocrit. D. Auscultate heart sounds for abnormalities. E. Assess for a moist cough.

A,B,D,E R: The nurse should assess for possible fluid overload in a client with cardiovascular disease who has just given birth. Signs of fluid overload in the client who has just labored include cough, progressive dyspnea, edema, palpitations, and crackles in the lung bases. Hemoglobin and hematocrit levels are not affected by laboring of the client with cardiovascular disease.

A nurse is assessing a newborn and suspects that the mother may have abused alcohol during her pregnancy. The nurse suspect this based on which newborn findings? Select all that apply. A. small head circumference B. thin upper lip C. large inset eyes D. macrocephaly E. limb abnormality

A,B,E R: Characteristics of FASD include craniofacial dysmorphia (thin upper lip, small head circumference, and small eyes), IUGR, microcephaly, and congenital anomalies such as limb abnormalities and cardiac defects.

The nurse notifies the obstetrical team immediately because the nurse suspects that the pregnant woman may be exhibiting signs and symptoms of amniotic fluid embolism. Which findings would the nurse most likely assess? (Select all that apply.) A) Significant difficulty breathing B) Hypertension C) Tachycardia D) Pulmonary edema E) Bleeding with bruising

A,C,D,E The clinical appearance is varied, but most women report difficulty breathing. Other symptoms include hypotension, cyanosis, seizures, tachycardia, coagulation failure, disseminated intravascular coagulation, pulmonary edema, uterine atony with subsequent hemorrhage, adult respiratory distress syndrome, and cardiac arrest.

A nurse is conducting a class for a group of pregnant women about the risk of substance use during pregnancy. When discussing the effects of nicotine on a pregnancy, which complications would the nurse include? Select all that apply. A. premature rupture of membranes B. ectopic pregnancy C. macrosomia D. placenta previa E. spontaneous abortion

A,D,E R: Smoking increases the risk of spontaneous abortion, tubal ectopic pregnancy, preterm labor and birth, fetal growth restriction, stillbirth, premature rupture of membranes, low fetal iron stores, maternal hypertension, placenta previa, and abruptio placentae.

A nurse is conducting a class for a group of pregnant women about the risk of substance use during pregnancy. When discussing the effects of nicotine on a pregnancy, which complications would the nurse include? Select all that apply. A. premature rupture of membranes B. ectopic pregnancy C. macrosomia D. placenta previa E. spontaneous abortion

A,D,E Smoking increases the risk of spontaneous abortion, tubal ectopic pregnancy, preterm labor and birth, fetal growth restriction, stillbirth, premature rupture of membranes, low fetal iron stores, maternal hypertension, placenta previa, and abruptio placentae.

A woman in labor received an opioid close to the time of birth. The nurse would assess the newborn for which of the following? A) Respiratory depression B) Urinary retention C) Abdominal distention D) Hyperreflexia

A.

24. During a follow-up prenatal visit, a pregnant woman asks the nurse, "How long do you think I will be in labor?" Which response by the nurse would be most appropriate? A) "It's difficult to predict how your labor will progress, but we'll be there for you the entire time." B) "Since this is your first pregnancy, you can estimate it will be about 10 hours." C) "It will depend on how big the baby is when you go into labor."

A. It is difficult to predict how a labor will progress and therefore equally difficult to determine how long a woman's labor will last.

2. Which of the following would indicate to the nurse that the placenta is separating? A) Uterus becomes globular B) Fetal head is at vaginal opening C) Umbilical cord shortens D) Mucous plug is expelled

A. Placental separation is indicated by the uterus changing shape to globular and upward rising of the uterus. Additional signs include a sudden trickle of blood from the vaginal opening, and lengthening (not shortening) of the umbilical cord. - Expulsion of the mucous plug is a premonitory sign of labor.

7. The fetus of a nulliparous woman is in a shoulder presentation. The nurse would most likely prepare the client for which type of birth? A) Cesarean B) Vaginal C) Forceps-assisted D) Vacuum extraction

A. The fetus is in an oblique lie with the shoulder as the presenting part, necessitating a cesarean birth.

Because a pregnant clients diabetes has been poorly controlled throughout her pregnancy, the nurse would be alert for which of the following in the neonate at birth? A)Macrosomia B)Hyperglycemia C)Low birth weight D)Hypobilirubinemia

A. -Poorly controlled diabetes during pregnancy can result in macrosomia due to hyperinsulinemia stimulated by fetal hyperglycemia. -Typically the neonate is hypoglycemic due to the ongoing hyperinsulinemia that occurs after the placenta is removed. - Infants of diabetic women typically are large and are at risk for hyperbilirubinemia due to excessive red blood cell breakdown.

A 10-week pregnant woman with diabetes has a glycosylated hemoglobin (HbA1C. level of 13%. At this time the nurse should be most concerned about which of the following possible fetal outcomes? A)Congenital anomalies B)Incompetent cervix C)Placenta previa D)Abruptio placentae

A. HbA1c level of 13% indicates poor glucose control. This, in conjunction with the woman being in the first trimester, increases the risk for congenital anomalies in the fetus. Elevated glucose levels are not associated with incompetent cervix, placenta previa, or abruptio placentae.

A woman has just entered the second stage of labor. The nurse would focus care on which of the following? A) Encouraging the woman to push when she has a strong desire to do so B) Alleviating perineal discomfort with the application of ice packs C) Palpating the woman's fundus for position and firmness D) Completing the identification process of the newborn with the mother

A. During the second stage of labor, nursing interventions focus on motivating the woman, encouraging her to put all her efforts toward pushing.

19. The nurse is reviewing the monitoring strip of a woman in labor who is experiencing a contraction. The nurse notes the time the contraction takes from its onset to reach its highest intensity. The nurse interprets this time as which of the following? A) Increment B) Acme C) Peak D) Decrement

A. The time from the onset to the highest intensity corresponds to the increment.

The nurse is performing Leopold's maneuvers to determine fetal presentation, position, and lie. Which action would the nurse do first? A) Feel for the fetal buttocks or head while palpating the abdomen. B) Feel for the fetal back and limbs as the hands move laterally on the abdomen. C) Palpate for the presenting part in the area just above the symphysis pubis. D) Determine flexion by pressing downward toward the symphysis pubis.

A. - The first maneuver involves feeling for the buttocks and head. - Next the nurse palpates on which side the fetal back is located. - The third maneuver determines presentation and involves palpating the area just above the symphysis pubis. - The final maneuver determines attitude and involves applying downward pressure in the direction of the symphysis pubis.

The nurse is caring for a client experiencing a prolonged second stage of labor. The nurse would place priority on preparing the client for which intervention? A) a forceps and vacuum-assisted birth B) a cesarean birth C) a precipitous birth D) artificial rupture of membranes

A. - a forceps and vacuum-assisted birth A forceps-and-vacuum-assisted birth is required for the client having a prolonged second stage of labor. * The client may require a cesarean birth if the fetus cannot be delivered with assistance.

Which of the following is a priority when caring for a woman during the fourth stage of labor? A) Assessing the uterine fundus B) Offering fluids as indicated C) Encouraging the woman to void D) Assisting with perineal care

A. During the fourth stage of labor, a priority is to assess the woman's fundus to prevent postpartum hemorrhage.

A nurse palpates a woman's fundus to determine contraction intensity. Which of the following would be most appropriate for the nurse to use for palpation? A) Finger pads B) Palm of the hand C) Finger tips D) Back of the hand

A. To palpate the fundus for contraction intensity, the nurse would place the pads of the fingers on the fundus and describe how it feels.

A nurse is describing the different types of regional analgesia and anesthesia for labor to a group of pregnant women. Which statement by the group indicates that the teaching was successful? A) "We can get up and walk around after receiving combined spinal-epidural analgesia." B) "Higher anesthetic doses are needed for patient-controlled epidural analgesia. C) "A pudendal nerve block is highly effective for pain relief in the first stage of labor." D) "Local infiltration using lidocaine is an appropriate method for controlling contraction pain."

A. When compared with traditional epidural or spinal analgesia, which often keeps the woman lying in bed, combined spinal-epidural analgesia allows the woman to ambulate ("walking epidural").

A client's membranes spontaneously ruptured, as evidenced by a gush of clear fluid with a contraction. Which of the following would the nurse do next? A) Check the fetal heart rate. B) Perform a vaginal exam. C) Notify the physician immediately. D) Change the linen saver pad.

A. When membranes rupture, the PRIORITY focus is on assessing fetal heart rate first to identify a deceleration, which might indicate cord compression secondary to cord prolapse.

Which of the following practices would not be included in a physiologic birth? A. Early induction of labor <39 weeks' gestation B. Freedom of movement for the laboring woman C. Continuous presence and support throughout labor D. Encouraging spontaneous pushing when urge felt

A. since inducing labor artificially, rather than waiting for spontaneous labor to start doesn't provide for a physiologic birth. Nature should be allowed to take its course without artificial means to initiate labor.

A group of nursing students are reviewing information about cesarean birth. The students demonstrate understanding of the information when they identify which of the following as an appropriate indication? (Select all that apply) A) Active genital herpes infection B) Placenta previa C) Previous cesarean birth D) Prolonged labor E) Fetal distress

Active genital herpes infection Placenta previa Previous cesarean birth Fetal distress

Which of the following would be most appropriate for the nurse to suggest about pushing to a woman in the second stage of labor? A) "Lying flat with your head elevated on two pillows makes pushing easier." B) "Choose whatever method you feel most comfortable with for pushing." C) " Let me help you decide when it is time to start pushing." D) "Bear down like you're having a bowel movement with every contraction."

Ans. B "Choose whatever method you feel most comfortable with for pushing." * The role of the nurse should be to support the woman in her choice of pushing method and to encourage confidence in her maternal instinct of when and how to push.

After reviewing a client's history, which factor would the nurse identify as placing her at risk for gestational hypertension? A) Mother had gestational hypertension during pregnancy. B) Client has a twin sister. C) Sister-in-law had gestational hypertension. D) This is the client's second pregnancy.

Ans: A A family history of gestational hypertension, such as a mother or sister, is considered a risk factor for the client. Having a twin sister or having a sister-in-law with gestational hypertension would not increase the client's risk. If the client had a history of preeclampsia in her first pregnancy, then she would be at risk in her second pregnancy.

After teaching a woman who has had an evacuation for a hydatidiform mole (molar pregnancy) about her condition, which of the following statements indicates that the nurse's teaching was successful? A) "I will be sure to avoid getting pregnant for at least 1 year." B) "My intake of iron will have to be closely monitored for 6 months." C) "My blood pressure will continue to be increased for about 6 more months." D) "I won't use my birth control pills for at least a year or two."

Ans: A After evacuation of a hydatidiform mole, long-term follow-up is necessary to make sure any remaining trophoblastic tissue does not become malignant. Serial hCG levels are monitored closely for 1 year and the client is urged to avoid pregnancy for 1 year because it can interfere with the monitoring of hCG levels. Iron intake and blood pressure are not important aspects of follow-up after evacuation of a hydatidiform mole. Use of a reliable contraceptive is strongly recommended so that pregnancy is avoided.

When assessing a woman in her first trimester, which emotional response would the nurse most likely expect to find? A) Ambivalence B) Introversion C) Acceptance D) Emotional lability

Ans: A During the first trimester, the pregnant woman commonly experiences ambivalence, with conflicting feelings at the same time. Introversion heightens during the first and third trimesters when the woman's focus is on behaviors that will ensure a safe and healthy pregnancy outcome. Acceptance usually occurs during the second trimester. Emotional lability (mood swings) is characteristic throughout a woman's pregnancy.

Assessment of a pregnant woman reveals a pigmented line down the middle of her abdomen. The nurse documents this as which of the following? A) Linea nigra B) Striae gravidarum C) Melasma D) Vascular spiders

Ans: A Linea nigra refers to the darkened line of pigmentation down the middle of the abdomen in pregnant women. Striae gravidarum refers to stretch marks, irregular reddish streaks on the abdomen, breasts, and buttocks. Melasma refers to the increased pigmentation on the face, also known as the mask of pregnancy. Vascular spiders are small, spiderlike blood vessels that appear usually above the waist and on the neck, thorax, face, and arms.

Which of the following would the nurse have readily available for a client who is receiving magnesium sulfate to treat severe preeclampsia? A) Calcium gluconate B) Potassium chloride C) Ferrous sulfate D) Calcium carbonate

Ans: A The antidote for magnesium sulfate is calcium gluconate, and this should be readily available in case the woman has signs and symptoms of magnesium toxicity.

In a client's seventh month of pregnancy, she reports feeling ìdizzy, like I'm going to pass out, when I lie down flat on my back.î The nurse integrates which of the following in to the explanation? A) Pressure of the gravid uterus on the vena cava B) A 50% increase in blood volume C) Physiologic anemia due to hemoglobin decrease D) Pressure of the presenting fetal part on the diaphragm

Ans: A The client is describing symptoms of supine hypotension syndrome, which occurs when the heavy gravid uterus falls back against the superior vena cava in the supine position. The vena cava is compressed, reducing venous return, cardiac output, and blood pressure, with increased orthostasis. The increased blood volume and physiologic anemia are unrelated to the client's symptoms. Pressure on the diaphragm would lead to dyspnea.

In a woman who is suspected of having a ruptured ectopic pregnancy, the nurse would expect to assess for which of the following as a priority? A) Hemorrhage B) Jaundice C) Edema D) Infection

Ans: A With a ruptured ectopic pregnancy, the woman is at high risk for hemorrhage. Jaundice, edema, and infection are not associated with a ruptured ectopic pregnancy.

13. Assessment of a fetus identifies the buttocks as the presenting part, with the legs extended upward. The nurse identifies this as which type of breech presentation? A) Frank B) Full C) Complete D) Footling

Ans: A Feedback: In a frank breech, the buttocks present first, with both legs extended up toward the face. In a full or complete breech, the fetus sits cross-legged above the cervix. In a footling breech, one or both legs are presenting.

A pregnant woman undergoes maternal serum alpha-fetoprotein (MSAFP) testing at 16 to 18 weeks' gestation. Which of the following would the nurse suspect if the woman's level is decreased? A) Down syndrome B) Sickle-cell anemia C) Cardiac defects D) Open neural tube defect

Ans: A Feedback: Decreased levels might indicate Down syndrome or trisomy 18. Sickle cell anemia may be identified by chorionic villus sampling. MSAFP levels would be increased with cardiac defects, such as tetralogy of Fallot. A triple marker test would be used to determine an open neural tube defect.

A nurse is describing advances in genetics to a group of students. Which of the following would the nurse least likely include? A) Genetic diagnosis is now available as early as the second trimester. B) Genetic testing can identify presymptomatic conditions in children. C) Gene therapy can be used to repair missing genes with normal ones. D) Genetic agents may be used in the future to replace drugs.

Ans: A Feedback: Genetic diagnosis is now possible very early in pregnancy (see Evidence-Based Practice 10.1). Genetic testing can now identify presymptomatic conditions in children and adults. Gene therapy can be used to replace or repair defective or missing genes with normal ones. Gene therapy has been used for a variety of disorders, including cystic fibrosis, melanoma, diabetes, HIV, and hepatitis (Tamura, Kamuma, Nakazato, et al. 2010). The potential exists for creation of increased intelligence and size through genetic intervention. Recent research using gene therapy shows promise for the generation of insulin-producing cells to cure diabetes (Calne, Gan, & Lee 2010). In the future, genetic agents may replace drugs, general surgery may be replaced by gene surgery, and genetic intervention may replace radiation.

When describing genetic disorders to a group of childbearing couples, the nurse would identify which as an example of an autosomal dominant inheritance disorder? A) Huntington's disease B) Sickle cell disease C) Phenylketonuria D) Cystic fibrosis

Ans: A Feedback: Huntington's disease is an example of an autosomal dominant inheritance disorder. Sickle cell disease, phenylketonuria, and cystic fibrosis are examples of autosomal recessive inheritance disorders.

After teaching a group of students about fetal development, the instructor determines that the teaching was successful when the students identify which of the following as providing the barrier to other sperm after fertilization? A) Zona pellucida B) Zygote C) Cleavage D) Morula

Ans: A Feedback: The zona pellucida is the clear protein layer that acts as a barrier to other sperm once one sperm enters the ovum for fertilization. The zygote refers to the union of the nuclei of the ovum and sperm resulting in the diploid number of chromosomes. Cleavage is another term for mitosis. The morula is the result of four cleavages leading to 16 cells that appear as a solid ball of cells. The morula reaches the uterine cavity about 72 hours after fertilization.

A group of students are reviewing the signs of pregnancy. The students demonstrate understanding of the information when they identify which as presumptive signs? (Select all that apply. A) Amenorrhea B) Nausea C) Abdominal enlargement D) Braxton-Hicks contractions E) Fetal heart sounds

Ans: A, B Presumptive signs include amenorrhea, nausea, breast tenderness, urinary frequency and fatigue. Abdominal enlargement and Braxton-Hicks contractions are probable signs of pregnancy. Fetal heart sounds are a positive sign of pregnancy.

A group of students are reviewing information about genetic inheritance. The students demonstrate understanding of the information when they identify which of the following as an example of an autosomal recessive disorder? (Select all that apply.) A) Cystic fibrosis B) Phenylketonuria C) Tay-Sachs disease D) Polycystic kidney disease E) Achondroplasia

Ans: A, B, C Feedback: Examples of autosomal recessive disorders include cystic fibrosis, phenylketonuria, and Tay-Sachs disease. Polycystic kidney disease and achondroplasia are examples of autosomal dominant diseases.

While assessing a pregnant woman, the nurse suspects that the client may be at risk for hydramnios based on which of the following? (Select all that apply.) A) History of diabetes B) Complaints of shortness of breath C) Identifiable fetal parts on abdominal palpation D) Difficulty obtaining fetal heart rate E) Fundal height below that for expected gestational age

Ans: A, B, D Factors such as maternal diabetes or multiple gestations place the woman at risk for hydramnios. In addition, there is a discrepancy between fundal height and gestational age, such that a rapid growth of the uterus is noted. Shortness of breath may result from overstretching of the uterus due to the increased amount of amniotic fluid. Often, fetal parts are difficult to palpate and fetal heart rate is difficult to obtain because of the excess fluid present.

A nurse is assessing a pregnant woman on a routine checkup. When assessing the woman's gastrointestinal tract, which of the following would the nurse expect to find? (Select all that apply. A) Hyperemic gums B) Increased peristalsis C) Complaints of bloating D) Heartburn E) Nausea

Ans: A, C, D, E Gastrointestinal system changes include hyperemic gums due to estrogen and increased proliferation of blood vessels and circulation to the mouth; slowed peristalsis; acid indigestion and heartburn; bloating and nausea and vomiting.

18. A nurse is preparing a class for pregnant women about labor and birth. When describing the typical movements that the fetus goes through as it travels through the passageway, which of the following would the nurse most likely include? (Select all that apply.) A) Internal rotation B) Abduction C) Descent D) Pronation E) Flexion

Ans: A, C, E Feedback: The positional changes that occur as the fetus moves through the passageway are called the cardinal movements of labor and include engagement, descent, flexion, internal rotation, extension, external rotation, and expulsion. The fetus does not undergo abduction or pronation.

A nurse is discussing fetal development with a pregnant woman. The woman is 12 weeks pregnant and asks, "What's happening with my baby?" Which of the following would the nurse integrate into the response? (Select all that apply.) A) Continued sexual differentiation B) Eyebrows forming C) Startle reflex present D) Digestive system becoming active E) Lanugo present on the head

Ans: A, D Feedback: At 12 weeks, sexual differentiation continues and the digestive system shows activity. Eyebrows form and startle reflex is present between weeks 21 and 24. Lanugo on the head appears about weeks 13-16.

A woman just delivered a healthy term newborn. Upon assessing the umbilical cord, the nurse would identify which of the following as normal? (Select all that apply.) A) One vein B) Two veins C) One artery D) Two arteries E) One ligament F) Two ligaments

Ans: A, D Feedback: The normal umbilical cord contains one large vein and two small arteries.

A nurse suspects that a pregnant client may be experiencing abruption placenta based on assessment of which of the following? (Select all that apply.) A) Dark red vaginal bleeding B) Insidious onset C) Absence of pain D) Rigid uterus E) Absent fetal heart tones

Ans: A, D, E Feedback: Assessment findings associated with abruption placenta include a sudden onset, with concealed or visible bleeding, dark red bleeding, constant pain or uterine tenderness on palpation, firm to rigid uterine tone, and fetal distress or absent fetal heart tones.

A woman comes to the prenatal clinic suspecting that she is pregnant, and assessment reveals probable signs of pregnancy. Which of the following would be included as part of this assessment? (Select all that apply A) Positive pregnancy test B) Ultrasound visualization of the fetus C) Auscultation of a fetal heart beat D) Ballottement E) Absence of menstruation F) Softening of the cervix

Ans: A, D, F Probable signs of pregnancy include a positive pregnancy test, ballottement, and softening of the cervix (Goodell's sign). Ultrasound visualization of the fetus, auscultation of a fetal heart beat, and palpation of fetal movements are considered positive signs of pregnancy. Absence of menstruation is a presumptive sign of pregnancy.

A woman is receiving magnesium sulfate as part of her treatment for severe preeclampsia. The nurse is monitoring the woman's serum magnesium levels. Which level would the nurse identify as therapeutic? A) 3.3 mEq/L B) 6.1 mEq/L C) 8.4 mEq/L D) 10.8 mEq/L

Ans: B Although exact levels may vary among agencies, serum magnesium levels ranging from 4 to 7 mEq/L are considered therapeutic, whereas levels more than 8 mEq/dL are generally considered toxic.

A woman with gestational hypertension experiences a seizure. Which of the following would be the priority? A) Fluid replacement B) Oxygenation C) Control of hypertension D) Delivery of the fetus

Ans: B As with any seizure, the priority is to clear the airway and maintain adequate oxygenation both to the mother and the fetus. Fluids and control of hypertension are addressed once the airway and oxygenation are maintained. Delivery of fetus is determined once the seizures are controlled and the woman is stable.

A woman with placenta previa is being treated with expectant management. The woman and fetus are stable. The nurse is assessing the woman for possible discharge home. Which statement by the woman would suggest to the nurse that home care might be inappropriate? A) "My mother lives next door and can drive me here if necessary." B) "I have a toddler and preschooler at home who need my attention." C) "I know to call my health care provider right away if I start to bleed again." D) "I realize the importance of following the instructions for my care."

Ans: B Having a toddler and preschooler at home needing attention suggest that the woman would have difficulty maintaining bed rest at home. Therefore, expectant management at home may not be appropriate. Expectant management is appropriate if the mother and fetus are both stable, there is no active bleeding, the client has readily available access to reliable transportation, and can comprehend instructions.

A nurse strongly encourages a pregnant client to avoid eating swordfish and tilefish because these fish contain which of the following? A) Excess folic acid, which could increase the risk for neural tube defects B) Mercury, which could harm the developing fetus if eaten in large amounts C) Lactose, which leads to abdominal discomfort, gas, and diarrhea D) Low-quality protein that does not meet the woman's requirements

Ans: B Nearly all fish and shellfish contain traces of mercury and some contain higher levels of mercury that may harm the developing fetus if ingested by pregnant women in large amounts. Among these fish are shark, swordfish, king mackerel, and tilefish. Folic acid is found in dark green vegetables, baked beans, black-eyed peas, citrus fruits, peanuts, and liver. Folic acid supplements are needed to prevent neural tube defects. Women who are lactose intolerant experience abdominal discomfort, gas, and diarrhea if they ingest foods containing lactose. Fish and shellfish are an important part of a healthy diet because they contain high-quality proteins, are low in saturated fat, and contain omega-3 fatty acids.

Assessment of a pregnant woman reveals that she compulsively craves ice. The nurse documents this finding as which of the following? A) Quickening B) Pica C) Ballottement D) Linea nigra

Ans: B Pica refers to the compulsive ingestion of nonfood substances such as ice. Quickening refers to the mother's sensation of fetal movement. Ballottement refers to the feeling of rebound from a floating fetus when an examiner pushes against the woman's cervix during a pelvic examination. Linea nigra refers to the pigmented line that develops in the middle of the woman's abdomen.

A primiparous client is being seen in the clinic for her first prenatal visit. It is determined that she is 11 weeks pregnant. The nurse develops a teaching plan to educate the client about what she will most likely experience during this period. Which of the following would the nurse include? A) Ankle edema B) Urinary frequency C) Backache D) Hemorrhoids

Ans: B The client is in her first trimester and would most likely experience urinary frequency as the growing uterus presses on the bladder. Ankle edema, backache, and hemorrhoids would be more common during the later stages of pregnancy.

A pregnant woman comes to the clinic and tells the nurse that she has been having a whitish vaginal discharge. The nurse suspects vulvovaginal candidiasis, based on which assessment finding? A) Fever B) Vaginal itching C) Urinary frequency D) Incontinence

Ans: B Vaginal secretions become more acidic, white, and thick during pregnancy. Most women experience an increase in a whitish vaginal discharge, called leukorrhea. This is normal except when it is accompanied by itching and irritation, possibly suggesting Candida albicans, a monilial vaginitis, which is a very common occurrence in this glycogen-rich environment. Fever would suggest a more serious infection. Urinary frequency occurs commonly in the first trimester, disappears during the second trimester, and reappears during the third trimester. Incontinence would not be associated with a vulvovaginal candidiasis. Incontinence would require additional evaluation.

The nurse is developing a plan of care for a woman who is pregnant with twins. The nurse includes interventions focusing on which of the following because of the woman's increased risk? A) Oligohydramnios B) Preeclampsia C) Post-term labor D) Chorioamnionitis

Ans: B Women with multiple gestations are at high risk for preeclampsia, preterm labor, hydramnios, hyperemesis gravidarum, anemia, and antepartal hemorrhage. There is no association between multiple gestations and the development of chorioamnionitis.

The nurse is discussing the insulin needs of a primiparous client with diabetes who has been using insulin for the past few years. The nurse informs the client that her insulin needs will increase during pregnancy based on the nurse's understanding that the placenta produces: A) hCG, which increases maternal glucose levels B) hPL, which deceases the effectiveness of insulin C) Estriol, which interferes with insulin crossing the placenta D) Relaxin, which decreases the amount of insulin produced

Ans: B hPL acts as an antagonist to insulin, so the mother must produce more insulin to overcome this resistance. If the mother has diabetes, then her insulin need would most likely increase to meet this demand. hCG does not affect insulin and glucose level. Estrogen, not estriol, is believed to oppose insulin. In addition, insulin does not cross the placenta. Relaxin is not associated with insulin resistance.

11. After teaching a group of students about the maternal bony pelvis, which statement by the group indicates that the teaching was successful? A) The bony pelvis plays a lesser role during labor than soft tissue. B) The pelvic outlet is associated with the true pelvis. C) The false pelvis lies below the imaginary linea terminalis. D) The false pelvis is the passageway through which the fetus travels.

Ans: B Feedback: The maternal bony pelvis consists of the true and false portions. The true pelvis is made up of three planes—the inlet, the mid pelvis, and the outlet. The bony pelvis is the more important part of the passageway because it is relatively unyielding. The false pelvis lies above the imaginary linea terminalis. The true pelvis is the bony passageway through which the fetus must travel.

Assessment of a pregnant woman reveals oligohydramnios. The nurse would be alert for the development of which of the following? A) Maternal diabetes B) Placental insufficiency C) Neural tube defects D) Fetal gastrointestinal malformations

Ans: B Feedback: A deficiency of amniotic fluid, oligohydramnios, is associated with uteroplacental insufficiency and fetal renal abnormalities. Excess amniotic fluid is associated with maternal diabetes, neural tube defects, and malformations of the gastrointestinal tract and central nervous system.

After teaching a group of students about fetal development, the instructor determines that the teaching was successful when the students identify which of the following as essential for fetal lung development? A) Umbilical cord B) Amniotic fluid C) Placenta D) Trophoblasts

Ans: B Feedback: Amniotic fluid is essential for fetal growth and development, especially fetal lung development. The umbilical cord is the lifeline from the mother to the growing embryo. The placenta serves as the interface between the mother and developing fetus. It secretes hormones and supplies the fetus with nutrients and oxygen needed for growth. The trophoblasts differentiate into all the cells that form that placenta.

After the nurse describes fetal circulation to a pregnant woman, the woman asks why her fetus has a different circulation pattern than hers. In planning a response, the nurse integrates understanding of which of the following? A) Fetal blood is thicker than that of adults and needs different pathways. B) Fetal circulation carries highly oxygenated blood to vital areas first. C) Fetal blood has a higher oxygen saturation and circulates more slowly. D) Fetal heart rates are rapid and circulation time is double that of adults.

Ans: B Feedback: Fetal circulation functions to carry highly oxygenated blood to vital areas first while shunting it away from less vital ones. Fetal blood is not thicker than that of adults. Large volumes of oxygenated blood are not needed because the placenta essentially takes over the functions of the lung and liver during fetal life. Although fetal heart rates normally range from 120 to 160 beats per minute, circulation time is not doubled.

A woman is scheduled to undergo fetal nuchal translucency testing. Which of the following would the nurse include when describing this test? A) "A needle will be inserted directly into the fetus's umbilical vessel." B) "You'll have an intravaginal ultrasound to measure fluid in the fetus." C) "The doctor will take a sample of fluid from your bag of waters." D) "A small piece of tissue from the fetal part of the placenta is taken."

Ans: B Feedback: Fetal nuchal translucency testing involves an intravaginal ultrasound that measures fluid collection in the subcutaneous space between the skin and cervical spine of the fetus. Insertion of a needle into the fetus's umbilical vessel describes percutaneous umbilical blood sampling. Taking a sample of fluid from the amniotic sac (bag of waters) describes an amniocentesis. Obtaining a small tissue specimen from the fetal part of the placenta describes chorionic villus sampling.

A nursing instructor is preparing a teaching plan for a group of nursing students about the potential for misuse of genetic discoveries and advances. Which the following would the instructor most likely include? A) Gene replacement therapy for defective genes B) Individual risk profiling and confidentiality C) Greater emphasis on the causes of diseases D) Slower diagnosis of specific diseases

Ans: B Feedback: Individual risk profiling based on an individual's genetic makeup can raise issues related to privacy and confidentiality. Gene replacement therapy for defective genes and a greater emphasis on looking at the causes of disease are considered benefits associated with genetic advances. Rapid, more specific diagnosis of diseases would be possible.

A nurse is teaching a class on X-linked recessive disorders. Which of the following statements would the nurse most likely include? A) Males are typically carriers of the disorders. B) No male-to-male transmission occurs. C) Daughters are more commonly affected with the disorder. D) Both sons and daughters have a 50% risk of the disorder.

Ans: B Feedback: Most X-linked disorders demonstrate a recessive pattern of inheritance. Males are more affected than females. A male has only one X chromosome and all the genes on his X chromosome will be expressed, whereas a female will usually need both X chromosomes to carry the disease. There is no male-to-male transmission (since no X chromosome from the male is transmitted to male offspring), but any man who is affected will have carrier daughters. If a woman is a carrier, there is a 50% chance that her sons will be affected and a 50% chance that her daughters will be carriers.

The nurse is developing a presentation for a community group of young adults discussing fetal development and pregnancy. The nurse would identify that the sex of offspring is determined at the time of: A) Meiosis B) Fertilization C) Formation of morula D) Oogenesis

Ans: B Feedback: Sex determination occurs at the time of fertilization. Meiosis refers to cell division resulting in the formation of an ovum or sperm with half the number of chromosomes. The morula develops after a series of four cleavages following the formation of the zygote. Oogenesis refers to the development of a mature ovum, which has half the number of chromosomes.

When describing the structures involved in fetal circulation, the nursing instructor describes which structure as the opening between the right and left atrium? A) Ductus venosus B) Foramen ovale C) Ductus arteriosus D) Umbilical artery

Ans: B Feedback: The foramen ovale is the opening between the right and left atrium. The ductus venosus connects the umbilical vein to the inferior vena cava. The ductus arteriosus connects the main pulmonary artery to the aorta. The umbilical artery carries blood to the placenta.

During a prenatal class for a group of new mothers, the nurse is describing the hormones produced by the placenta. Which of the following would the nurse include? (Select all that apply.) A) Prolactin B) Estriol C) Relaxin D) Progestin E) Human chorionic somatomammotropin

Ans: B, C, D, E Feedback: Estriol, relaxin, progestin, and human chorionic somatomammotropin are secreted by the placenta. Prolactin is secreted after delivery for breast-feeding.

A nursing student is reviewing an article about preterm premature rupture of membranes. Which of the following would the student expect to find as factor placing a woman at high risk for this condition? (Select all that apply.) A) High body mass index B) Urinary tract infection C) Low socioeconomic status D) Single gestations E) Smoking

Ans: B, C, E High-risk factors associated with preterm PROM include low socioeconomic status, multiple gestation, low body mass index, tobacco use, preterm labor history, placenta previa, abruptio placenta, urinary tract infection, vaginal bleeding at any time in pregnancy, cerclage, and amniocentesis.

A pregnant woman is admitted with premature rupture of the membranes. The nurse is assessing the woman closely for possible infection. Which of the following would lead the nurse to suspect that the woman is developing an infection? (Select all that apply.) A) Fetal bradycardia B) Abdominal tenderness C) Elevated maternal pulse rate D) Decreased C-reactive protein levels E) Cloudy malodorous fluid

Ans: B, C, E Possible signs of infection associated with premature rupture of membranes include elevation of maternal temperature and pulse rate, abdominal/uterine tenderness, fetal tachycardia over 160 bpm, elevated white blood cell count and C-reactive protein levels, and cloudy, foul-smelling amniotic fluid.

A nurse is assessing a child with Klinefelter's syndrome. Which of the following would the nurse expect to assess? (Select all that apply.) A) Gross mental retardation B) Long arms C) Profuse body hair D) Gynecomastia E) Enlarged testicles

Ans: B, D Feedback: Manifestations of Klinefelter's syndrome include mild mental retardation, small testicles, infertility, long arms and legs, gynecomastia, scant facial and body hair, and decreased libido.

A nurse is developing a teaching plan about nutrition for a group of pregnant women. Which of the following would the nurse include in the discussion? (Select all that apply. A) Keep weight gain to 15 lb B) Eat three meals with snacking C) Limit the use of salt in cooking D) Avoid using diuretics E) Participate in physical activity

Ans: B, D, E To promote optimal nutrition, the nurse would recommend gradual and steady weight gain based on the client's prepregnant weight, eating three meals with one or two snacks daily, not restricting the use of salt unless instructed to do so by the health care provider, avoiding the use of diuretics, and participating in reasonable physical activity daily.

After teaching a class on the stages of fetal development, the instructor determines that the teaching was successful when the students identify which of the following as a stage? (Select all that apply.) A) Placental B) Preembryonic C) Umbilical D) Embryonic E) Fetal

Ans: B, D, E Feedback: The three stages of fetal development are the preembryonic, embryonic, and fetal stage. Placental and umbilical are not stages of fetal development.

A gravida 2 para 1 client in the 10th week of her pregnancy says to the nurse, ìI've never urinated as often as I have for the past three weeks.î Which response would be most appropriate for the nurse to make? A) ìHaving to urinate so often is annoying. I suggest that you watch how much fluid you are drinking and limit it.î B) ìYou shouldn't be urinating this frequently now; it usually stops by the time you're eight weeks pregnant. Is there anything else bothering you?î C) ìBy the time you are 12 weeks pregnant, this frequent urination should no longer be a problem, but it is likely to return toward the end of your pregnancy.î D) ìWomen having their second child generally don't have frequent urination. Are you experiencing any burning sensations?î

Ans: C As the uterus grows, it presses on the urinary bladder, causing the increased frequency of urination during the first trimester. This complaint lessens during the second trimester only to reappear in the third trimester as the fetus begins to descend into the pelvis, causing pressure on the bladder.

A woman in her second trimester comes for a follow-up visit and says to the nurse, ìI feel like I'm on an emotional roller-coaster.î Which response by the nurse would be most appropriate? A) How often has this been happening to you? B) Maybe you need some medication to level things out. C) Mood swings are completely normal during pregnancy. D) Have you been experiencing any thoughts of harming yourself?

Ans: C Emotional lability is characteristic throughout most pregnancies. One moment a woman can feel great joy, and within a short time she can feel shock and disbelief. Frequently, pregnant women will start to cry without any apparent cause. Some women feel as though they are riding an emotional roller-coaster. These extremes in emotion can make it difficult for partners and family members to communicate with the pregnant woman without placing blame on themselves for their mood changes. Clear explanations about how common mood swings are during pregnancy are essential.

A woman hospitalized with severe preeclampsia is being treated with hydralazine to control blood pressure. Which of the following would the lead the nurse to suspect that the client is having an adverse effect associated with this drug? A) Gastrointestinal bleeding B) Blurred vision C) Tachycardia D) Sweating

Ans: C Hydralazine reduces blood pressure but is associated with adverse effects such as palpitation, tachycardia, headache, anorexia, nausea, vomiting, and diarrhea. It does not cause gastrointestinal bleeding, blurred vision, or sweating. Magnesium sulfate may cause sweating.

Which of the following data on a client's health history would the nurse identify as contributing to the client's risk for an ectopic pregnancy? A) Use of oral contraceptives for 5 years B) Ovarian cyst 2 years ago C) Recurrent pelvic infections D) Heavy, irregular menses

Ans: C In the general population, most cases of ectopic pregnancy are the result of tubal scarring secondary to pelvic inflammatory disease. Oral contraceptives, ovarian cysts, and heavy, irregular menses are not considered risk factors for ectopic pregnancy.

The nurse is assessing a pregnant woman in the second trimester. Which of the following tasks would indicate to the nurse that the client is incorporating the maternal role into her personality? A) The woman demonstrates concern for herself and her fetus as a unit. B) The client identifies what she must give up to assume her new role. C) The woman acknowledges the fetus as a separate entity within her. D) The client demonstrates unconditional acceptance without rejection.

Ans: C Incorporation of the maternal role into her personality indicates acceptance by the pregnant woman. In doing so, the woman becomes able to identify the fetus as a separate individual. Demonstrating concern for herself and her fetus as a unit is associated with introversion and more commonly occurs during the third trimester. Identification of what the mother must give up to assume the new role occurs during the first trimester. Demonstrating unconditional acceptance without rejection occurs during the third trimester.

A woman suspecting she is pregnant asks the nurse about which signs would confirm her pregnancy. The nurse would explain that which of the following would confirm the pregnancy? A) Absence of menstrual period B) Abdominal enlargement C) Palpable fetal movement D) Morning sickness

Ans: C Only positive signs of pregnancy would confirm a pregnancy. The positive signs of pregnancy confirm that a fetus is growing in the uterus. Visualizing the fetus by ultrasound, palpating for fetal movements, and hearing a fetal heartbeat are all signs that make the pregnancy a certainty. Absence of menstrual period and morning sickness are presumptive signs, which can be due to conditions other than pregnancy. Abdominal enlargement is a probable sign.

A woman with hyperemesis gravidarum asks the nurse about suggestions to minimize nausea and vomiting. Which suggestion would be most appropriate for the nurse to make? A) "Make sure that anything around your waist is quite snug." B) "Try to eat three large meals a day with less snacking." C) "Drink fluids in between meals rather than with meals." D) "Lie down for about an hour after you eat"

Ans: C Suggestions to minimize nausea and vomiting include avoiding tight waistbands to minimize pressure on the abdomen, eating small frequent meals throughout the day, separating fluids from solids by consuming fluids in between meals; and avoiding lying down or reclining for at least 2 hours after eating.

Upon entering the room of a client who has had a spontaneous abortion, the nurse observes the client crying. Which of the following responses by the nurse would be most appropriate? A) "Why are you crying?" B) "Will a pill help your pain?" C) "I'm sorry you lost your baby." D) "A baby still wasn't formed in your uterus."

Ans: C Telling the client that the nurse is sorry for the loss acknowledges the loss to the woman, validates her feelings, and brings the loss into reality. Asking why the client is crying is ineffective at this time. Offering a pill for the pain ignores the client's feelings. Telling the client that the baby wasn't formed is inappropriate and discounts any feelings or beliefs that the client has.

After teaching a group of nursing students about the possible causes of spontaneous abortion, the instructor determines that the teaching was successful when the students identify which of the following as the most common cause of first trimester abortions? A) Maternal disease B) Cervical insufficiency C) Fetal genetic abnormalities D) Uterine fibroids

Ans: C The causes of spontaneous abortion are varied and often unknown. The most common cause for first-trimester abortions is fetal genetic abnormalities, usually unrelated to the mother. Chromosomal abnormalities are more likely causes in first trimester and maternal disease is more likely in the second trimester. Those occurring during the second trimester are more likely related to maternal conditions, such as cervical insufficiency, congenital or acquired anomaly of the uterine cavity (uterine septum or fibroids), hypothyroidism, diabetes mellitus, chronic nephritis, use of crack cocaine, inherited and acquired thrombophilias, lupus, polycystic ovary syndrome, severe hypertension and acute infection such as rubella virus, cytomegalovirus, herpes simplex virus, bacterial vaginosis, and toxoplasmosis.

A woman is at 20 weeks' gestation. The nurse would expect to find the fundus at which of the following? A) Just above the symphysis pubis B) Mid-way between the pubis and umbilicus C) At the level of the umbilicus D) Mid-way between the umbilicus and xiphoid process

Ans: C The uterus, which starts as a pear-shaped organ, becomes ovoid as length increases over width. By 20 weeks' gestation, the fundus, or top of the uterus, is at the level of the umbilicus and measures 20 cm. A monthly measurement of the height of the top of the uterus in centimeters, which corresponds to the number of gestational weeks, is commonly used to date the pregnancy.

A client with hyperemesis gravidarum is admitted to the facility after being cared for at home without success. Which of the following would the nurse expect to include in the client's plan of care? A) Clear liquid diet B) Total parenteral nutrition C) Nothing by mouth D) Administration of labetalol

Ans: C Typically, on admission, the woman with hyperemesis has oral food and fluids withheld for the first 24 to 36 hours to rest the gut and receives parenteral fluids to rehydrate and reduce the symptoms. Once the condition stabilizes, oral intake is gradually increased. Total parenteral nutrition may be used if the client's condition does not improve with several days of bed rest, gut rest, IV fluids, and antiemetics. Labetalol is an antihypertensive agent that may be used to treat gestational hypertension, not hyperemesis.

The health care provider orders PGE2 for a woman to help evacuate the uterus following a spontaneous abortion. Which of the following would be most important for the nurse to do? A) Use clean technique to administer the drug. B) Keep the gel cool until ready to use. C) Maintain the client for 1/2 hour after administration. D) Administer intramuscularly into the deltoid area.

Ans: C When PGE2 is ordered, the gel should come to room temperature before administering it. Sterile technique should be used and the client should remain supine for 30 minutes after administration. RhoGAM is administered intramuscularly into the deltoid area.

A client is diagnosed with gestational hypertension and is receiving magnesium sulfate. Which finding would the nurse interpret as indicating a therapeutic level of medication? A) Urinary output of 20 mL per hour B) Respiratory rate of 10 breaths/minute C) Deep tendons reflexes 2+ D) Difficulty in arousing

Ans: C With magnesium sulfate, deep tendon reflexes of 2+ would be considered normal and therefore a therapeutic level of the drug. Urinary output of less than 30 mL, a respiratory rate of less than 12 breaths/minute, and a diminished level of consciousness would indicate magnesium toxicity.

Which of the following changes in the musculoskeletal system would the nurse mention when teaching a group of pregnant women about the physiologic changes of pregnancy? A) Ligament tightening B) Decreased swayback C) Increased lordosis D) Joint contraction

Ans: C With pregnancy, the woman's center of gravity shifts forward, requiring a realignment of the spinal curvatures. There is an increase in the normal lumbosacral curve (lordosis). Ligaments of the sacroiliac joints and pubis symphysis soften and stretch. Increased swayback and an upper spine extension to compensate for the enlarging abdomen occur. Joint relaxation and increased mobility occur due to the influence of the hormones relaxin and progesterone.

During a vaginal exam, the nurse notes that the cervix has a bluish color. The nurse documents this finding as: A) Hegar's sign B) Goodell's sign C) Chadwick's sign D) Ortolani's sign

Ans: C Bluish coloration of the cervix is termed Chadwick's sign. Hegar's sign refers to the softening of the lower uterine segment or isthmus. Goodell's sign refers to the softening of the cervix. Ortolani's sign is a maneuver done to identify developmental dysplasia of the hip in infants.

When describing amniotic fluid to a pregnant woman, the nurse would include which of the following? A) "This fluid acts as transport mechanism for oxygen and nutrients." B) "The fluid is mostly protein to provide nourishment to your baby." C) "This fluid acts as a cushion to help to protect your baby from injury." D) "The amount of fluid remains fairly constant throughout the pregnancy."

Ans: C Feedback: Amniotic fluid protects the floating embryo and cushions the fetus from trauma. The placenta acts as a transport mechanism for oxygen and nutrients. Amniotic fluid is primarily water with some organic matter. Throughout pregnancy, amniotic fluid volume fluctuates.

While talking with a pregnant woman who has undergone genetic testing, the woman informs the nurse that her baby will be born with Down syndrome. The nurse understands that Down syndrome is an example of: A) Multifactorial inheritance B) X-linked recessive inheritance C) Trisomy numeric abnormality D) Chromosomal deletion

Ans: C Feedback: Down syndrome is an example of a chromosomal abnormality involving the number of chromosomes (trisomy numeric abnormality), in particular chromosome 21, in which the individual has three copies of that chromosome. Multifactorial inheritance gives rise to disorders such as cleft lip, congenital heart disease, neural tube defects, and pyloric stenosis. X-linked recessive inheritance is associated with disorders such as hemophilia. Chromosomal deletion is involved with disorders such as cri du chat syndrome.

Prenatal testing is used to assess for genetic risks and to identify genetic disorders. In explaining to a couple about an elevated alpha-fetoprotein screening test result, the nurse would discuss the need for: A) Special care needed for a Down syndrome infant B) A more specific determination of the acid-base status C) Further, more definitive evaluations to conclude anything D) Immediate termination of the pregnancy based on results

Ans: C Feedback: Increased maternal serum alpha fetoprotein levels may indicate a neural tube defect, Turner syndrome, tetralogy of Fallot, multiple gestation, omphalocele, gastroschisis, or hydrocephaly. Therefore, additional information and more specific determinations need to be done before any conclusion can be made. Down syndrome is associated with decreased maternal serum alpha fetoprotein levels. This type of testing provides no information about the acid-base status of the fetus. Immediate termination is not warranted; more information is needed.

A nurse is teaching a pregnant woman with preterm premature rupture of membranes who is about to be discharged home about caring for herself. Which statement by the woman indicates a need for additional teaching? A) "I need to keep a close eye on how active my baby is each day." B) "I need to call my doctor if my temperature increases." C) "It's okay for my husband and me to have sexual intercourse." D) "I can shower but I shouldn't take a tub bath."

Ans: C Feedback: The woman with preterm premature rupture of membranes should monitor her baby's activity by performing fetal kick counts daily, check her temperature and report any increases to the health care provider, not insert anything into her vagina or vaginal area, such as tampons or vaginal intercourse, and avoid sitting in a tub bath.

Upon entering the room of a client who has had a spontaneous abortion, the nurse observes the client crying. Which of the following responses by the nurse would be most appropriate? A) "Why are you crying?" B) "Will a pill help your pain?" C) "I'm sorry you lost your baby." D) "A baby still wasn't formed in your uterus."

Ans: C Telling the client that the nurse is sorry for the loss acknowledges the loss to the woman, validates her feelings, and brings the loss into reality.

Which assessment finding would lead the nurse to suspect infection as the cause of a client's PROM? A) Yellow-green fluid B) Blue color on Nitrazine testing C) Ferning D) Foul odor

Ans: D A foul odor of the amniotic fluid indicates infection. Yellow-green fluid would suggest meconium. A blue color on Nitrazine testing and ferning indicate the presence of amniotic fluid.

The nurse is teaching a pregnant woman about recommended weight gain. The woman has a prepregnancy body mass index of 26. The nurse determines that the teaching was successful when the woman states that she should gain no more than which amount during pregnancy? A) 35 to 40 pounds B) 25 to 35 pounds C) 28 to 40 pounds D) 15 to 25 pounds

Ans: D A woman with a body mass index of 26 is considered overweight and should gain no more than 15 to 25 pounds during pregnancy. Women with a body mass index of 18.5 to 24.9 (considered healthy weight) should gain 25 to 35 pounds. A woman with a body mass index less than 18.5 should gain 28 to 40 pounds.

Which of the following findings on a prenatal visit at 10 weeks might lead the nurse to suspect a hydatidiform mole? A) Complaint of frequent mild nausea B) Blood pressure of 120/84 mm Hg C) History of bright red spotting 6 weeks ago D) Fundal height measurement of 18 cm

Ans: D Findings with a hydatidiform mole may include uterine size larger than expected. Mild nausea would be a normal finding at 10 weeks' gestation. Blood pressure of 120/84 would not be associated with hydatidiform mole and depending on the woman's baseline blood pressure may be within acceptable parameters for her. Bright red spotting might suggest a spontaneous abortion.

Which of the following findings would the nurse interpret as suggesting a diagnosis of gestational trophoblastic disease? A) Elevated hCG levels, enlarged abdomen, quickening B) Vaginal bleeding, absence of FHR, decreased hPL levels C) Visible fetal skeleton on ultrasound, absence of quickening, enlarged abdomen D) Gestational hypertension, hyperemesis gravidarum, absence of FHR

Ans: D Gestational trophoblastic disease may be manifested by early development of preeclampsia (gestational hypertension), severe morning sickness due to high hCG levels, and absence of fetal heart rate or activity. There is no fetus, so quickening and evidence of a fetal skeleton would not be seen. The abdominal enlargement is greater than expected for pregnancy dates, but hCG, not hPL, levels are increased.

The nurse is reviewing the laboratory test results of a pregnant client. Which one of the following findings would alert the nurse to the development of HELLP syndrome? A) Hyperglycemia B) Elevated platelet count C) Leukocytosis D) Elevated liver enzymes

Ans: D HELLP is an acronym for hemolysis, elevated liver enzymes, and low platelets. Hyperglycemia or leukocytosis is not a part of this syndrome.

While talking with a woman in her third trimester, which behavior indicates to the nurse that the woman is learning to give of oneself? A) Showing concern for self and fetus as a unit B) Unconditionally accepting the pregnancy without rejection C) Longing to hold infant D) Questioning ability to become a good mother

Ans: D Learning to give of oneself would be demonstrated when the woman questions her ability to become a good mother to the infant. Showing concern for herself and fetus as a unit reflects the task of ensuring safe passage throughout pregnancy and birth. Unconditionally accepting the pregnancy reflects the task of seeking acceptance of the infant by others. Longing to hold the infant reflects the task of seeking acceptance of self in the maternal role to the infant.

The nurse teaches a primigravida client that lightening occurs about 2 weeks before the onset of labor. The mother will most likely experience which of the following at that time? A) Dysuria B) Dyspnea C) Constipation D) Urinary frequency

Ans: D Lightening refers to the descent of the fetal head into the pelvis and engagement. With this descent, pressure on the diaphragm decreases, easing breathing, but pressure on the bladder increases, leading to urinary frequency. Dysuria might indicate a urinary tract infection. Constipation may occur throughout pregnancy due to decreased peristalsis, but it is unrelated to lightening.

A nursing instructor is teaching a class to a group of students about pregnancy, insulin, and glucose. Which of the following would the instructor least likely include as opposing insulin? A) Prolactin B) Estrogen C) Progesterone D) Cortisol

Ans: D Prolactin, estrogen, and progesterone are all thought to oppose insulin. As a result, glucose is less likely to enter the mother's cells and is more likely to cross over the placenta to the fetus. After the first trimester, hPL from the placenta and steroids (cortisol) from the adrenal cortex act against insulin. hPL acts as an antagonist against maternal insulin, and thus more insulin must be secreted to counteract the increasing levels of hPL and cortisol during the last half of pregnancy.

A nurse is assessing a pregnant woman with gestational hypertension. Which of the following would lead the nurse to suspect that the client has developed severe preeclampsia? A) Urine protein 300 mg/24 hours B) Blood pressure 150/96 mm Hg C) Mild facial edema D) Hyperreflexia

Ans: D Severe preeclampsia is characterized by blood pressure over 160/110 mm Hg, urine protein levels greater than 500 mg/24 hours and hyperreflexia. Mild facial edema is associated with mild preeclampsia.

It is determined that a client's blood Rh is negative and her partner's is positive. To help prevent Rh isoimmunization, the nurse anticipates that the client will receive RhoGAM at which time? A) At 34 weeks' gestation and immediately before discharge B) 24 hours before delivery and 24 hours after delivery C) In the first trimester and within 2 hours of delivery D) At 28 weeks' gestation and again within 72 hours after delivery

Ans: D To prevent isoimmunization, the woman should receive RhoGAM at 28 to 32 weeks gestation and again within 72 hours after delivery.

A pregnant client in her second trimester has a hemoglobin level of 11 g/dL. The nurse interprets this as indicating which of the following? A) Iron-deficiency anemia B) A multiple gestation pregnancy C) Greater-than-expected weight gain D) Hemodilution of pregnancy

Ans: D Feedback: During pregnancy, the red blood cell count increases along with an increase in plasma volume. However, there is a greater increase in the plasma volume as a result of hormonal factors and sodium and water retention. Thus, the plasma increase exceeds the increase in RBCs, resulting in hemodilution of pregnancy, which is also called physiologic anemia of pregnancy. Changes in maternal iron levels would be more indicative of an iron-deficiency anemia. Although anemia may be present with a multiple gestation, an ultrasound would be a more reliable method of identifying it. Weight gain does not correlate with hemoglobin levels.

When teaching a pregnant client about the physiologic changes of pregnancy, the nurse reviews the effect of pregnancy on glucose metabolism. Which of the following would the nurse include as the underlying reason for the effect? A) Pancreatic function is affected by pregnancy. B) Glucose is utilized more rapidly during a pregnancy. C) The pregnant woman increases her dietary intake. D) Glucose moves through the placenta to assist the fetus.

Ans: D Feedback: The growing fetus has large needs for glucose, amino acids, and lipids, placing demands on maternal glucose stores. During the first half of pregnancy, much of the maternal glucose is diverted to the growing fetus. The pancreas continues to function during pregnancy. However, the placental hormones can affect maternal insulin levels. The demand for glucose by the fetus during pregnancy is high, but it is not necessarily used more rapidly. Placental hormones, not the woman's dietary intake, play a major role in glucose metabolism during pregnancy.

After teaching a pregnant woman about the hormones produced by the placenta, the nurse determines that the teaching was successful when the woman identifies which hormone produced as being the basis for pregnancy tests? A) Human placental lactogen (hPL) B) Estrogen (estriol) C) Progesterone (progestin) D) Human chorionic gonadotropin (hCG)

Ans: D Feedback: The placenta produces hCG, which is the basis for pregnancy tests. This hormone preserves the corpus luteum and its progesterone production so that the endometrial lining is maintained. Human placental lactogen modulates fetal and maternal metabolism and participates in the development of the breasts for lactation. Estrogen causes enlargement of the woman's breasts, uterus, and external genitalia and stimulates myometrial contractility. Progesterone maintains the endometrium.

A couple comes to the clinic for preconception counseling and care. As part of the visit, the nurse teaches the couple about fertilization and initial development, stating that the zygote formed by the union of the ovum and sperm consists of how many chromosomes? A) 22 B) 23 C) 44 D) 46

Ans: D Feedback: With fertilization, the ovum, containing 23 chromosomes, and the sperm, containing 23 chromosomes, join, forming a zygote with a diploid number or 46 chromosomes.

Which of the following findings on a prenatal visit at 10 weeks might lead the nurse to suspect a hydatidiform mole? A) Complaint of frequent mild nausea B) Blood pressure of 120/84 mm Hg C) History of bright red spotting 6 weeks ago D) Fundal height measurement of 18 cm

Ans: D Findings with a hydatidiform mole may include uterine size larger than expected. Mild nausea would be a normal finding at 10 weeks' gestation. Blood pressure of 120/84 would not be associated with hydatidiform mole and depending on the woman's baseline blood pressure may be within acceptable parameters for her. Bright red spotting might suggest a spontaneous abortion.

The United States ranks 50th in the world for maternal mortality and 41st among industrialized nations for infant mortality rate. When developing programs to assist in decreasing these rates, which factor would most likely need to be addressed as having the greatest impact? A) Resolving all language and cultural differences B) Assuring early and adequate prenatal care C) Providing more extensive women's shelters D) Encouraging all women to eat a balanced die

Assuring early and adequate prenatal care

The nurse is providing care to a neonate whose mother abuses heroin. Which finding would the nurse expect to assess? A. easy consolability B. sneezing C. vigorous sucking D. hypotonicity

B

What criteria would the physician base his decision on to begin insulin therapy for a gestational diabetic mother? A. Urine is 2+ for glucose and serum blood glucose is 120. B. A 2-hour postprandial glucose level cannot be kept below 120 mg/dL. C. Weight gain is over 30 pounds (13.6 kg) and blood sugars are fluctuating between 95 and 130 throughout the day. D. Client cannot keep fasting blood sugar lower than 90 mg/dL.

B R:A physician usually recommends beginning a woman with gestational diabetes on insulin therapy when exercise and diet are ineffective and if she is unable to keep her fasting blood sugar levels below 95 mg/dL or her 2-hour postprandial glucose levels below 120 mg/dL.

A woman with diabetes is in labor. To reduce the likelihood of neonatal hypoglycemia, the nurse monitors the client's blood glucose level closely with the goal to maintain which level? A. below 105 mg/dL B. below 110 mg/dL C. below 120 mg/dL D. below 115 mg/dL

B R: For the laboring woman with diabetes, the blood glucose levels are monitored every 1 to 2 hours with the goal to maintain the levels below 110 mg/dL throughout the labor to reduce the likelihood of neonatal hypoglycemia. If necessary, an infusion of regular insulin may be given to maintain this level.

What criteria would the physician base his decision on to begin insulin therapy for a gestational diabetic mother? A. Urine is 2+ for glucose and serum blood glucose is 120. B. A 2-hour postprandial glucose level cannot be kept below 120 mg/dL. C. Weight gain is over 30 pounds (13.6 kg) and blood sugars are fluctuating between 95 and 130 throughout the day. D. Client cannot keep fasting blood sugar lower than 90 mg/dL.

B A physician usually recommends beginning a woman with gestational diabetes on insulin therapy when exercise and diet are ineffective and if she is unable to keep her fasting blood sugar levels below 95 mg/dL or her 2-hour postprandial glucose levels below 120 mg/dL.

9. A client is admitted to the labor and birthing suite in early labor. On review of her medical record, the nurse determines that the client's pelvic shape as identified in the antepartal progress notes is the most favorable one for a vaginal delivery. Which pelvic shape would the nurse have noted? A) Platypelloid B) Gynecoid C) Android D) Anthropoid

B Girl pelvis

The nursing student demonstrates an understanding of dystocia with which statement? A) "Dystocia is not diagnosed until after birth." B) "Dystocia is diagnosed after labor has progressed for a time." C) "Dystocia is diagnosed at the start of labor." D) "Dystocia cannot be diagnosed until just before birth."

B) "Dystocia is diagnosed after labor has progressed for a time." Nursing management of the woman with dystocia, regardless of etiology, requires patience. The nurse needs to provide physical and emotional support to the client and family. Dystocia is diagnosed not at the start of labor, but rather after it has progressed for a time.

A pregnant client tells her nurse that she is interested in arranging a home birth. After educating the client on the advantages and disadvantages, which statement would indicate that the client understood the information? A) "I like having the privacy,but it might be too expensive for me to set up in my home." B) "I want to have more control, but I am concerned if an emergency would arise." C) "It is safer because I will have a midwife." D) "The midwife is trained to resolve any emergency, and she canbring any pain meds."

B) "I want to have more control, but I am concerned if an emergency would arise."

Which of the following would the nurse include when teaching a pregnant woman about chorionic villus sampling? A) "The results should be available in about a week." B) "You'll have an ultrasound first and then the test." C) "Afterwards, you can resume your exercise program." D) "This test is very helpful for identifying spinal defects."

B) "You'll have an ultrasound first and then the test."

A nurse is describing the structure and function of the reproductive system to an adolescent health class. The nurse describes the secretion of the seminal vesicles as which of the following? A) Mucus-like B) Alkaline C) Acidic D) Semen

B) Alkaline

A nursing instructor is describing the hormones involved in the menstrual cycle to a group of nursing students. The instructor determines the teaching was successful when the students identify follicle-stimulating hormone as being secreted by which of the following? A) Hypothalamus B) Anterior pituitary gland C) Ovaries D) Corpus luteum

B) Anterior pituitary gland

When preparing a schedule of follow-up visits for a pregnant woman with chronic hypertension, which of the following would be most appropriate? A) Monthly visits until 32 weeks, then bi-monthly visits B) Bi-monthly visits until 28 weeks, then weekly visits C) Monthly visits until 20 weeks, then bi-monthly visits D) Bi-monthly visits until 36 weeks, then weekly visits

B) Bi-monthly visits until 28 weeks, then weekly visits

A nurse practicing in the community is preparing a presentation for a group of nursing students about this practice setting. Which of the following would the nurse include as characteristic of this role? A) Greater emphasis ondirect physical care B) Broader assessment to include the environment C) Increased dependency on physician D) Limited decision making and support

B) Broader assessment to include the environment

A group of students are reviewing an article describing information related to indicators for women's health and the results of a national study. Which of the following would the students identify as being satisfactory for women? Select all that apply. A) Smoking cessation B) Colorectal cancer screening C) Violence against women D) Health insurance coverage E) Mammograms

B) Colorectal cancer screening E) Mammograms

A group of nursing students are preparing a presentation for their class about measures to prevent toxoplasmosis. Which of the following would the students be least likely to include? Select all that apply. A) Washing raw fruits and vegetables before eating them B) Cooking all meat to an internal temperature of 140 F C) Wearing gardening gloves when working in the soil D) Avoiding contact with a cats litter box.

B) Cooking all meat to an internal temperature of 140 F

The nurse is assessing a 13-year-old girl who has had her first menses. Which of the following events would the nurse expect to have occurred first? A) Evidence of pubic hair B) Development of breast buds C) Onset of menses D) Growth spurt

B) Development of breast buds

The nurse would recommend the use of which supplement as a primary prevention strategy to prevent neural tube defects with pregnant women? A) Calcium B) Folic acid C) Vitamin C D) Iron

B) Folic acid

A group of students are reviewing information about sexually transmitted infections and their effect on pregnancy. The students demonstrate understanding of the information when they identify which infection as being responsible for ophthalmia neonatorum? A) Syphilis B) Gonorrhea C) Chlamydia D) HPV

B) Gonorrhea

A neonate born to a mother who was abusing heroin is exhibiting signs and symptoms of withdrawal. Which of the following would the nurse assess? (Select all that apply.) A) Low whimpering cry B) Hypertonicity C) Lethargy D) Excessive sneezing E) Overly vigorous sucking F) Tremors

B) Hypertonicity C) Lethargy D) Excessive sneezing

During a routine prenatal visit, a client, 36 weeks pregnant, states she has difficulty breathing and feels like her pulse rate is really fast. The nurse finds her pulse to be 100 beats per minute (increased from baseline readings of 70 to 74 beats per minute) and irregular, with bilateral crackles in the lower lung bases. Which nursing diagnosis would be the priority for this client? A) Ineffective tissue perfusion related to supine hypotensive syndrome B) Impaired gas exchange related to pulmonary congestion C) Activity intolerance related to increased metabolic requirements D) Anxiety related to fear of pregnancy outcome

B) Impaired gas exchange related to pulmonary congestion

A client's last menstrual period was April 11. Using Nagele's rule, her expected date of birth (EDB) would be: A) January 4 B) January 18 C) January 25 D) February 24

B) January 18

A woman comes to the clinic complaining of a vaginal discharge. The nurse suspects that theclient has an infection. When gathering additional information, which of the following would thenurse be least likely to identify as placing the client at risk for an infection? A) Recent antibiotic therapy for an upper respiratory infection B) Last menstrual period about 5 days ago. C) Weekly douching D) Frequent use of feminine hygiene sprays

B) Last menstrual period about 5 days ago

When describing the hormones involved in the menstrual cycle, a nurse identifies which hormone as responsible for initiating the cycle? A) Estrogen B) Luteinizing hormone C) Progesterone D) Prolactin

B) Luteinizing hormone

When discussing fetal mortality with a group of students, a nurse addresses maternal factors. Which of the following would the nurse most likely include? Select all that apply. A) Chromosomal abnormalities B) Malnutrition C) Preterm cervical dilation D) Underlying disease condition E) Poor placental attachment

B) Malnutrition C) Preterm cervical dilation D) Underlying disease condition

When describing the male sexual response to a group of students, the instructor determines that the teaching was successful when they identify emission as which of the following? A) Semen forced through the urethra to the outside B) Movement of sperm from the testes and fluid into the urethras C) Dilation of the penile arteries with increased blood flow to the tissues. D) Body's return to the physiologic nonstimulated state

B) Movement of sperm from the testes and fluid into the urethras

A client's maternal serum alpha-fetoprotein (MSAFP) level was unusually elevated at 17 weeks. The nurse suspects which of the following? A) Fetal hypoxia B) Open spinal defects C) Down syndrome D) Maternal hypertension

B) Open spinal defects

After teaching a group of students about the different perinatal education methods, the instructor determines that the teaching was successful when the students identify which of the following as the Bradley method? A) Psychoprophylactic method B) Partner-coached method C) Natural childbirth method D) Mind prevention method

B) Partner-coached method

The nurse is preparing an outline for a class on the physiology of the male sexual response. Which event would the nurse identify as occurring first? A) Sperm emission B) Penile vasodilation C) Psychological release D) Ejaculation

B) Penile vasodilation -With sexual stimulation, the arteries leading to the penis dilate and increase blood flow into erectile tissue. Blood accumulates, causing the penis to swell and elongate. -Sperm emission (movement of sperm from the testes and fluid from the accessory glands) occurs with orgasm.

A woman comes to the clinic for an evaluation. During the visit, the woman tells the nurse that her menstrual cycles have become irregular. "I've also been waking up at night feeling really hot and sweating. The nurse interprets these findings as which of the following? A) Menopause B) Perimenopause C) Climacteric D) Menarche

B) Perimenopause

A nurse is reviewing the medical record of a pregnant woman and notes that she is gravid II. The nurse interprets this to indicate the number of: A) Deliveries B) Pregnancies C) Spontaneous abortions D) Pre-term births

B) Pregnancies

When describing the menstrual cycle to a group of young women, the nurse explains that estrogen levels are highest during which phase of the endometrial cycle? A) Menstrual B) Proliferative C) Secretory D) Ischemic

B) Proliferative

When describing perinatal education to a pregnant woman and her partner, the nurse emphasizes that the primary goal of these classes is to: A) Equip a couple with the knowledge to experience a pain-free childbirth B) Provide knowledge and skills to actively participate in birth and parenting C) Eliminate anxiety so that they can have an uncomplicated birth D) Empower the couple to totally control the birth process

B) Provide knowledge and skills to actively participate in birth and parenting

The nurse is assessing a newborn of a woman who is suspected of abusing alcohol. Which newborn finding would provide additional evidence to support this suspicion? A) Wide large eyes B) Thin upper lip C) Protruding jaw D) Elongated nose

B) Thin upper lip

A group of nursing students are reviewing information about the male reproductive structures. The students demonstrate understanding of the information when they identify which of the following as accessory organs? (Select all that apply.) A) Testes B) Vas deferens C) Bulbourethral glands D) Prostate gland E) Penis

B) Vas deferens C) Bulbourethral glands D) Prostate gland

Which of the following statements is accurate regarding women's health care in today's system? A) Women spend 95 cents of every dollar spent on health care. B) Women make almost 90% of all health caredecisions. C) Women are still the minority in the United States. D) Men use more health services than women

B) Women make almost 90% of all health caredecisions.

A nurse is developing a program for pregnant women with diabetes about reducing complications. Which factor would the nurse identify as being most important in helping to reduce the maternal/fetal/neonatal complications associated with pregnancy and diabetes? A)Stability of the woman's emotional and psychological status B)Degree of glycemic control achieved during the pregnancy C)Evaluation of retinopathy by an ophthalmologist D)Blood urea nitrogen level (BUN. within normal limits

B)Degree of glycemic control achieved during the pregnancy

The nurse is teaching a pregnant woman with iron deficiency anemia about foods high in iron. Which foods if selected by the woman indicate a successful teaching program? Select all that apply. A. potatoes B. broccoli C. peanut butter D. corn E. yogurt F. raisins

B,C,F R:Foods high in iron include dried fruits such as raisins, whole grains, green leafy vegetables such as broccoli and spinach, peanut butter, and iron-fortified cereals. Potatoes and corn are high in carbohydrates. Yogurt is a good source of calcium.

3. When assessing cervical effacement of a client in labor, the nurse assesses which of the following characteristics? A) Extent of opening to its widest diameter B) Degree of thinning C) Passage of the mucous plug D) Fetal presenting part

B. Effacement refers to the degree of thinning of the cervix. Cervical dilation refers to the extent of opening at the widest diameter. occurs with bloody show is a .

4. A woman calls the health care facility stating that she is in labor. The nurse would urge the client to come to the facility if the client reports which of the following? A) Increased energy level with alternating strong and weak contractions B) Moderately strong contractions every 4 minutes, lasting about 1 minute C) Contractions noted in the front of abdomen that stop when she walks D) Pink-tinged vaginal secretions and irregular contractions lasting about 30 seconds

B. - Moderately strong regular contractions 60 seconds in duration indicate that the client is probably in the active phase of the first stage of labor. - Alternating strong and weak contractions, contractions in the front of the abdomen that change with activity - pink-tinged secretions with irregular contractions suggest false labor.

After teaching a group of students about fetal development, the instructor determines that the teaching was successful when the students identify which of the following as essential for fetal lung development? A) Umbilical cord B) Amniotic fluid C) Placenta D) Trophoblasts

B. Amniotic fluid is essential for fetal growth and development, especially fetal lung development. The umbilical cord is the lifeline from the mother to the growing embryo. The placenta serves as the interface between the mother and developing fetus. It secretes hormones and supplies the fetus with nutrients and oxygen needed for growth. The trophoblasts differentiate into all the cells that form that placenta.

A woman with preterm labor is receiving magnesium sulfate. Which finding would require the nurse to intervene immediately? A) Respiratory rate of 16 breaths per minute B) Diminished deep tendon reflexes C) Urine output of 45 mL/hour D) Alert level of consciousness

B. Diminished deep tendon reflexes suggest magnesium toxicity, which requires immediate intervention. Additional signs of magnesium toxicity include a respiratory rate less than 12 breaths/minute, urine output less than 30 mL/hour, and a decreased level of consciousness.

After teaching a couple about what to expect with their planned cesarean birth, which statement indicates the need for additional teaching? A) "Holding a pillow against my incision will help me when I cough." B) "I'm going to have to wait a few days before I can start breast-feeding." C) "I guess the nurses will be getting me up and out of bed rather quickly." D) "I'll probably have a tube in my bladder for about 24 hours or so."

B. Typically, breast-feeding is initiated early as soon as possible after birth to promote bonding. The woman may need to use alternate positioning techniques to reduce incisional discomfort.

During a routine prenatal visit, a client, 36 weeks pregnant, states she has difficulty breathing and feels like her pulse rate is really fast. The nurse finds her pulse to be 100 beats per minute (increased from baseline readings of 70 to 74 beats per minute) and irregular, with bilateral crackles in the lower lung bases. Which nursing diagnosis would be the priority for this client? A) Ineffective tissue perfusion related to supine hypotensive syndrome B) Impaired gas exchange related to pulmonary congestion C) Activity intolerance related to increased metabolic requirements D) Anxiety related to fear of pregnancy outcome

B. Typically, heart rate increases by approximately 10 to 15 beats per minute during pregnancy and the lungs should be clear. Dyspnea may occur during the third trimester as the enlarging uterus presses on the diaphragm. However, the findings described indicate that the woman is experiencing impaired gas exchange. There is no evidence to support problems with tissue perfusion, activity, or anxiety.

When assessing several women for possible VBAC, which woman would the nurse identify as being the best candidate? A) One who has undergone a previous myomectomy B) One who had a previous cesarean birth via a low transverse incision C) One who has a history of a contracted pelvis D) One who has avertical incision from a previous cesarean birth

B. VBAC is an appropriate choice for women who have had a previous cesarean birth with a lower abdominal transverse incision. It is contraindicated in women who have a prior classic uterine incision (vertical), prior transfundal surgery, such as myomectomy, or a contracted pelvis.

After describing continuous internal electronic fetal monitoring to a laboring woman and her partner, which of the following would indicate the need for additional teaching? A) "This type of monitoring is the most accurate method for our baby." B) "Unfortunately, I'm going to have to stay quite still in bed while it is in place." C) "This type of monitoring can only be used after my membranes rupture." D) "You'll be inserting a special electrode into my baby's scalp."

B. With continuous internal electronic monitoring, maternal position changes and movement do not interfere with the quality of the tracing. *Continuous internal monitoring is considered the most accurate method, but it can be used only if certain criteria are met, such as rupture of membranes.

21. When describing the stages of labor to a pregnant woman, which of the following would the nurse identify as the major change occurring during the first stage? A) Regular contractions B) Cervical dilation C) Fetal movement through the birth canal D) Placental separation

B. The primary change occurring during the first stage of labor is progressive cervical dilation.

10. A woman telephones her health care provider and reports that her "water just broke." Which suggestion by the nurse would be most appropriate? A) "Call us back when you start having contractions." B) "Come to the clinic or emergency department for an evaluation." C) "Drink 3 to 4 glasses of water and lie down." D) "Come in as soon as you feel the urge to push."

B. When the amniotic sac ruptures, the barrier to infection is gone and there is the danger of cord prolapse if engagement has not occurred. Therefore, the nurse should suggest that the woman come in for an evaluation.

The nurse would anticipate a cesarean birth for a client who has which active infection present at the onset of labor? A. Hepatitis B. Herpes simplex virus C. Toxoplasmosis D. Human papillomavirus

B. -Herpes exposure during the birth process poses a high risk for mortality to the neonate. If the woman has active herpetic lesions in the genital tract, a surgical birth is planned to avoid this exposure. - Hepatitis is a chronic liver disorder, and the fetus if exposed would at most become a carrier; a surgical birth would not be expected for this woman. -Toxoplasmosis is passed through the placenta to the fetus prior to birth, so a cesarean birth would not prevent exposure. -HPV would be manifested clinically by genital warts on the woman, and a surgical birth would not be anticipated to prevent exposure unless the warts caused an obstruction.

A woman in labor is to receive continuous internal electronic fetal monitoring. The nurse reviews the woman's medical record to ensure which of the following as being required? A) Intact membranes B) Cervical dilation of 2 cm or more C) Floating presenting fetal part D) A neonatologist to insert the electrode

B. For continuous internal electronic fetal monitoring, four criteria must be met: 1. ruptured membranes 2. cervical dilation of at least 2 cm 3. fetal presenting part low enough to allow placement of the electrode 4. skilled practitioner available to insert the electrode.

14. A woman in her third trimester comes to the clinic for a prenatal visit. During assessment the woman reports that her breathing has become much easier in the last week but she has noticed increased pelvic pressure, cramping, and lower back pain. The nurse determines that which of the following has most likely occurred? A) Cervical dilation B) Lightening C) Bloody show D) Braxton-Hicks contractions

B. Lightening occurs when the fetal presenting part begins to descend into the maternal pelvis. The uterus lowers and moves into the maternal pelvis. The shape of the abdomen changes as a result of the change in the uterus. The woman usually notes that her breathing is much easier. However, she may complain of increased pelvic pressure, cramping, and lower back pain. * Although cervical dilation also may be occurring, it does not account for the woman's complaints. * Bloody show refers to passage of the mucous plug that fills the cervical canal during pregnancy. It occurs with the onset of labor.

The nurse is caring for a woman experiencing hypertonic uterine dystocia. The woman's contractions are erratic in their frequency, duration, and of high intensity. The priority nursing intervention would be to: A. Encourage ambulation every 30 minutes B. Provide pain relief measures C. Monitor the Pitocin infusion rate closely D. Prepare the woman for an amniotomy

B. as women with hypertonic uterine contractions experience a high level of pain related to the high intensity of contractions. - Providing comfort measures along with pharmacologic agents to reduce would be a priority. - Response "A" is incorrect since a woman experiencing a high level of pain secondary to contraction intensity would not feel like ambulating during this challenging time period. - Response "C" is incorrect because with this type of dystocia, augmentation of labor contractions would not be needed. If Pitocin had been infusing prior to the identification of this dystocia pattern, it would be discontinued to reduce the intensity of the contractions.

A nurse is assessing a woman after birth and notes a second-degree laceration. The nurse interprets this as indicating that the tear extends through which of the following? A) Skin B) Muscles of perineal body C) Anal sphincter D) Anterior rectal wall

B. Muscle of perineal body

After a vaginal examination, the nurse determines that the client's fetus is in an occiput posterior position. The nurse would anticipate that the client will have:

BACK PAIN

_________________ contractions increase in strength and frequency and aid in moving the cervix from a posterior position to an anterior position. They also help in ripening and softening the cervix.

Braxton-Hicks

1. A woman in her 40th week of pregnancy calls the nurse at the clinic and says she's not sure whether she is in true or false labor. Which statement by the client would lead the nurse to suspect that the woman is experiencing false labor? A) "I'm feeling contractions mostly in my back." B) "My contractions are about 6 minutes apart and regular." C) "The contractions slow down when I walk around." D) "If I try to talk to my partner during a contraction, I can't."

C

Many women develop iron-deficient anemia during pregnancy. What diagnostic criteria would the nurse monitor for to determine anemia in the pregnant woman? A. Hemoglobin of 13 or lower B. Heart rate of 84 C. Hematocrit of 32% or less D. Blood pressure of 100/68

C R:Iron-deficiency anemia is diagnosed in a pregnant woman if the hematocrit is less that 33% or the hemoglobin is less than 11 g/dL. Tachycardia, hypotension and tachypnea are all symptoms of iron-deficiency anemia but are not diagnostic criteria.

Which of the following data on a client's health history would the nurse identify as contributing to the client's risk for an ectopic pregnancy? A) Use of oral contraceptives for 5 years B) Ovarian cyst 2 years ago C) Recurrent pelvic infections D) Heavy, irregular menses

C In the general population, most cases of ectopic pregnancy are the result of tubal scarring secondary to pelvic inflammatory disease. Oral contraceptives, ovarian cysts, and heavy, irregular menses are not considered risk factors for ectopic pregnancy.

A group of students are reviewing the historical aspects about childbirth. The students demonstrate understanding of the information when they identify the use of twilight sleep as a key event during which time frame? A) 1700s B) 1800s C) 1900s D) 2000s

C) 1900s

A nurse is developing a plan of care for a woman to ensure continuity of care during pregnancy, labor, and childbirth. Which of the following would be most important for the nurse to incorporate into that plan? A) Adhering to strict, specific routines B) Involving a pediatric physician C) Educating the client about the importance of a support person D) Assigning several nurses as a support team

C) Educating the client about the importance of a support person

A nurse is providing care to a woman who has just delivered a healthy newborn. Which action would least likely demonstrate application of the concept of family-centered care? A) Focusing on the birth as a normal healthy event for the family B) Creating opportunities for the family to make informed decisions C) Encouraging the woman to keep her other children at home D) Fostering a sense of respect for the mother and the family

C) Encouraging the woman to keep her other children at home

A nurse is preparing a presentation for a local community group about health status and children's health. Which of the following would the nurse include as one of the most significant measures? A) Fetal mortality rate B) Neonatal mortality rate C) Infant mortality rate D) Maternal mortality rate

C) Infant mortality rate

A nurse is preparing a presentation for a local women's group about heart disease and women. Which of the following would the nurse expect to address when discussing measures to promote health. A) Women have similar symptoms as men for a heart attack. B) Heart disease is no longer viewed as a "man's disease." C) Women experiencing a heart attack are at greater risk for dying. D) Heart attacks in women are more easily diagnosed.

C) Women experiencing a heart attack are at greater risk for dying.

A nurse is working with a pregnant woman to schedule follow-up visits for her pregnancy. Which statement by the woman indicates that she understands the scheduling? A) "I need to make visits every 2 months until I'm 36 weeks pregnant." B) "Once I get to 28 weeks, I have to come twice a month." C) "From now until I'm 28 weeks, I'll be coming once a month." D) "I'll make sure to get a day off every 2 weeks to make my visits."

C) "From now until I'm 28 weeks, I'll be coming once a month."

A pregnant woman is flying across the country to visit her family. After teaching the woman about traveling during pregnancy, which statement indicates that the teaching was successful? A) "I'll sit in a window seat so I can focus on the sky to help relax me." B) "I won't drink too much fluid so I don't have to urinate so often." C) "I'll get up and walk around the airplane about every 2 hours." D) "I'll do some upper arm stretches while sitting in my seat."

C) "I'll get up and walk around the airplane about every 2 hours."

During class, a nursing student asks, "I read an article that was talking about integrative medicine. What is that?" Which response by the instructor would be most appropriate? A) "It refers to the use of complementary and alternative medicine in place of traditional therapies for a condition." B) "It means that complementary and alternative medicine is used together with conventional therapies to reduce pain or discomfort." C) "It means that mainstream medical therapies and complementary and alternative therapies are combined based on scientific evidence for being effective." D) "It refers to situations when a client and his or her family prefer to use an unproven method of treatment over a proven one."

C) "It means that mainstream medical therapies and complementary and alternative therapies are combined based on scientific evidence for being effective."

A nurse is educating a client about a care plan. Which of the following statements would be appropriate to assess the client's learning ability? A) "Did you graduate from high school; how many years of schooling did you have?" B) "Do you have someone in your family who would understand this information?" C) "Many people have trouble remembering information; is this a problem for you?" D) "Wouldyou prefer that the doctor give you more detailed medical information?"

C) "Many people have trouble remembering information; is this a problem for you?"

A pregnant woman in the 36th week of gestation complains that her feet are quite swollen at the end of the day. After careful assessment, the nurse determines that this is an expected finding at this stage of pregnancy. Which intervention would be most appropriate for the nurse to suggest? A) "Limit your intake of fluids." B) "Eliminate salt from your diet." C) "Try elevating your legs when you sit." D) "Wear Spandex-type full-length pants."

C) "Try elevating your legs when you sit."

A nurse measures a pregnant woman's fundal height and finds it to be 28 cm. The nurse interprets this to indicate which of the following? A) 14 weeks' gestation B) 20 weeks' gestation C) 28 weeks' gestation D) 36 weeks' gestation

C) 28 weeks' gestation

A client who is 4 months pregnant is at the prenatal clinic for her initial visit. Her history reveals she has 7-year-old twins who were born at 34 weeks gestation, a 2-year old son born at 39 weeks gestation, and a spontaneous abortion 1 year ago at 6 weeks gestation. Using the GTPAL method, the nurse would document her obstetric history as: A) 3 2 1 0 3 B) 3 1 2 2 3 C) 4 1 1 1 3 D) 4 2 1 3 1

C) 4 1 1 1 3

When caring for childbearing families from cultures different from one's own, which of the following must be accomplished first? A) Adapt to the practices of the family's culture B) Determine similarities between both cultures C) Assess personal feelings about that culture D) Learn as much as possible about that culture

C) Assess personal feelings about that culture

On the first prenatal visit, examination of the woman's internal genitalia reveals a bluish coloration of the cervix and vaginal mucosa. The nurse records this finding as: A) Hegar's sign B) Goodell's sign C) Chadwick's sign D) Homans' sign

C) Chadwick's sign

When integrating the principles of family-centered care, the nurse would include which of the following? A) Childbirth is viewed as a procedural event B) Families are unable to make informed choices C) Childbirth results in changes in relationships D) Families require little information to make appropriate decisions

C) Childbirth results in changes in relationships

A group of nurses are reviewing the steps for developing cultural competence. The students demonstrate understanding when they identify which of the following as the final step? A) Cultural knowledge B) Cultural skills C) Cultural encounter D) Cultural awareness

C) Cultural encounter

A nurse is preparing a presentation for a group of young adult pregnant women about common infections and their effect on pregnancy. When describing the infections, which infection would the nurse include as the most common congenital and perinatal viral infection in the world? A) Rubella B) Hepatitis B C) Cytomegalovirus D) Parvovirus B19

C) Cytomegalovirus

When comparing community-based nursing with nursing in the acute care setting to a group of nursing students, the nurse describes the challenges associated with community-based nursing. Which of the following would the nurse include? A) Increased time available for education B) Improved access to resources C) Decision making in isolation D) Greater environmental structure

C) Decision making in isolation

When assuming the role of discharge planner for a woman requiring ventilator support at home, the nurse would do which of the following? A) Confer with the client's mother B) Teach new self-care skills to the client C) Determine if there is a need for back-up power D) Discuss coverage with the insurance company

C) Determine if there is a need for back-up power

Which action would the nurse include in a primary prevention program in the community to help reduce the incidence of HIV infection? A) Provide treatment for clients who test positive for HIV B) Monitor viral load counts periodically C) Educate clients in how to practice safe sex D) Offer testing for clients who practice unsafe sex

C) Educate clients in how to practice safe sex

When preparing a woman for an amniocentesis, the nurse would instruct her to do which of the following? A) Shower with an antiseptic scrub. B) Swallow the preprocedure sedative. C) Empty her bladder. D) Lie on her left side.

C) Empty her bladder.

After teaching a group of students about female reproductive anatomy, the instructor determines that the teaching was successful when the students identify which of the following as the site of fertilization? A) Vagina B) Uterus C) Fallopian tubes D) Vestibule

C) Fallopian tubes

After teaching a pregnant woman with iron deficiency anemia about her prescribed iron supplement, which statement indicates successful teaching? A) I should take my iron with milk. B) I should avoid drinking orange juice. C) I need to eat foods high in fiber. D) I'll call the doctor if my stool is black and tarry.

C) I need to eat foods high in fiber.

A pregnant woman has a rubella titer drawn on her first prenatal visit. The nurse explains that this test measures which of the following? A) Platelet level B) Rh status C) Immunity to German measles D) Red blood cell count

C) Immunity to German measles

After teaching a group of adolescent girls about female reproductive development, the nurse determines that teaching was successful when the girls state that menarche is defined as a woman's first: A) Sexual experience B) Full hormonal cycle C) Menstrual period D) Sign of breast development

C) Menstrual period

A nursing student is reviewing information about documenting client care and education in the medical record and the purposes that it serves. The student demonstrates a need for additional study when the nurse identifies which of the following as a reason? A) Serves as a communication tool for the interdisciplinary team. B) Demonstrates education the family has received if legal matters arise. C) Permits others access to allow refusal of medical insurance coverage. D) Verifies meeting client education standards set by the Joint Commission.

C) Permits others access to allow refusal of medical insurance coverage.

A nurse is developing cultural competence. Which of the following indicates that the nurse is in the process of developing cultural knowledge? Select all that apply. A) Examiningpersonal sociocultural heritage B) Reviewing personal biases and prejudices C) Seeking resources to further understanding of other cultures D) Becoming familiar with other culturally diverse lifestyles E) Performing a competent cultural assessment F) Advocating for social justice to eliminate disparities.

C) Seeking resources to further understanding of other cultures D) Becoming familiar with other culturally diverse lifestyles

The nurse would prepare a client for amnioinfusion when which action occurs? A) Maternal pushing is compromised due to anesthesia. B) The fetus shows abnormal fetal heart rate patterns. C) Severe variable decelerations occur and are due to cord compression. D) Fetal presenting part fails to rotate fully and descend into the pelvis.

C) Severe variable decelerations occur and are due to cord compression. Indications for amnioinfusion include severe variable decelerations resulting from cord compression, oligohydramnios (decreased amniotic fluid), postmaturity, preterm labor with rupture of the membranes, and thick meconium fluid. Failure of the fetal presenting part to rotate fully, descend in the pelvis, abnormal fetal heart rate patterns or acute pulmonary edema, and compromised maternal pushing sensations from anesthesia are indications for forceps-assisted birth, and not for amniofusion.

Which female reproductive tract structure would the nurse describe to a group of young women as containing rugae that enable it to dilate during labor and birth? A) Cervix B) Fallopian tube C) Vagina D) Vulva

C) Vagina

After teaching a group of nursing students about the impact of pregnancy on the older woman, the instructor determines that the teaching was successful when the students state which of the following? A) The majority of women who become pregnant over age 35 experience complications. B) Women over the age of 35 who become pregnant require a specialized type of assessment. C) Women over age 35 and are pregnant have an increased risk for spontaneous abortions. D) Women over age 35 are more likely to have substance abuse problems.

C) Women over age 35 and are pregnant have an increased risk for spontaneous abortions.

A client is experiencing shoulder dystocia during birth. The nurse would place priority on performing which assessment postbirth? A) monitor for a cardiac anomaly B) assess for cleft palate C) brachial plexus assessment D) extensive lacerations

C) brachial plexus assessment The nurse should identify nerve damage as a risk to the fetus in cases of shoulder dystocia. Other fetal risks include asphyxia, clavicle fracture, central nervous system injury or dysfunction, and death. Extensive lacerations is a poor maternal outcome due to the occurrence of shoulder dystocia.

The nurse is teaching a pregnant woman with type 1 diabetes about her diet during pregnancy. Which client statement indicates that the nurses teaching was successful? A)I'll basically follow the same diet that I was following before I became pregnant. B)Because I need extra protein, I'll have to increase my intake of milk and meat. C)Pregnancy affects insulin production, so I'll need to make adjustments in my diet. D)I'll adjust my diet and insulin based on the results of my urine tests for glucose.

C)Pregnancy affects insulin production, so I'll need to make adjustments in my diet.

The nurse is doing meal planning with a pregnant woman with iron-deficiency anemia. What dietary recommendations would the nurse make to enhance the woman's intake of iron? Select all that apply. A. Since fortified cereals are a poor source of iron, eat eggs or pancakes for breakfast. B. Limit intake of dried fruits, eating only fresh fruit. C. Drink orange juice with the iron supplement. D. Cook food in an iron skillet, if possible. E. Increase intake of dried beans and green leafy vegetables.

C,D,E R:Dried fruits, fortified grains and cereals, and animal protein are all good sources of iron for a pregnant woman. Cooking in an iron skillet also will increase the amount of iron ingested. Vitamin C, like what is found in orange juice, enhances absorption of iron and is recommended to drink when taking iron supplements. Folate also increases the effectiveness of iron supplements; foods high in folate include green leafy vegetables, fortified grains and dried beans.

During a routine prenatal check up, the nurse interviews a pregnant client to identify possible risk factors for developing gestational diabetes. Which factor would the nurse identify as increasing the woman's risk? Select all that apply. A. younger maternal age at pregnancy B. previous birth of small for gestational age baby C. maternal obesity with body mass index more than 35 D. client of African-American lineage E. previous history of spontaneous abortion

C,E R:The risk factors for gestational diabetes include previous history of spontaneous abortion, maternal obesity with body mass index (BMI) more than 35, and client of a high-risk ethnic group such as Native American, Hispanic, Asian. The other risk factors for gestational diabetes are previous history of stillbirth, birth of large for gestational age infant, and advancing maternal age.

26. Assessment of a pregnant woman reveals that the presenting part of the fetus is at the level of the maternal ischial spines. The nurse documents this as which station? A) -2 B) -1 C) 0 D) +1

C.

When planning the care of a woman in the active phase of labor, the nurse would anticipate assessing the fetal heart rate at which interval? A) Every 2 to 4 hours B) Every 45 to 60 minutes C) Every 15 to 30 minutes D) Every 10 to 15 minutes

C. During the active phase of labor, FHR is monitored every 15 to 30 minutes.

After teaching a pregnant woman with iron deficiency anemia about her prescribed iron supplement, which statement indicates successful teaching? A) I should take my iron with milk. B) I should avoid drinking orange juice. C) I need to eat foods high in fiber. D) I'll call the doctor if my stool is black and tarry.

C. Iron supplements can lead to constipation, so the woman needs to increase her intake of fluids and high-fiber foods. Milk inhibits absorption and should be discouraged. Vitamin C-containing fluids such as orange juice are encouraged because they promote absorption.

A woman in labor who received an opioid for pain relief develops respiratory depression. The nurse would expect which agent to be administered? A) Butorphanol B) Fentanyl C) Naloxone D) Promethazine

C. Naloxone is an opioid antagonist used to reverse the effects of opioids such as respiratory depression.

5. A woman is in the first stage of labor. The nurse would encourage her to assume which position to facilitate the progress of labor? A) Supine B) Lithotomy C) Upright D) Knee-chest

C. The use of any upright position helps to reduce the length of labor. - The knee-chest position would assist in rotating the fetus in a posterior position.

17. The nurse is reviewing the medical record of a woman in labor and notes that the fetal position is documented as LSA. The nurse interprets this information as indicating which of the following is the presenting part? A) Occiput B) Face C) Buttocks D) Shoulder

C. - The second letter denotes the presenting part which in this case is "S" or the sacrum or buttocks. - The letter "O" would denote the occiput or vertex presentation. - The letter "M" would denote the mentum (chin) or face presentation. - The letter "A" would denote the acromion or shoulder presentation.

When applying the ultrasound transducers for continuous external electronic fetal monitoring, at which location would the nurse place the transducer to record the FHR? A) Over the uterine fundus where contractions are most intense B) Above the umbilicus toward the right side of the diaphragm C) Between the umbilicus and the symphysis pubis D) Between the xiphoid process and umbilicus

C. - The ultrasound transducer is positioned on the maternal abdomen in the midline between the umbilicus and the symphysis pubis. - The tocotransducer is placed over the uterine fundus in the area of greatest contractility.

When palpating the fundus during a contraction, the nurse notes that it feels like a chin. The nurse interprets this finding as indicating which type of contraction? A) Intense B) Strong C) Moderate D) Mild

C. -A contraction that feels like the chin typically represents a moderate contraction. -A contraction described as feeling like the tip of the nose indicates a mild contraction. - A strong contraction feels like the forehead.

A nurse is assisting with the delivery of a newborn. The fetal head has just emerged. Which of the following would be done next? A) Suctioning of the mouth and nose B) Clamping of the umbilical cord C) Checking for the cord around the neck D) Drying of the newborn

C. Checking for the cord around the neck once the fetal head has emerged, the primary care provider explores the fetal neck to see if the umbilical cord is wrapped around it. If it is, the cord is slipped over the head to facilitate delivery. * Then the health care provider suctions the newborn's mouth first (because the newborn is an obligate nose breather) and then the nares with a bulb syringe to prevent aspiration of mucus, amniotic fluid, or meconium.

The nurse notes the presence of transient fetal accelerations on the fetal monitoring strip. Which intervention would be most appropriate? A. Reposition the client on the left side. B. Begin 100% oxygen via face mask. C. Document this as indicating a normal pattern. D. Call the health care provider immediately.

C. Fetal accelerations denote an intact central nervous system and appropriate oxygenation levels demonstrated by an increase in heart rate associated with fetal movement. Accelerations are a reassuring pattern, so no intervention is needed.

16. A nurse is documenting fetal lie of a woman in labor. Which term would the nurse most likely use? A) Flexion B) Extension C) Longitudinal D) Cephalic

C. Fetal lie refers to the relationships of the long axis (spine) of the fetus to the long axis (spine) of the mother. There are two primary lies: longitudinal and transverse.

When assessing fetal heart rate, the nurse finds a heart rate of 175 bpm, accompanied by a decrease in variability and late decelerations. Which of the following would the nurse do next? A) Have the woman change her position. B) Administer oxygen. C) Notify the health care provider. D) Continue to monitor the pattern every 15 minutes.

C. Fetal tachycardia as evidenced by a fetal heart rate greater than 160 bpm accompanied by a decrease in variability and late decelerations is an ominous sign indicating the need for prompt intervention. The health care provider should be notified immediately and then measures should be instituted such as having the woman lie on her side and administering oxygen.

A nurse is providing care to a woman during the third stage of labor. Which of the following would alert the nurse that the placenta is separating? A) Boggy, soft uterus B) Uterus becoming discoid shaped C) Sudden gush of dark blood from the vagina D) Shortening of the umbilical cord

C. Sudden gush of dark blood from the vagina

A woman is admitted to the labor and birthing suite. Vaginal examination reveals that the presenting part is approximately 2 cm above the ischial spines. The nurse documents this finding as: A) +2 station B) 0 station C) -2 station D) Crowning

C. The ischial spines serve as landmarks and are designated as zero status. If the presenting part is palpated higher than the maternal ischial spines, a negative number is assigned. Therefore, the nurse would document the finding as -2 station.

6. A client has not received any medication during her labor. She is having frequent contractions every 1 to 2 minutes and has become irritable with her coach and no longer will allow the nurse to palpate her fundus during contractions. Her cervix is 8 cm dilated and 90% effaced. The nurse interprets these findings as indicating: A) Latent phase of the first stage of labor B) Active phase of the first stage of labor C) Transition phase of the first stage of labor D) Pelvic phase of the second stage of labor

C. The transition phase is characterized by cervical dilation of 8 to 10 cm, effacement of 80% to 100%, contractions that are strong, painful, and frequent (every 1 to 2 minutes) and last 60 to 90 seconds, and irritability, apprehension, and feelings of loss of control.

A nurse is explaining the use of therapeutic touch as a pain relief measure during labor. Which of the following would the nurse include in the explanation? A) "This technique focuses on manipulating body tissues." B) "The technique requires focusing on a specific stimulus." C) "This technique redirects energy fields that lead to pain." D) "The technique involves light stroking of the abdomen with breathing."

C. "This technique redirects energy fields that lead to pain." -Therapeutic touch is an energy therapy and is based on the premise that the body contains energy fields that lead to either good or ill health and that the hands can be used to redirect the energy fields that lead to pain. - Attention focusing and imagery involve focusing on a specific stimulus. - Massage focuses on manipulating body tissues. - Effleurage involves light stroking of the abdomen in rhythm with breathing.

A woman gave birth to a newborn via vaginal delivery with the use of a vacuum extractor. The nurse would be alert for which of the following in the newborn? A) Asphyxia B) Clavicular fracture C) Caput succedaneum D) Central nervous system injury

Caput succedaneum Use of forceps or a vacuum extractor poses the risk of tissue trauma, such as ecchymoses, facial and scalp lacerations, facial nerve injury, cephalhematoma, and caput succedaneum.

When preparing a teaching plan for a group of first-time pregnant women, the nurse expects to review how maternity care has changed over the years. Which of the following would the nurse include when discussing events of the 20th century? A) Epidemics of puerperal fever B) Performance of the first cesarean birth C) Development of the x-ray to assess pelvic size D) Creation of free-standing birth center

Creation of free-standing birth centers

Many women develop iron-deficient anemia during pregnancy. What diagnostic criteria would the nurse monitor for to determine anemia in the pregnant woman? A. Blood pressure of 100/68 B. Heart rate of 84 C.Hemoglobin of 13 or lower D. Hematocrit of 32% or less

D R: Iron-deficiency anemia is diagnosed in a pregnant woman if the hematocrit is less that 33% or the hemoglobin is less than 11 g/dL. Tachycardia, hypotension and tachypnea are all symptoms of iron-deficiency anemia but are not diagnostic criteria.

A woman with type 2 diabetes is considering becoming pregnant and asks the nurse whether she will be able to continue taking her current oral hypoglycemics. The nurse's response will point out which factor? A. are usually suggested primarily for women who develop gestational diabetes. B. can be taken until the degeneration of the placenta occurs. C. can be used as long as they control serum glucose levels. D. have been shown to be effective and safe in recent short term studies.

D R: Recent studies have examined the use of oral hypoglycemic medications in pregnancy with much success. Several studies have used glyburide with promising results. Many health care providers are using glyburide and metformin as an alternative to insulin therapy because they do not cross the placenta and therefore do not cause fetal/neonatal hypoglycemia. Some oral hypoglycemic medications are considered safe and may be used if nutrition and exercise are not adequate alone. Maternal and newborn outcomes are similar to those seen in women who are treated with insulin. Oral hypoglycemic agents, however, must be further investigated to determine their safety with confidence and provide better treatment options for diabetes in pregnancy. Currently, there is a growing acceptance of glyburide use as a primary therapy for gestational diabetes. Glyburide and metformin have also been found to be safe, effective, and economical for the treatment of gestational diabetes, although neither drug has been approved by the FDA for use in pregnancy.

The health care provider of a newly pregnant client determines the woman also has mitral stenosis and will need appropriate therapy. Which medication should the nurse prepare to teach this client to provide her with the best possible care? A. warfarin B. digoxin C. aspirin D. heparin

D This client has an increased risk for developing blood clots. If an anticoagulant is required, heparin is the drug of choice as it does not cross the placenta barrier. Warfarin crosses the placenta and may have teratogenic effects. Aspirin is not recommended in this situation. If digoxin is not used to prevent blood clots.

A nursing instructor is preparing a class discussion on the trends in health care and health care delivery over the past several centuries. When discussing the changes during the past century, which of the following would the instructor be least likely to include? A) Disease prevention B) Health promotion C) Wellness D) Analysis of morbidity and mortality

D) Analysis of morbidity and mortality

A nurse is assigned to care for an Asian American client. The nurse develops a plan of care with the understanding that based on this client's cultural background, the client most likely views illness as which of the following? A) Caused by supernatural forces. B) A punishment for sins. C) Due to spirits or demons. D) From an imbalance of yin and yang

D) From an imbalance of yin and yang

When assessing a pregnant woman in her last trimester, which question would be most appropriate to use to gather information about weight gain and fluid retention? A) "What's your usual dietary intake for a typical day?" B) "What size maternity clothes are you wearing now?" C) "How puffy does your face look by the end of a day?" D) "How swollen do your ankles appear before you go to bed?

D) "How swollen do your ankles appear before you go to bed?

10.When assessing a pregnant woman with heart disease throughout the antepartal period, the nurse would be especially alert for signs and symptoms of cardiac decompensation at which time? A) 16 to 20 weeks gestation B) 20 to 24 weeks gestation C) 24 to 28 weeks gestation D) 28 to 32 weeks gestation

D) 28 to 32 weeks gestation

A nurse is providing care to several pregnant women at the clinic. The nurse would screen for group B streptococcus infection in a client at: A) 16 weeks gestation B) 28 week gestation C) 32 weeks gestation D) 36 weeks gestation

D) 36 weeks gestation

The nurse is making a home visit to a client who had a cesarean birth 3 days ago. Assessment reveals that the client is complaining of intermittent pain, rating it as 8 on a scale of 1 to 10. She states, "I'm pretty tired. And with this pain, I haven't been drinking and eating like I should. The medication helps a bit but not much. My mom has been helping with the baby." Her incision is clean, dry, and intact. Which nursing diagnosis would the nurse identify as the priority for this client? A) Impaired skin integrity related to cesarean birth incision B) Fatigue related to effects of surgery and caretaking activities C) Imbalanced nutrition, less than body requirements related to poor fluid and food intake D) Acute pain related to incision and cesarean birth

D) Acute pain related to incision and cesarean birth

A client who is in labor presents with shoulder dystocia of the fetus. Which is an important nursing intervention? A) Assess for prolonged second stage of labor with arrest of descent. B) Assess for reports of intense back pain in first stage of labor. C) Anticipate possible use of forceps to rotate to anterior position at birth. D) Assist with positioning the woman in squatting position.

D) Assist with position the woman in squatting position. The nurse caring for the client in labor with shoulder dystocia of the fetus should assist with positioning the client in squatting position. The client can also be helped into the hands and knees position or lateral recumbent position for birth, to free the shoulders.

When assessing a woman at follow-up prenatal visits, the nurse would anticipate which of the following to be performed? A) Hemoglobin and hematocrit B) Urine for culture C) Fetal ultrasound D) Fundal height measurement

D) Fundal height measurement

A nurse is engaged in providing family-centered care for a woman and her family. The nurse is providing instrumental support with which activity? A) Explaining to the woman and family what to expect during the birth process. B) Assisting the woman in breathing techniques to cope with labor contractions. C) Reinforcing the woman's role as a mother after birth D) Helping the family obtain extra financial help for prescribed phototherapy

D) Helping the family obtain extra financial help for prescribed phototherapy

A pregnant woman asks the nurse, I'm a big coffee drinker. Will the caffeine in my coffee hurt my baby? Which response by the nurse would be most appropriate? A) The caffeine in coffee has been linked to birth defects. B) Caffeine has been shown to cause growth restriction in the fetus. C) Caffeine is a stimulant and needs to be avoided completely. D) If you keep your intake to less than 300 mg/day, you should be okay.

D) If you keep your intake to less than 300 mg/day, you should be okay.

The nurse is explaining the events that lead up to ovulation. Which hormone would the nurse identify as being primarily responsible for ovulation? A) Estrogen B) Progesterone C) Follicle-stimulating hormone D) Luteinizing hormone

D) Luteinizing hormone At ovulation, a mature follicle ruptures in response to a surge of luteinizing hormone. Estrogen is predominant at the end of the follicular phase, directly preceding ovulation. Progesterone peaks 5 to 7 days after ovulation. Follicle-stimulating hormone is highest during the first week of the follicular phase of the cycle.

A nurse is counseling a pregnant woman with rheumatoid arthritis about medications that can be used during pregnancy. Which drug would the nurse emphasize as being contraindicated at this time? A) Hydroxychloroquine B) Nonsteroidal anti-inflammatory drug C) Glucocorticoid D) Methotrexate

D) Methotrexate

Which medication would the nurse question if ordered to control a pregnant woman's asthma? A) Budesonide B) Albuterol C) Salmeterol D) Oral prednisone

D) Oral prednisone

A nursing instructor is presenting a class for a group of students about community-based nursing interventions. The instructordetermines that additional teaching is needed when the students identify which of the following? A) Conducting childbirth education classes B) Counseling a pregnant teen with anemia C) Consulting with a parent of a child who is vomiting D) Performing epidemiologic investigations

D) Performing epidemiologic investigations

After teaching a group of pregnant women about breast-feeding, the nurse determines that the teaching was successful when the group identifies which hormone as important for the production of breast milk after childbirth? A) Placental estrogen B) Progesterone C) Gonadotropin-releasing hormone D) Prolactin

D) Prolactin

During a nonstress test, when monitoring the fetal heart rate, the nurse notes that when the expectant mother reports fetal movement, the heart rate increases 15 beats or more above the baseline. The nurse interprets this as: A) Variable decelerations B) Fetal tachycardia C) A nonreactive pattern D) Reactive pattern

D) Reactive pattern

A woman comes to the clinic complaining that she has little sexual desire. As part of the client's evaluation, the nurse would anticipate the need to evaluate which hormone level? A) Progesterone B) Estrogen C) Gonadotropin-releasing hormone D) Testosterone

D) Testosterone

A nursing instructor is describing the various childbirth methods. Which of the following would the instructor include as part of the Lamaze method? A) Focus on the pleasurable sensations of childbirth B) Concentration on sensations while turning on to own bodies C) Interruption of the fear-tension-pain cycle D) Use of specific breathing and relaxation techniques

D) Use of specific breathing and relaxation techniques

The nurse is creating a diagram that illustrates the components of the male reproductive system. Which structure would be inappropriate for the nurse to include as an accessory gland? A) Seminal vesicles B) Prostate gland C) Cowper's glands D) Vas deferens

D) Vas deferens

A client who is HIV-positive is in her second trimester and remains asymptomatic. She voices concern about her newborns risk for the infection. Which of the following statements by the nurse would be most appropriate? A)Youll probably have a cesarean birth to prevent exposing your newborn. B)Antibodies cross the placenta and provide immunity to the newborn. C)Wait until after the infant is born and then something can be done. D)Antiretroviral medications are available to help reduce the risk of transmission.

D)Antiretroviral medications are available to help reduce the risk of transmission.

Assessment of a pregnant woman and her fetus reveals tachycardia and hypertension. There is also evidence suggesting vasoconstriction. The nurse would question the woman about use of which substance? A)Marijuana B)Alcohol C)Heroin D)Cocaine

D)Cocaine

A woman with diabetes is considering becoming pregnant. She asks the nurse whether she will be able to take oral hypoglycemics when she is pregnant. The nurses response is based on the understanding that oral hypoglycemics: A)Can be used as long as they control serum glucose levels B)Can be taken until the degeneration of the placenta occurs C)Are usually suggested primarily for women who develop gestational diabetes D)Show promising results but more studies are needed to confirm their effectiveness

D)Show promising results but more studies are needed to confirm their effectiveness

12. A fetus is assessed at 2 cm above the ischial spines. The nurse would document fetal station as: A) +4 B) +2 C) 0 D) -2

D.

The rationale for using a prostaglandin gel for a client prior to the induction of labor is to: a. Stimulate uterine contractions b. Numb cervical pain receptors c. Prevent cervical lacerations d. Soften and efface the cervix

D. Prostaglandins soften and thin out the cervix in preparation for labor induction. Although they do irritate the uterus, they aren't as effective as oxytocin in stimulating contractions. Prostaglandin gel would stimulate cervical nerve receptors rather than numb them. - Prostaglandins have no power to prevent cervical lacerations.

22. A nurse is caring for several women in labor. The nurse determines that which woman is in the transition phase of labor? A) Contractions every 5 minutes, cervical dilation 3 cm B) Contractions every 3 minutes, cervical dilation 5 cm C) Contractions every 21/2 minutes, cervical dilation 7 cm D) Contractions every 1 minute, cervical dilation 9 cm

D. The transition phase is characterized by strong contractions occurring every 1 to 2 minutes and cervical dilation from 8 to 10 cm.

A client states, "I think my waters broke! I felt this gush of fluid between my legs." The nurse tests the fluid with Nitrazine paper and confirms membrane rupture if the paper turns: A) Yellow B) Olive green C) Pink D) Blue

D. Amniotic fluid is alkaline and turns Nitrazine paper blue. * Nitrazine paper that remains yellow to olive green suggests that the membranes are most likely intact.

When reviewing the medical record of a client, the nurse notes that the woman has a condition in which the fetus cannot physically pass through the maternal pelvis. The nurse interprets this as: A. Cervical insufficiency B. Contracted pelvis C. Maternal disproportion D. Fetopelvic disproportion

D. Fetopelvic disproportion is defined as a condition in which the fetus is too large to pass through the maternal pelvis.

The nurse teaches a primigravida client that lightening occurs about 2 weeks before the onset of labor. The mother will most likely experience which of the following at that time? A) Dysuria B) Dyspnea C) Constipation D) Urinary frequency

D. Lightening refers to the descent of the fetal head into the pelvis and engagement. With this descent, pressure on the diaphragm decreases, easing breathing, but pressure on the bladder increases, leading to urinary frequency. Dysuria might indicate a urinary tract infection. Constipation may occur throughout pregnancy due to decreased peristalsis, but it is unrelated to lightening.

20. A nurse is assessing a woman in labor. Which finding would the nurse identify as a cause for concern during a contraction? A) Heart rate increase from 76 bpm to 90 bpm B) Blood pressure rise from 110/60 mm Hg to 120/74 C) White blood cell count of 12,000 cells/mm3 D) Respiratory rate of 10 breaths /minute

D. During labor, there is an increase in respiratory rate with greater oxygen consumption due to the increase in metabolism. *A drop in respiratory rate would be a cause for concern.

A client who is HIV-positive is in her second trimester and remains asymptomatic. She voices concern about her newborns risk for the infection. Which of the following statements by the nurse would be most appropriate? A)Youll probably have a cesarean birth to prevent exposing your newborn. B)Antibodies cross the placenta and provide immunity to the newborn. C)Wait until after the infant is born and then something can be done. D)Antiretroviral medications are available to help reduce the risk of transmission.

D. Drug therapy is the mainstay of treatment for pregnant women infected with HIV. The goal of therapy is to reduce the viral load as much as possible; this reduces the risk of transmission to the fetus. Decisions about the method of delivery should be based on the woman's viral load, duration of ruptured membranes, progress of labor, and other pertinent clinical factors. The newborn is at risk for HIV because of potential perinatal transmission. Waiting until after the infant is born may be too late.

A woman in labor has chosen to use hydrotherapy as a method of pain relief. Which statement by the woman would lead the nurse to suspect that the woman needs additional teaching? A) "The warmth and buoyancy of the water has a nice relaxing effect." B) "I can stay in the bath for as long as I feel comfortable." C) "My cervix should be dilated more than 5 cm before I try using this method." D) "The temperature of the water should be at least 105°F."

D. The water temperature should not exceed body temperature. Therefore, a temperature of 105° F would be too warm.

The health care provider of a newly pregnant client determines the woman also has mitral stenosis and will need appropriate therapy. Which medication should the nurse prepare to teach this client to provide her with the best possible care? A. warfarin B. digoxin C. aspirin D. heparin

D. This client has an increased risk for developing blood clots. If an anticoagulant is required, heparin is the drug of choice as it does not cross the placenta barrier. Warfarin crosses the placenta and may have teratogenic effects. Aspirin is not recommended in this situation. If digoxin is not used to prevent blood clots.

A group of nursing students are reviewing the various medications used for pain relief during labor. The students demonstrate understanding of the information when they identify which agent as the most commonly used opioid? A) Butorphanol B) Nalbuphine C) Fentanyl D) Meperidine

D. Meperidine

A woman who is 42 weeks pregnant comes to the clinic. Which of the following would be most important? A) Determining an accurate gestational age B) Asking her about the occurrence of contractions C) Checking for spontaneous rupture of membranes D) Measuring the height of the fundus

Determining an accurate gestational age Incorrect dates account for the majority of prolonged or postterm pregnancies; many women have irregular menses and thus cannot identify the date of their last menstrual period accurately. Therefore, accurate gestational dating via ultrasound is essential. - Asking about contractions and checking for ruptured membranes, although important assessments, would be done once the gestational age is confirmed.

Tocolytic

Drug that inhibits uterine contractions.

A nurse is assessing a pregnant woman who has come to the clinic. The woman reports that she feels some heaviness in her thighs since yesterday. The nurse suspects that the woman may be experiencing preterm labor based on which additional assessment findings? A) Dull low backache B) Malodorous vaginal discharge C) Dysuria D) Constipation

Dysuria Symptoms of preterm labor are often subtle and may include change or increase in vaginal discharge with mucus, water, or blood in it; pelvic pressure; low, dull backache; nausea, vomiting or diarrhea, and intestinal cramping with or without diarrhea.

one of the P's of labor, power has three phases which are:

Each contraction has three phases: - increment or the buildup of the contraction - acme or the peak or highest intensity - the decrement or relaxation of the uterine muscle fibers.

After teaching a group of students about pregnancy-related mortality, the instructor determines that additional teaching is needed when the students identify which condition as a leading cause? A) Hemorrhage B) Embolism C) Obstructed labor D) Infection

Embolism

Assessment of a woman in labor who is experiencing hypertonic uterine dysfunction would reveal contractions that are: A) Well coordinated B) Poor in quality C) Rapidly occurring D) Erratic

Erratic Hypertonic contractions occur when the uterus never fully relaxes between contractions, making the contractions erratic and poorly coordinated because more than one uterine pacemaker is sending signals for contraction.

The nurse is assessing the laboring client to determine fetal oxygenation status. What indirect assessment method will the nurse likely use? A. External electronic fetal monitoring B. Fetal blood pH C. Fetal oxygen saturation D. Fetal position

External electronic fetal monitoring is the first option that we use

____ ____ allows the shoulders to rotate internally to fit the maternal pelvis

External rotation

A pregnant client in labor has to undergo a sonogram to confirm the fetal position of a shoulder presentation. The nurse should assess for which of the following conditions associated with shoulder presentation during a vaginal birth?

Fetal anomalies

A primigravida whose labor was initially progressing normally is now experiencing a decrease in the frequency and intensity of her contractions. The nurse would assess the woman for which condition? A) A low-lying placenta B) Fetopelvic disproportion C) Contraction ring D) Uterine bleeding

Fetopelvic disproportion The woman is experiencing dystocia most likely due to hypotonic uterine dysfunction and fetopelvic disproportion associated with a large fetus. - A low-lying placenta, contraction ring, or uterine bleeding would not be associated with a change in labor pattern.

When assessing a family for possible barriers to health care, the nurse would consider which factor to be most important? A) Language B) Health care workers' attitudes C) Transportation D) Finances

Finances

A nurse is completing the assessment of a woman admitted to the labor and birth suite.Which of the following would the nurse expect to include as part of the physical assessment? (Select all that apply.) A) Current pregnancy history B) Fundal height measurement C) Support system D) Estimated date of birth E) Membrane status F) Contraction pattern

Fundal height measurement Membrane status Contraction pattern As part of the admission physical assessment, the nurse would assess: - fundal height - membrane status and contractions * Current pregnancy history, support systems, and estimated date of birth would be obtained when collecting the maternal health history.

A pregnant client with a history of spinal injury is being prepared for a cesarean birth. Which method of anesthesia is to be administered to the client? A. Local infiltration B. Epidural block C. Regional anesthesia D. General anesthesia

General anesthesia is administered in emergency cesarean births

The nurse would be alert for possible placental abruption during labor when assessment reveals which of the following? A) Macrosomia B) Gestational hypertension C) Gestational diabetes D) Low parity

Gestational hypertension Risk factors for placental abruption include preeclampsia, gestational hypertension, seizure activity, uterine rupture, trauma, smoking, cocaine use, coagulation defects, previous history of abruption, domestic violence, and placental pathology.

The nurse is working with a group of community health members to develop a plan to address the special health needs of women. Which of the following conditions would the group address as the major problem? A) Smoking B) Heart disease C) Diabetes D) Cancer

Heart disease

A pregnant woman tests positive for HBV. Which of the following would the nurse expect to administer? A) HBV immune globulin B) HBV vaccine C) Acylcovir D) Valacyclovir

Hepatitis B A) HBV immune globulin

A nurse is caring for a pregnant client who is in labor. Which of the following maternal physiologic responses should the nurse monitor for in the client as the client progresses through childbirth?

Increase in HR Increase in BP Increase RR

A nurse is describing the risks associated with prolonged pregnancies as part of an inservice presentation. Which of the following would the nurse be least likely to incorporate in the discussion as an underlying reason for problems in the fetus? A) Aging of the placenta B) Increased amniotic fluid volume C) Meconium aspiration D) Cord compression

Increased amniotic fluid volume

Amnioinfusion

Infusion of a sterile isotonic solution into the uterine cavity during labor to reduce umbilical cord compression; may also be done to dilute meconium in amniotic fluid and reduce the risk that the infant will aspirate thick meconium at birth.

A pregnant client is admitted to a maternity clinic for childbirth. The client wishes to adopt the kneeling position during labor. The nurse knows that which of the following is the advantage of adopting a kneeling position during labor?

It helps to rotate the fetus in a posterior position

After spontaneous rupture of membranes, the nurse notices a prolapsed cord. The nurse immediately places the woman in which position? A) Supine B) Side-lying C) Sitting D) Knee-chest

Knee-chest Pressure on the cord needs to be relieved. Therefore, the nurse would position the woman in a modified Sims, Trendelenburg, or knee-chest position. Supine, side-lying, or sitting would not provide relief of cord compression.

A group of nursing students are reviewing information about methods used for cervical ripening. The students demonstrate understanding of the information when they identify which of the following as a mechanical method? A) Herbal agents B) Laminaria C) Membrane stripping D) Amniotomy

Laminaria is a hygroscopic dilator that is used as a mechanical method for cervical ripening. Herbal agents are a nonpharmacologic method. Membrane stripping and amniotomy are considered surgical methods.

What are the phases of labor (in proper order)

Latent Phase Active Phase Transition Phase Pelvic Phase Perineal Phase Placental Separation Placental Expulsion

A client in her third trimester of pregnancy arrives at a health care facility complaining of cramping and low back pain; she also notes that she is urinating more frequently and that her breathing has become easier the past few days. Physical examination conducted by the RN indicates that the client has what?

Lightening

A pregnant client is admitted to a maternity clinic for childbirth. Which assessment finding indicates that the client's fetus is in the transverse lie position?

Long axis of fetus is perpendicular to that of client

A nurse is required to obtain the fetal heart rate (FHR) for a pregnant client. If the presentation is cephalic, which maternal site should the nurse monitor to hear the FHR clearly?

Lower quadrant of the maternal abdomen

A nurse is caring for a client who is in the first stage of labor. The client is experiencing extreme pain due to the labor. the nurse understands that which of the following is causing extreme pain the client? Select all

Lower uterine segment distention Stretching and tearing of structures Dilation of cervix

A nurse caring for a pregnant client in labor observes that the fetal heart rate is below 110 per minute. Which interventions should the nurse perform?

O2 mask

A woman in labor is experiencing hypotonic uterine dysfunction. Assessment reveals no fetopelvic disproportion. Which group of medications would the nurse expect to administer? A) Sedatives B) Tocolytics C) Oxytocins D) Corticosteroids

Oxytocins - For hypotonic labor, a uterine stimulant such as oxytocin may be ordered once fetopelvic disproportion is ruled out. - Sedatives might be helpful for the woman with hypertonic uterine contractions to promote rest and relaxation. - Tocolytics would be ordered to control preterm labor. - Corticosteroids may be given to enhance fetal lung maturity for women experiencing preterm labor.

The fetus of a woman in labor is determined to be in persistent occiput posterior position. Which of the following would the nurse identify as the priority intervention? A) Position changes B) Pain relief measures C) Immediate cesarean birth D) Oxytocin administration

Pain relief measures Intense back pain is associated with persistent occiput posterior position. Therefore, a priority is to provide pain relief measures. Position changes that can promote fetal head rotation are important after the nurse institutes pain relief measures. Additionally, the woman's ability to cooperate and participate in these position changes is enhanced when she is experiencing less pain. *Oxytocin would add to the woman's already high level of pain.

What indicates placenta separation?

Placental separation is indicated by the uterus changing shape to globular and upward rising of the uterus. Additional signs include a sudden trickle of blood from the vaginal opening, and lengthening (not shortening) of the umbilical cord.

A woman's amniotic fluid is noted to be cloudy. The nurse interprets this finding as?

Possible infection

A pregnant client is admitted to a maternity clinic after experiencing contractions. The nurse knows that what are the importance of the pauses between contractions during labor?

Restoration of blood flow to uterus and placenta

The ____ suture is located between the parietal bones and divides the skull into right and left halves

Sagittal

Which finding would indicate to the nurse that a woman's cervix is ripe in preparation for labor induction? A) Posterior position B) Firm C) Closed D) Shortened

Shortened A ripe cervix is shortened, centered (anterior), softened, and partially dilated.

After teaching a group of students about fetal heart rate patterns, the instructor determines the need for additional teaching when the students identify which of the following as indicating normal fetal acid-base status? (Select all that apply.) A) Sinusoidal pattern B) Recurrent variable decelerations C) Fetal bradycardia D) Absence of late decelerations E) Moderate baseline variability

Sinusoidal pattern Recurrent variable decelerations Fetal bradycardia

Research validates that nonmoving back-lying positions such as supine and lithotomy positions during labor are not healthy.

TRUE

A pregnant client wants to know why the labor of a first-time-pregnant woman usually lasts longer than that of a woman who has already delivered once and is pregnant a second time. What explanation should the nurse offer the client?

The cervix takes around 12-16 hours to dilate during first pregnancy

When caring for a client during the active phase of labor without continuous electronic fetal monitoring, the nurse would intermittently assess FHR every: A. 15 to 30 minutes B. 5 to 10 minutes C. 45 to 60 minutes D. 60 to 75 minutes

The correct response is A. The current recommendation is that intermittent FHR is assessed every 15 minutes during the active phase of labor.

How is fetal presenting part determined?

The fetal presenting part is determined by vaginal examination and is commonly the head (cephalic), pelvis (breech), or shoulder.

These drugs promote uterine relaxation by interfering with uterine contraction

Tocolytic

A pregnant client undergoing labor induction is receiving an oxytocin infusion. Which of the following findings would require immediate intervention? a) Fetal heart rate of 150 beats/minute B) Contractions every 2 minutes, lasting 45 seconds C) Uterine resting tone of 14 mm Hg D) Urine output of 20 mL/hour

Urine output of 20 mL/hour Oxytocin can lead to water intoxication. Therefore, a urine output of 20 mL/hour is below acceptable limits of 30 mL/hour and requires intervention. - Contractions should occur every 2 to 3 minutes, lasting 40 to 60 seconds. - A uterine resting tone greater than 20 mm Hg would require intervention.

A pregnant woman is receiving misoprostol to ripen her cervix and induce labor. The nurse assesses the woman closely for which of the following? A) Uterine hyperstimulation B) Headache C) Blurred vision D) Hypotension

Uterine hyperstimulation A major adverse effect of the obstetric use of Cytotec is hyperstimulation of the uterus, which may progress to uterine tetany with marked impairment of uteroplacental blood flow, uterine rupture (requiring surgical repair, hysterectomy, and/or salpingo-oophorectomy), or amniotic fluid embolism. Headache, blurred vision, and hypotension are associated with magnesium sulfate.

A woman with a history of crack cocaine abuse is admitted to the labor and birth area. While caring for the client, the nurse notes a sudden onset of fetal bradycardia. Inspection of the abdomen reveals an irregular wall contour. The client also complains of acute abdominal pain that is continuous. Which of the following would the nurse suspect? A) Amniotic fluid embolism B) Shoulder dystocia C) Uterine rupture D) Umbilical cord prolaps

Uterine rupture Uterine rupture is associated with crack cocaine use, and generally the first and most reliable sign is sudden fetal distress accompanied by acute abdominal pain, vaginal bleeding, hematuria, irregular wall contour, and loss of station in the fetal presenting part.

A nurse is reviewing the fetal heart rate pattern and observes abrupt decreases in FHR below the baseline, appearing as a U-shape. The nurse interprets these changes as reflecting which of the following? A) Early decelerations B) Variable decelerations C) Prolonged decelerations D) Late decelerations

Variable decelerations present as visually apparent abrupt decreases in FHR below baseline and have an unpredictable shape on the FHR baseline, possibly demonstrating no consistent relationship to uterine contractions. The shape of variable decelerations may be U, V, or W, or they may not resemble other patterns.

A client has been in labor for 10 hours, with contractions occurring consistently about 5 minutes apart. The resting tone of the uterus remains at about 9 mm Hg, and the strength of the contractions averages 21 mm Hg. The nurse recognizes which condition in this client?

With hypotonic uterine contractions, the number of contractions is unusually infrequent (not more than two or three occurring in a 10-minute period). The resting tone of the uterus remains less than 10 mm Hg, and the strength of contractions does not rise above 25 mm Hg.

zygote

a diploid cell resulting from the fusion of two haploid gametes; a fertilized ovum. pg.337

placenta

a flattened circular organ in the uterus of pregnant eutherian mammals, nourishing and maintaining the fetus through the umbilical cord. pg.336

umbilical cord

a flexible cordlike structure containing blood vessels and attaching a human or other mammalian fetus to the placenta during gestation.

trophoblast

a layer of tissue on the outside of a mammalian blastula, supplying the embryo with nourishment and later forming the major part of the placenta. pg. 336

morula

a solid ball of cells resulting from division of a fertilized ovum, and from which a blastula is formed. pg. 337

genes

a unit of heredity that is transferred from a parent to offspring and determines some characteristic of the offspring.

Fourteen-year-old Ann and her parents have presented at the obstetrician's office in the second trimester, the teen had been hiding the pregnancy. The nurse is helping them develop a plan of care. What is the best thing she can say to the clearly angry parents? a) "I know you must be very upset and angry about Ann's pregnancy but because she's still an adolescent herself, she'll need your guidance in making nutritional and health choices that will be good for the baby and for herself." b) "Anger won't help this situation at all. You'll only push Ann away and she'll be less likely to make good choices." c) "Ann needs to make decisions about this pregnancy for herself." d) "I understand your anger but if you had encouraged Ann to use condoms she would probably not be in this situation."

a) "I know you must be very upset and angry about Ann's pregnancy but because she's still an adolescent herself, she'll need your guidance in making nutritional and health choices that will be good for the baby and for herself." Rationale: The nurse needs to acknowledge the anger of the parents but remember her role is as the patient advocate. The nurse needs to encourage the relationship of support between the parents and the patient. Option A is incorrect as it is attempting to lay down ground rules between the patient and her parents, it also does not acknowledge the parents' feelings in this situation. Option C is incorrect as it might be interpreted as a lecture to the parents, and it does not acknowledge their feelings in this situation. Option D is incorrect; again it sounds like lecturing and it places blame on the parents, which is inappropriate.

A pregnant woman with type 2 diabetes is scheduled for a laboratory test of glycosylated hemoglobin (HbA). What does the nurse tell the patient is a normal level for this test? a) 6% b) 14% c) 8% d) 12%

a) 6% Rationale: The upper normal level of HbA is 6% of total hemoglobin.

A client with juvenile rheumatoid arthritis (JRA) is in week 38 of her pregnancy. Which of the following interventions should the nurse make with this client? a) Ask the client to decrease her intake of salicylates b) Perform the Snellen eye test c) Urge the client to be on bed rest d) Advise the client to continue her normal dosage of methotrexate

a) Ask the client to decrease her intake of salicylates Rationale: Although women with JRA should continue to take their medications during pregnancy to prevent joint damage, large amounts of salicylates have the potential to lead to increased bleeding at birth or prolonged pregnancy. The infant may be born with a bleeding defect and may also experience premature closure of the ductus arteriosus because of the drug's effects. For this reason, a woman is asked to decrease her intake of salicylates approximately 2 weeks before term. A number of women also take low-dose methotrexate, a carcinogen. As a rule, they should stop taking this prepregnancy because of the danger of head and neck defects in the fetus. There is no need for the client to be on bed rest or to perform the Snellen eye test.

A woman in week 40 of her pregnancy has developed a urinary tract infection (UTI). The nurse recognizes that which of the following treatments would be safe and appropriate to use with this client? (Select all that apply.) a) Cephalosporins b) Heparin c) Amoxicillin d) Sulfonamides e) Tetracyclines f) Ampicillin

a) Cephalosporins c) Amoxicillin f) Ampicillin Rationale: Amoxicillin, ampicillin, and cephalosporins are effective against most organisms causing UTIs and are safe antibiotics during pregnancy. The sulfonamides can be used early in pregnancy but not near term because they can interfere with protein binding of bilirubin, which then leads to hyperbilirubinemia in the newborn. Tetracyclines are contraindicated during pregnancy as they cause retardation of bone growth and staining of the fetal teeth. Heparin is an anticoagulant and is used to prevent clot formation; it would not be prescribed for a UTI.

A woman with a positive history of genital herpes is in active labor. She has small pin-point vesicles in the perineum area. Her membranes are ruptured, she is dilated 5cm, effaced 70%. The nurse should anticipate what type of delivery? a) Cesarean. b) Forceps assisted. c) Spontaneous vaginal. d) Vacuum assisted.

a) Cesarean. Rationale: An active herpes infection can be passed to the fetus during labor or with ruptured amniotic membranes. The nurse should anticipate the infant will be delivered via a cesarean birth. The risk of transmitting herpes to the baby would be increased if the baby were born by spontaneous vaginal delivery, vacuum assisted delivery, or forceps assisted delivery.

A 29-year-old client has gestational diabetes. The nurse is teaching her about managing her glucose levels. Which therapy would be most appropriate for this client? a) Diet b) Glucagon c) Long-acting insulin d) Oral hypoglycemic drugs

a) Diet Rationale: Clients with gestational diabetes are usually managed by diet alone to control their glucose intolerance. Long-acting insulin usually isn't needed for blood glucose control in the client with gestational diabetes. Oral hypoglycemic drugs are contraindicated in pregnancy. Glucagon raises blood glucose and is used to treat hypoglycemic reactions.

The nurse should encourage a pregnant client who is taking short-acting insulin for her diabetes to avoid eating after self-administering the insulin. a) False b) True

a) False Rationale: Caution women with diabetes to eat almost immediately after injecting short-acting insulin to prevent hypoglycemia before mealtimes.

A pregnant woman with sickle cell anemia comes to the emergency department in crisis. Which of the following would the nurse expect to find? Select all that apply. a) Fever b) Pallor c) Increased skin turgor d) Joint pain e) Fatigue

a) Fever d) Joint pain Rationale: Signs and symptoms of a sickle cell crisis commonly include severe abdominal pain, muscle spasm, leg pains, joint pain, fever, stiff neck, nausea and vomiting, and seizures. Skin turgor would most likely be poor because the client would probably be dehydrated. Pallor and fatigue are associated with sickle cell anemia and would not help identify a crisis.

You are the clinic nurse caring for a pregnant woman in her third trimester. The woman is HIV positive and voices concerns about passing the infection on to her baby. What is your best response? a) If you are taking antiretroviral medications and you don't breastfeed your baby, you greatly reduce the risk of perinatal transmission of the disease. b) Perinatal transmission of the virus is a real fear. Would you like to talk to a social worker? c) There is nothing you can do. You will just have to wait and see if your baby is born HIV positive. d) Do you have other children? Your baby has a one-in-four chance of having HIV at birth, so if you have three other children who are not HIV positive, then this one will be HIV positive.

a) If you are taking antiretroviral medications and you don't breastfeed your baby, you greatly reduce the risk of perinatal transmission of the disease. Rationale: Receiving appropriate antiretroviral treatment during pregnancy and childbirth and refraining from breastfeeding substantially reduce the risk of perinatal transmission.

A 16-year-old girl comes to the public health office and tells you she is pregnant. She is afraid to tell her parents. As a nurse, what is important for you to know that can help this 16 year old? a) Know about community resources for the pregnant teen b) Know who the father of the baby is c) Know who the mother's parents are d) Know what school district she resides in

a) Know about community resources for the pregnant teen Rationale: Be knowledgeable regarding community resources for the pregnant teen. If you or the primary-care practitioner refers the teen to another entity, follow up to make certain the adolescent receives the services for which she was referred. If she does not, try to determine the barriers that prevent her from following through with treatment. Assist her to work through the barriers to obtain needed services.

A pregnant woman at 4 weeks' gestation who has preexisting diabetes mellitus visits her primary care provider for a check-up. Which fetal complications might occur because of this maternal condition? Select all that apply. a) Macrosomia (oversized fetus) b) Respiratory disorder c) Congenital malformations d) Fetus with juvenile diabetes e) Smaller than gestational age baby

a) Macrosomia (oversized fetus) b) Respiratory disorder c) Congenital malformations Rationale: Potential problems during pregnancy involving maternal diabetes mellitus include fetal death, macrosomia (oversized fetus), a fetus with a respiratory disorder, difficult labor, preeclampsia or eclampsia, polyhydramnios, and congenital malformations.

A pregnant woman diagnosed with diabetes should be instructed to do which of the following? a) Notify the physician if unable to eat because of nausea and vomiting. b) Prepare foods with increased carbohydrates to provide needed calories. c) Ingest a smaller amount of food prior to sleep to prevent nocturnal hyperglycemia. d) Discontinue insulin injections until 15 weeks gestation.

a) Notify the physician if unable to eat because of nausea and vomiting. Rationale: During pregnancy, the insulin levels change in response to the production of HPL. The patient needs to alert her provider if she is not able to eat or hold down appropriate amounts of nutrition. The patient is at risk for episodes of hypoglycemia during the first trimester. She should never discontinue insulin therapy without her provider's directions. The increase of carbohydrates needs to be balanced with protein, and smaller meals would result in hypoglycemia rather than hyperglycemia.

A nurse is assessing a client in her seventh month of pregnancy who has an artificial valve prosthesis. The client is taking an oral anticoagulant to prevent the formation of clots at the valve site. Which of the following nursing interventions is most appropriate in this situation? a) Observe the client for signs of petechiae and premature separation of the placenta b) Instruct the client to avoid wearing constrictive knee-high stockings c) Urge the client to discontinue the anticoagulant to prevent pregnancy complications d) Put the client on bed rest

a) Observe the client for signs of petechiae and premature separation of the placenta Rationale: Subclinical bleeding from continuous anticoagulant therapy in the woman has the potential to cause placental dislodgement. Observe a woman who is taking an anticoagulant for signs of petechiae and signs of premature separation of the placenta, therefore, during both pregnancy and labor. The nurse should not urge the client to discontinue the anticoagulant, as this is not within the nurse's scope of practice and, in any case, the client still needs the anticoagulant to prevent clots. Bed rest is prescribed for clients with a thrombus, to prevent it from moving and becoming a pulmonary embolus. Avoiding the use of constrictive knee-high stockings is to prevent thrombus formation.

A woman with an artificial mitral valve develops heart failure at the 20th week of pregnancy. Which of the following measures would you stress with her during the remainder of the pregnancy? a) Obtaining enough rest b) Beginning a low-impact aerobics program c) Discontinuing her prepregnancy anticoagulant d) Maintaining a high fluid intake

a) Obtaining enough rest Rationale: As the blood volume doubles during pregnancy, heart failure can occur. The pregnant woman needs to obtain adequate rest to prevent overworking the heart. Fluid may need to be restricted.

A young woman with scoliosis has just learned that she is pregnant. Several years ago, she had stainless-steel rods surgically implanted on both sides of her vertebrae to strengthen and straighten her spine. However, her pelvis is unaffected by the condition. Which of the following does the nurse anticipate in this woman's pregnancy? a) Potential for greater than usual back pain b) Cesarean birth c) Increased risk of fetal trauma d) Increased risk of miscarriage

a) Potential for greater than usual back pain Rationale: Surgical correction of scoliosis (lateral curvature of the spine) involves implanting stainless-steel rods on both sides of the vertebrae to strengthen and straighten the spine. Such rod implantations do not interfere with pregnancy; a woman may notice more than usual back pain, however, from increased tension on back muscles. If a woman's pelvis is distorted due to scoliosis, a cesarean birth may be scheduled to ensure a safe birth, but this is not required in this scenario. Vaginal birth, if permitted, requires the same management as for any woman. With the improved management of scoliosis, the high maternal and perinatal risks associated with the disorder reported in earlier literature no longer exist.

Which of the following would the nurse expect to find in a newborn of a mother who abuses heroin? a) Sneezing b) Easy consolability c) Vigorous sucking d) Hypotonicity

a) Sneezing Rationale: Newborns of mothers who abuse heroin or other narcotics display irritability, hypertonicity, a high-pitched cry, vomiting, diarrhea, respiratory distress, disturbed sleeping, sneezing, diaphoresis, fever, poor sucking, tremors, and seizures.

Working with pregnant teenagers as a special population requires the nurse to have knowledge of adolescent development. Which of the following is crucial for a positive pregnancy and outcome for the mother and fetus? a) Support network b) Cultural sensitivity c) Acceptance by peers d) Involvement of the father

a) Support network Rationale: One crucial part of management of teenage adolescent pregnancy includes helping the teens to develop an adequate support network. The network may include parents, teachers, friends and the father of the baby in addition to resources needed to provide care for the infant and self. Cultural sensitivity, involvement of the father, and acceptance by peers are important to the teenager who is pregnant, but they are not considered crucial for a positive pregnancy and outcome for the mother and fetus.

A pregnant woman in her second trimester comes to the prenatal clinic for a routine visit. She reports that she has a new kitten. The nurse would have the woman evaluated for which infection? a) Toxoplasmosis b) Parvovirus B19 c) Cytomegalovirus d) Herpes simplex virus

a) Toxoplasmosis Rationale: Toxoplasmosis is transferred by hand to mouth after touching cat feces while changing the litter box or through gardening in contaminated soil. Cytomegalovirus is transmitted via sexual contract, blood transfusions, kissing, and contact with children in daycare centers. Parvovirus B19 is a common self-limiting benign childhood virus that causes fifth disease. A pregnant woman may transmit the virus transplacentally to her fetus if she is exposed to an infected child. Herpesvirus infection occurs by direct contact of the skin or mucous membranes with an active lesion through kissing, sexual contact, or routine skin-to-skin contact.

As a rule, women can receive chemotherapy in the second and third trimesters without adverse fetal effects. a) True b) False

a) True Rationale: none

When caring for a pregnant woman with cardiac problems, the nurse must be alert for signs and symptoms of cardiac decompensation (congestive heart failure), which include a) dyspnea, crackles, irregular weak pulse. b) regular heart rate, hypertension. c) shortness of breath, bradycardia, hypertension. d) increased urinary output, tachycardia, dry cough.

a) dyspnea, crackles, irregular weak pulse. Rationale: Signs of cardiac decompensation to congestive heart failure include crackles in the lungs from fluid, difficulty breathing, and weak pulse from heart exhaustion. The heart rate would not be regular, and a cough would not be dry. The heart rate would increase rather than decrease.

A nurse informs a pregnant woman with cardiac disease that she will need two rest periods each day and a full night's sleep. The nurse further instructs the patient that the best position for this rest is which of the following? a) left lateral recumbent b) right lateral recumbent c) prone d) on her back

a) left lateral recumbent Rationale: The pregnant woman should rest in the left lateral recumbent position to prevent supine hypotension syndrome and increased heart effort.

After teaching a group of nursing students about tocolytic therapy, the instructor determines that the teaching was successful when they identify which drug as being used for tocolysis? (Select all that apply.) A) Nifedipine B) Terbutaline C) Dinoprostone D) Misoprostol E) Indomethacin

a, b, e Medications most commonly used for tocolysis include: - magnesium sulfate (which reduces the muscle's ability to contract) - terbutaline (Brethine, a beta-adrenergic) - indomethacin (Indocin, a prostaglandin synthetase inhibitor) - nifedipine (Procardia, a calcium channel blocker). * Dinoprostone and misoprostol are used to ripen the cervix.

which postoperative intervention should a nurse perform when caring for a client who undergone cesarean birth? A. assess uterine tone and determine fundal firmness B. ensure that the client does not cough or breath deeply

a. * nurse should encoarage client to cough, perform deep-breathing excercises, and use of incentive spirometer every 2 hours

Full-term pregnant client is being assessed for induction of labor. Her bishop score is less than 6. Which order the nurse anticipate? a. insertion of foley catherer into endocervical canal b. prepare client for a cesarian birth c. administer oxytocin IV at 10 mU/minute d. artificial rupture of membrane

a. score less than 6 indicates that cervical ripening method should be used before inducing labor

The assessment of a pregnant client, who is toward the end of her third trimester, reveals that she has increased prostiglandin levels. Which of the following factors should the nurse assess for in the client? Select all that apply

a. reduction in cervical resistance b. Myometrial contraction d. Softening and thinning of the cervix

Contractions every 3 minutes with cervical dilation of 5 cm and contractions every 21/2 minutes with cervical dilation of 7 cm suggest the __________ phase of labor.

active

cleavage (aka mitosis):

after the sperm meets the egg, in the fallopian tube, the mix (zygote) travels towards the uterus via tubal muscular, movements. during travel, the zygote goes through cleavage x 4, where it becomes a a ball of 16 cells known as a morula.

fetopelvic disproportion

also called cephalopelvic; the head of the fetus is larger than the pelvic outlet

teratogen

an agent or factor that causes malformation of an embryo.

cervical ripeness

an assessment of the readiness of the cervix to afface and dilate in response to uterine contractions

___________________ describes the irregular variations or absence of fetal heart rate (FHR) due to erroneous causes on the fetal monitor record.

artifact

amniotomy

artificial rupture of membranes

Phosphatidylglycerol

assess fetal lung activiy

A woman's baby is HIV positive at birth. She asks the nurse if this means the baby will develop AIDS. Which of the following statements would be the nurse's best answer? a) "She already has AIDS. That's what being HIV positive means." b) "The antibodies may be those transferred across the placenta; the baby may not develop AIDS." c) "HIV is transmitted at birth; having a cesarean birth prevented transmission." d) "HIV antibodies do not cross the placenta; this means the baby will develop AIDS."

b) "The antibodies may be those transferred across the placenta; the baby may not develop AIDS." Rationale: Infants born of HIV-positive women test positive for HIV antibodies at birth because these have crossed the placenta. An accurate disease status cannot be determined until the antibodies fade at about 18 months. Testing positive for HIV antibodies does not mean the infant has AIDS. Having a cesarean birth does decrease the risk of transmitting the virus to the infant at birth; it does not prevent the transmission of the disease. HIV antibodies do cross the placenta, which is why babies born of HIV positive mothers are HIV positive.

A pregnant woman with diabetes is having a glycosylated hemoglobin level drawn. Which result would require the nurse to revise the client's plan of care? a) 6.0% b) 8.5% c) 7% d) 5.5%

b) 8.5% Rationale: A glycosylated hemoglobin level of more than 8% indicates poor control and the need for intervention, necessitating a revision in the woman's plan of care.

A woman with a long history of controlled asthma has just had her first antenatal visit for her fourth child. She is late for a meeting and says she knows what to do. What is the best action the nurse can take? a) Note in the chart that the woman was not counseled about her asthma. b) Acknowledge her need to leave but ask her to demonstrate the use of her inhaler and her peak flow meter before she goes; make any necessary corrections to her technique. Remind her to take her regular medications. c) Remind her to continue taking her asthma medications, to monitor her peak flow daily, and to monitor the baby's kicks in the second and third trimesters. d) Schedule an appointment for her to return to discuss her asthma management.

b) Acknowledge her need to leave but ask her to demonstrate the use of her inhaler and her peak flow meter before she goes; make any necessary corrections to her technique. Remind her to take her regular medications. Rationale: Management of asthma during pregnancy is very important, the nurse must document the patient has the proper ability to manage her asthma for her health and the health of the fetus. Reminding the patient to continue taking her prescribed medication to monitor her peak flow daily is not enough. It is the nurse's responsibility to KNOW that the patient knows how to take her medications. Monitoring the baby's kicks in the second and third trimester is an appropriate action. Scheduling a return appointment to discuss asthma management is not appropriate. She could have an asthma attack between the time you see her and the time you schedule a return appointment. Noting in the chart that the woman was not counseled does not relieve the nurse of her obligation to ensure that the woman knows how to use her inhaler and her peak flow meter.

A pregnant client with a history of heart disease has been admitted to a healthcare center with complaints of breathlessness. The client also complains of shortness of breath and easy fatigue when doing ordinary activity. The client's condition is markedly compromised. The nurse would document the client's condition using the New York Heart Association (NYHA) classification system as which class? a) Class IV b) Class III c) Class I d) Class II

b) Class III Rationale: The nurse should classify the client's condition as belonging to class III of NYHA. In class III of NYHA classification, the client will be symptomatic with ordinary activity and her condition is markedly compromised. The client is asymptomatic with all kinds of activity and is in uncompromised state in class I. The client is symptomatic with increased activity and is in slight compromised state in class II. The client is symptomatic when resting and is incapacitated in class IV.

You encourage a woman with gestational diabetes to maintain an active exercise period during pregnancy. Prior to this exercise period, you would advise her to do which of the following? a) Eat a high-carbohydrate snack b) Eat a sustaining-carbohydrate snack c) Inject a bolus of insulin d) Add a bolus of long-acting insulin

b) Eat a sustaining-carbohydrate snack Rationale: Because exercise uses up glucose, women with diabetes should take a sustaining-carbohydrate snack before hard exercise to prevent hypoglycemia.

When educating a pregestational patient on how to control her blood sugar, the nurse knows there are three main facets to glycemic control: diet, exercise and _______. Which of the following is the third facet? a) Glucose tablets b) Insulin c) Folic acid d) Niacin

b) Insulin Rationale: The three main facets to glycemic control for the woman with pregestational diabetes are diet, exercise, and insulin. Folic acid does not impact glycemic control. Glucose tablets are not a facet of glycemic control.

The nurse explains to a pregnant patient that she will need to take iron during her pregnancy after being diagnosed with iron-deficiency anemia. The nurse suggests that absorption of the supplemental iron can be increased by taking it with which of the following? a) Milk b) Orange juice c) Meals high in iron d) Legumes

b) Orange juice Rationale: Anemia is a condition in which the blood is deficient in red blood cells, from an underlying cause. The woman needs to take iron to manufacture enough red blood cells. Taking an iron supplement will help improve her iron levels, and taking iron with foods containing ascorbic acid, such as orange juice, improves the absorption of iron.

The nurse is teaching a pregnant woman with iron deficiency anemia about foods high in iron. Which food(s) if selected by the woman indicates a successful teaching program? Select all that apply. a) Potatoes b) Peanut butter c) Corn d) Raisins e) Yogurt f) Broccoli

b) Peanut butter d) Raisins f) Broccoli Rationale: Foods high in iron include dried fruits such as raisins, whole grains, green leafy vegetables such as broccoli and spinach, peanut butter, and iron-fortified cereals. Potatoes and corn are high in carbohydrates. Yogurt is a good source of calcium.

A client in her fifth month of pregnancy is having a routine clinic visit. The nurse should assess the client for which common second trimester condition? a) Metabolic alkalosis b) Physiological anemia c) Mastitis d) Respiratory acidosis

b) Physiological anemia Rationale: Hemoglobin level and hematocrit decrease during pregnancy as the increase in plasma volume exceeds the increase in red blood cell production. Mastitis is an infection in the breast characterized by a swollen tender breast and flulike symptoms. This condition is most commonly seen in breast-feeding clients. Alterations in acid-base balance during pregnancy result in a state of respiratory alkalosis, compensated by mild metabolic acidosis.

A client is diagnosed with peripartum cardiomyopathy (PPCM). Which of the following would the nurse expect to administer to the client? a) Methadone therapy b) Restricted sodium intake c) Ginger therapy d) Monoamine oxidase inhibitors

b) Restricted sodium intake Rationale: The client with peripartum cardiomyopathy should be prescribed restricted sodium intake to control the BP. Monoamine oxidase inhibitors are given to treat depression in pregnancy, and not peripartum cardiomyopathy. Methadone is a drug given for the treatment of substance abuse during pregnancy. Complimentary therapies like ginger therapy help in the alleviation of hyperemesis gravidarum, and not peripartum cardiomyopathy.

When dealing with a pregnant adolescent, the nurse assists the client to integrate the tasks of pregnancy while at the same time fostering development of which of the following? a) Trust b) Self-identity c) Autonomy d) Dependence

b) Self-identity Rationale: The nurse assists the pregnant adolescent to integrate the tasks of pregnancy, bonding, and preparing to care for another with the tasks of developing self-identity and independence. Trust is a developmental task of infancy. Autonomy is a developmental task of toddlerhood. Independence, not dependence, is fostered.

You are doing a nursing assessment on a new patient in the obstetric clinic. The woman estimates that she is approximately 16 weeks pregnant. While assessing her you ask about what appear to be scratch marks on her hands, and she tells you that she has three cats at home. What screening would be ordered for this woman? a) Cytomegalovirus b) Toxoplasmosis c) Herpes Simplex Virus d) Hepatitis C

b) Toxoplasmosis Rationale: Toxoplasmosis is an infection caused by the protozoan Toxoplasma gondii, also referred to as T. gondii. Transmission is via undercooked meat and through cat feces. Toxoplasmosis is a common infection in humans and usually produces no symptoms. However, when the infection passes from the woman through the placenta to the fetus, a condition called congenital toxoplasmosis can occur. Approximately 400 to 4,000 cases of congenital toxoplasmosis occur per year in the United States (Williams, 2007). The classic triad of symptoms for congenital toxoplasmosis is chorioretinitis, intracranial calcification, and hydrocephalus in the newborn.

Which factor would contribute to a high-risk pregnancy? a) First pregnancy at age 33 b) Type 1 diabetes c) History of allergy to honey bee pollen d) Blood type O positive

b) Type 1 diabetes Rationale: A woman with a history of diabetes has an increased risk for perinatal complications, including hypertension, preeclampsia, and neonatal hypoglycemia. The age of 33 without other risk factors doesn't increase risk, nor does type O-positive blood or environmental allergens.

A pregnant single mom living alone tells the nurse she is considering getting a cat for her two year old daughter. Which is the best response by the nurse? a) The exposure to the cat litter may cause you to need a C-section b) You should wait until after you deliver to obtain the cat for your daughter c) This will cut down on the jealousy for your two year old when the baby comes d) If you don't think caring for a cat is too much work, that would be great

b) You should wait until after you deliver to obtain the cat for your daughter Rationale: Toxoplasma gondii is a protozoan that can be transmitted via undercooked meat and through cat litter. Having a cat is not an issue, but cleaning the litter box may expose the mother to the infection and result in fetal anomalies. Option A is incorrect; exposure to the cat litter will not necessitate a cesarean section. Option B is incorrect; having a cat will not cut down on any jealousy the 2-year-old might feel when the new baby is born. Option C is incorrect; the nurse would not encourage the mother to get her child a cat until after the new baby is born.

what happens to the lungs + the foramen ovale, after baby's first breath?

baby breathes = inflation of lungs. this causes increased blood flow to lungs via RV, which then cause pressure to increase in LA. the increased pressure in the LA causes septum primum, a flap that closes to the ductus arteriosus, to snap shut (now the atria are seperated by a wall). the foramen ovale functionally closes w/in 1-2 hrs. physiologically closed by 1 mo d/t deposits of fibrin. permanently closed by 6th month of life.

Client experiencing shoulder dystocia during birth. The nurse would place PRIORITY on performing which assessment postbirth? a. extensive lacerations b. monitor of cardiac anomaly c. brachial plexus assessment

brachial plexus to identify nerve damage

A pregnant woman who has had cardiovascular disease for the last 3 years asks the nurse why this disorder makes her pregnancy an "at-risk" pregnancy. What is the nurse's best response? a) "The fact that you are receiving prenatal care will help." b) "Our facility has a lot of experience in dealing with this." c) "Pregnancy taxes the circulatory system of every woman." d) "Don't worry. You have an excellent doctor."

c) "Pregnancy taxes the circulatory system of every woman." Rationale: Pregnancy taxes the circulatory system of every woman because both the blood volume and cardiac output increase by approximately 30% to 50%. Half of these increases occur by 8 weeks; they are maximized by mid-pregnancy.

A client is 33 weeks pregnant and has had diabetes since age 21. When checking her fasting blood glucose level, which value would indicate the client's disease is controlled? a) 45 mg/dl b) 120 mg/dl c) 85 mg/dl d) 136 mg/dl

c) 85 mg/dl Rationale: Recommended fasting blood glucose levels in pregnant clients with diabetes are 60 to 95 mg/dl. A fasting blood glucose level of 45 g/dl is low and may result in symptoms of hypoglycemia. A blood glucose level below 120 mg/dl is recommended for 2-hour postprandial values. A blood glucose level above 136 mg/dl in a pregnant client indicates hyperglycemia.

A postpartum mother has the following lab data recorded: a negative rubella titer. What is the appropriate nursing intervention? a) Assess the rubella of the baby b) No action needed. c) Administer rubella vaccine before discharge. d) Notify the health care provider.

c) Administer rubella vaccine before discharge. Rationale: Rubella is a virus, which when contracted during pregnancy has significant complications for the fetus. The illness is mild to the adult but can result in the infant being born deaf and blind. There is no cure, the CDC recommends all individuals be vaccinated against rubella. If the titer is negative, the mother does not have protection against rubella and the next pregnancy would be at risk. She should receive the vaccination prior to discharge from the hospital. This makes option A incorrect. Assessing the rubella titer of the baby would not mean anything. The baby has not had rubella and has not received antibodies against rubella from the mother. Notifying the health care provider is not a priority, as most institutions have standing orders to administer the rubella vaccine if the mother's rubella titer is negative.

A pregnant client is screened for tuberculosis during her first prenatal visit. An intradermal injection of purified protein derivative (PPD) of the tuberculin bacilli is given. Which sign would indicate a positive test result? a) An indurated wheal under 10 mm in diameter appears in 6 to 12 hours. b) A flat circumscribed area over 10 mm in diameter appears in 48 to 72 hours. c) An indurated wheal over 10 mm in diameter appears in 48 to 72 hours. d) A flat, circumscribed area under 10 mm in diameter appears in 6 to 12 hours.

c) An indurated wheal over 10 mm in diameter appears in 48 to 72 hours. Rationale: A positive PPD result would be an indurated wheal over 10 mm in diameter that appears in 48 to 72 hours. The area must be a raised wheal, not a flat, circumscribed area.

A woman with a positive history of genital herpes is in active labor. She has small pin-point vesicles in the perineum area. Her membranes are ruptured, she is dilated 5cm, effaced 70%. The nurse should anticipate what type of delivery? a) Forceps assisted. b) Vacuum assisted. c) Cesarean. d) Spontaneous vaginal.

c) Cesarean. Rationale: An active herpes infection can be passed to the fetus during labor or with ruptured amniotic membranes. The nurse should anticipate the infant will be delivered via a cesarean birth. The risk of transmitting herpes to the baby would be increased if the baby were born by spontaneous vaginal delivery, vacuum assisted delivery, or forceps assisted delivery.

Nursing care for women diagnosed with gestational diabetes includes which of the following? a) Education for the women on life-long diabetic needs b) Counseling the patient on the need for cesarean birth c) Encourage blood glucose control d) Referral for the infant to diabetic care after delivery

c) Encourage blood glucose control Rationale: The patient will need to have control of her blood sugar during her pregnancy to decrease any complications associated with gestational diabetes. The patient with gestational diabetes does not need to be counseled on the need to have a cesarean delivery, receive education on life-long diabetic needs, or have the infant referred for diabetic care after delivery.

Human papillomavirus (HPV) can cause condylomata acuminata that can develop in clusters on the vulva, within the vagina, on the cervix, or around the anus. What is their risk? a) Neonatal auricular papillomas b) Blockage of the birth canal c) Heavy bleeding during vaginal delivery d) Neonatal hemorrhage

c) Heavy bleeding during vaginal delivery Rationale: Genital warts have a tendency to increase in size during pregnancy. This may result in heavy bleeding during vaginal delivery. The pregnant woman can pass HPV to her fetus during the birth process. In rare instances, neonatal HPV infection can result in life-threatening laryngeal papillomas. HPV infection transmitted to the infant may not appear for as long as 10 years after birth.

For which of the following problems would the nurse be alert in a pregnant woman with gestational diabetes? a) Placenta previa related to diabetes mellitus b) Hypotension related to glucose/insulin imbalance c) Hydramnios related to glucose/insulin imbalance d) Cerebral vascular accident related to diabetes mellitus

c) Hydramnios related to glucose/insulin imbalance Rationale: Hyperglycemia tends to lead to excessive amniotic fluid (hydramnios) because of osmotic pressure fluid shifts.

Which of the following changes in pregnancy would the nurse identify as a contributing factor for arterial thrombosis, especially for the woman with atrial fibrillation? a) Increased cardiac output b) Elevation of diaphragm c) Hypercoagulable state d) Increase in blood volume

c) Hypercoagulable state Rationale: The nurse should identify that the increased risk of arterial thrombosis in atrial fibrillation is due to hypercoagulable state of pregnancy. During pregnancy there is a state of hypercoagulation. This increases the risk of arterial thrombosis in clients having atrial fibrillation and artificial valves. Increased cardiac output and blood volume do not cause arterial thrombosis. Elevation of the diaphragm is due to the uterine distension and it causes a shift in the QRS axis and is not a associated with arterial thrombosis.

A client in her eighth month of pregnancy who has cardiac disease is experiencing profound shortness of breath and a cough that produces blood-speckled sputum, in addition to systemic hypotension. The nurse recognizes that this patient most likely is experiencing which of the following conditions? a) Peripartal cardiomyopathy b) Pulmonary embolism c) Left-sided heart failure d) Right-sided heart failure

c) Left-sided heart failure Rationale: In left-sided heart failure, the left ventricle cannot move the large volume of blood forward that it has received by the left atrium from the pulmonary circulation. It is characterized by a decrease in systemic blood pressure and pulmonary edema that produces profound shortness of breath. If pulmonary capillaries rupture under the pressure, small amounts of blood leak into the alveoli and the woman develops a productive cough with blood-speckled sputum. Right-sided heart failure is characterized by extreme liver enlargement, distention of abdominal and lower extremity vessels, ascites, and peripheral edema. A woman with peripartal cardiomyopathy develops signs of myocardial failure such as shortness of breath, chest pain, and nondependent edema. Her heart increases in size (cardiomegaly).The signs of a pulmonary embolism include chest pain, a sudden onset of dyspnea, a cough with hemoptysis, tachycardia or missed beats, or dizziness and fainting.

Which of the following changes in insulin is most likely to occur in a woman during pregnancy? a) Unavailable because it is used by the fetus b) Not released because of pressure on the pancreas c) Less effective than normal d) Enhanced secretion from normal

c) Less effective than normal Rationale: Somatotropin released by the placenta makes insulin less effective. This is a safeguard against hypoglycemia.

A woman with Class II heart disease is in the third trimester of her pregnancy. She's been taking good care of herself and has had little difficulty, but to be on the safe side the obstetrician has ordered bed rest for her for the final month. For her own and the baby's safety, in what position should the nurse advise the patient to sleep? a) Lie flat on her back. b) Stay in high Fowler's position. c) Lie in a semirecumbent position. d) Use pillows and wedges to stay in a fully recumbent position.

c) Lie in a semirecumbent position. Rationale: Semirecumbent position is the best position for circulation of the mother and fetus. Laying flat on the back can induce supine hypotensive syndrome and fully recumbent impedes other circulation. Therefore options A, B, and D are incorrect answers for this question.

During a routine prenatal check-up, the nurse interviews a pregnant client to identify possible risk factors for developing gestational diabetes. Which of the following would alert the nurse to an increased risk? Select all that apply. a) Previous history of spontaneous abortion b) Younger maternal age at pregnancy c) Maternal obesity with body mass index more than 35 d) Previous birth of small for gestational age baby e) Client of African-American lineage

c) Maternal obesity with body mass index more than 35 d) Previous birth of small for gestational age baby e) Client of African-American lineage Rationale: The risk factors for gestational diabetes include previous history of spontaneous abortion, maternal obesity with body mass index (BMI) more than 35, and client of African-American lineage. The other risk factors for gestational diabetes are previous history of stillbirth, birth of large for gestational age infant and advancing maternal age. High-risk ethnic groups include African American, Hispanic and Native North American.

Which medication is prescribed most commonly for a pregnant woman with chronic hypertension? a) Nifedipine b) Atenolol c) Methyldopa d) Labetolol

c) Methyldopa Rationale: Although labetolol, atenolol, and nifedipine may be ordered, methyldopa the most commonly prescribed agent because of its safety record during pregnancy. It is a slow-acting antihypertensive agent that also helps to improve uterine perfusion.

Your patient is pregnant and she has tested positive for cytomegalovirus. What can this cause in the newborn? a) Hypertension b) Clubbed fingers and toes c) Microcephaly d) Bicuspid valve stenosis

c) Microcephaly Rationale: Signs that are likely to be present in the 10 percent of newborns who are symptomatic at birth include microcephaly, seizures, IUGR, hepatosplenomegaly, jaundice, and rash.

A woman with cardiac disease delivered a seven pound baby by C-Section. Which of the following interventions should be implemented during the immediate postpartum period? a) Limit visits with the infant so mom may rest. b) Ambulate to bathroom only. c) Rest, stool softeners, and monitoring tolerance of activity. d) Restrict activity to bedrest.

c) Rest, stool softeners, and monitoring tolerance of activity. Rationale: A woman who has a cardiac condition is at increased risk in the postpartum period. She needs frequent assessment and observation for tolerance. She would also be given education to avoid straining activities such as bowel movements and would be encouraged to have stool softeners and increase fluid and fiber. Restricting the patient's activity to bed rest could be detrimental to the patient, as could be ambulating to the bathroom only. There is no reason to limit the visits with the infant.

You are doing patient teaching with a 28 weeks' gestation woman who has tested positive for gestational diabetes mellitus (GDM). What would be important to include in your patient teaching? a) Her baby is at increased risk for neonatal diabetes mellitus. b) She is at increased risk for type I diabetes mellitus after her baby is born. c) She is at increased risk for type II diabetes mellitus after her baby is born. d) Her baby is at increased risk for type I diabetes mellitus.

c) She is at increased risk for type II diabetes mellitus after her baby is born. Rationale: The woman who develops GDM is at increased risk for developing type 2 DM after pregnancy.

A pregnant patient with mitral stenosis needs to begin taking an anticoagulant. The nurse identifies the drug of choice, which is used in early pregnancy and again during the last month of pregnancy, to be which of the following? a) aspirin b) coumadin c) heparin d) levonox

c) heparin Rationale: If an anticoagulant is required, heparin is the drug of choice for the beginning and the end of pregnancy. Heparin does not cross the placenta barrier.

A pregnant woman diagnosed with cardiac disease 4 years ago is told that her pregnancy is a high-risk pregnancy. The nurse then explains that the danger occurs primarily because of the increase in circulatory volume. The nurse informs the patient that the most dangerous time for her is when? a) in weeks 12-20 b) in weeks 8-12 c) in weeks 28-32 d) in weeks 20-28

c) in weeks 28-32 Rationale: The danger of pregnancy in a woman with cardiac disease occurs primarily because of the increase in circulatory volume. The most dangerous time for a woman is in weeks 28 to 32, just after the blood volume peaks.

A nursing instructor is teaching students about anemia during pregnancy. Which type of anemia does the instructor teach students is most prevalent during pregnancy? a) sickle-cell anemia b) pernicious anemia c) iron-deficiency anemia d) folic acid anemia

c) iron-deficiency anemia Rationale: Iron-deficiency anemia is the most common type in pregnancy. Many woman enter pregnancy with a low iron count because of poor diet, heavy menstrual periods, unwise weight-loss programs, or a combination of these.

After teaching a group of nursing students about risk factors associated with dystocia, the instructor determines that the teaching was successful when the students identify which of the following as increasing the risk? (Select all that apply) A) Pudendal block anesthetic use B) Multiparity C) Short maternal stature D) Maternal age over 35 E) Breech fetal presentation

c,d,e According to American College of Obstetrics and Gynecology (ACOG, 2009a), factors associated with an increased risk for dystocia include epidural analgesia, excessive analgesia, multiple gestation, hydramnios, maternal exhaustion, ineffective maternal pushing technique, occiput posterior position, longer first stage of labor, nulliparity, short maternal stature (less than 5 feet tall), fetal birth weight (more than 8.8 lb), shoulder dystocia, abnormal fetal presentation or position (breech), fetal anomalies (hydrocephalus), maternal age older than 35 years, high caffeine intake, overweight, gestational age more than 41 weeks, chorioamnionitis, ineffective uterine contractions, and high fetal station at complete cervical dilation.

a client is admitted to the health facility. The fetus has a gestational age of 42 weeks and is suspected to have cephalopelvic disproportion. Which should the nurse do next? A. place client in lithonomy position for birth B. administer oxytocin 4 mU/min C. artificial rupture of membranes D. prepare client to have a cesarean birth

cephalopelvic disproportion is associated with postterm pregnancy. Client wouldn't be able to deliver vaginally

chorion + chorionic villi:

chorion, made by the trophoblast, consist of the trophoblast cells and a mesodermal lining. it has finger-like projections called chorionic villi which form the fetal portion of the placenta.

An increase in prostiglandins leads to myometrial ______ and to a reduction in cervical resistance

contractions

The fetal head at the vaginal opening is termed _________ and occurs before birth of the head.

crowning

A nursing instructor is teaching students about pre-existing illnesses and how they can complicate a pregnancy. The instructor recognizes a need for further education when one of the students makes which statement? a) "A pregnant woman needs to be careful of and cautious about accidents and illnesses during her pregnancy." b) "A pregnant woman with a chronic illness can put the fetus at risk." c) "A pregnant woman with a chronic condition can put herself at risk." d) "A pregnant woman does not have to worry about contracting new illnesses during pregnancy."

d) "A pregnant woman does not have to worry about contracting new illnesses during pregnancy." Rationale: When a woman enters a pregnancy with a chronic illness, it can put both her and the fetus at risk. She needs to be cautious about developing a new illness during her pregnancy as well as having an accident during the pregnancy.

A young patient with a cardiac problem wants to get pregnant and tells the nurse that she is sad that she will never be able to have a baby. What is the best response by the nurse? a) "Cardiovascular problems are not a concern during pregnancy." b) "Women with your problem should never get pregnant, because the risks and dangers are too high for you and the fetus." c) "If you get pregnant, you are likely to face many complications." d) "Because of improved management, more women with cardiac problems can complete pregnancies successfully."

d) "Because of improved management, more women with cardiac problems can complete pregnancies successfully." Rationale: Because of improved management of cardiac disease, women who might never have risked pregnancy in the past can complete pregnancies successfully today.

The nurse is helping an indigent HIV-positive pregnant patient set up a postdelivery care plan for her baby. What is an appropriate question/statement during that discussion? a) "You understand that you can't breast-feed, right? Even though formula's expensive, you'll need to figure out a way to get it." b) "You're not planning to breast-feed are you? That would be dangerous for the baby." c) "HIV can be passed to the baby from breast-feeding so it's important that you give the baby formula. You probably can't afford formula can you?" d) "HIV can be passed to the baby from breast-feeding so it's important that you give the baby formula. Formula's pretty expensive so I'll give you some information for places you can contact if you ever need some help getting it."

d) "HIV can be passed to the baby from breast-feeding so it's important that you give the baby formula. Formula's pretty expensive so I'll give you some information for places you can contact if you ever need some help getting it." Rationale: HIV is possible to transmit via breastfeeding and formula is the only option for feeding. The nurse needs to provide positive information and offer to make referral or get assistance for the patient in financial need. Assuming the patient understands the reason she cannot breastfeed is not adequate nursing care. The patient needs the nurse to explain to her the reason for not breastfeeding her infant. Option B is incorrect as it does not fully answer the question being asked. Option C is inappropriate because telling the patient that she probably can't get formula since it is expensive is inappropriate. It is a negative comment about the patient and her status in life.

Your pregnant patient has had asthma since she was a teenager. What statement by the patient would alert you to the fact her asthma may not be in control? a) Daytime cough b) Decreased respiratory rate c) Feeling of euphoria d) "I keep waking up at night."

d) "I keep waking up at night." Rationale: Complaint of nocturnal awakening is a classic symptom.

A nurse is talking to a newly pregnant woman who had a mitral valve replacement in the past. Which of the following statements by the patient reveals an understanding about the pre-existing condition? a) "I know I will be fine, but I worry about the fetus." b) "I know my baby will be fine, but I am worried about having a personal complication." c) "I don't have to worry about this, because I had the problem fixed before I became pregnant." d) "I understand that my fetus and I both are at risk for complications."

d) "I understand that my fetus and I both are at risk for complications." Rationale: When a woman enters pregnancy with a pre-existing condition, both she and her fetus can be at risk of developing complications.

A pregnant woman is determined to be at high risk for gestational diabetes. At which time would the nurse expect the client to undergo rescreening? a) 28 to 32 weeks b) 16 to 20 weeks c) 20 to 24 weeks d) 24 to 28 weeks

d) 24 to 28 weeks Rationale: A woman identified as high risk would undergo rescreening between 24 and 28 weeks.

An alert, diabetic, pregnant woman in the hospital experiences some shakiness and diaphoresis with a fasting blood sugar of 60 mg/dl when she awakens in the morning. Which action should the nurse take first? a) Recheck her blood sugar for accuracy. b) Stay with her and ask another nurse to bring her insulin. c) Withhold her insulin and notify the health care provider. d) Administer the patient's glucose tablets.

d) Administer the patient's glucose tablets. Rationale: The patient is hypoglycemic when awakening in the morning. The nurse should provide glucose and be prepared to reassess. The nurse should not recheck at this point, since the patient is symptomatic. She does not need insulin, and she will have her morning dose adjusted after breakfast.

A client in week 38 of her pregnancy arrives at the emergency room reporting a sharp pain between her umbilicus and the iliac crest in her lower right abdomen that is increasing. She reports having experienced intense nausea and vomiting for the past 3 hours. Given these symptoms, the nurse suspects which of the following conditions? a) Ectopic pregnancy in conjunction with morning sickness b) Pulmonary embolism c) Left-sided heart failure d) Appendicitis

d) Appendicitis Rationale: With appendicitis, the nausea and vomiting is much more intense than with morning sickness and the pain is sharp and localized at McBurney's point (a point halfway between the umbilicus and the iliac crest on the lower right abdomen). With a ruptured ectopic pregnancy, a woman may experience abdominal pain that is either diffuse or sharp, but it is less likely to occur precisely at McBurney's point. The symptoms described do not match those of pulmonary embolism or left-sided heart failure.

You are doing patient teaching with a 30-year-old gravida 1 who has sickle cell anemia. She is not currently in crisis. Providing education on which topic is the highest nursing priority? a) Control of pain b) Constipation prevention c) Iron-rich foods d) Avoidance of infection

d) Avoidance of infection Rationale: Prevention of crises, if possible, is the focus of treatment for the pregnant woman with sickle cell anemia. Maintaining adequate hydration, avoiding infection, getting adequate rest, and eating a balanced diet are all common-sense strategies that decrease the risk of a crisis.

A pregnant woman in her 39th week of pregnancy presents to the clinic with a vaginal infection. She tests positive for chlamydia. What would this make her infant at risk for? a) Neonatal laryngeal papillomas b) Deafness c) Chicken pox d) Blindness

d) Blindness Rationale: A pregnant woman who contracts chlamydia is at increased risk for spontaneous abortion (miscarriage), pre-term rupture of membranes, and pre-term labor. The postpartum woman is at higher risk for endometritis (Fletcher & Ball, 2006). The fetus can encounter bacteria in the vagina during the birth process. If this happens, the newborn can develop pneumonia or conjunctivitis that can lead to blindness.

When providing nutritional counseling to a pregnant woman with diabetes, the nurse would urge the client to obtain most of her calories from which of the following? a) Protein sources b) Unsaturated fats c) Saturated fats d) Complex carbohydrates

d) Complex carbohydrates Rationale: The pregnant woman with diabetes is encouraged to eat three meals a day plus three snacks, with 40% of calories derived from good-quality complex carbohydrates, 35% of calories from protein sources, and 35% of calories from unsaturated fats. The intake of saturated fats should be limited during pregnancy, just as they should be for any person to reduce the risk of heart disease.

A woman with cardiac disease is 32 weeks gestation and alerts the nurse she has been having spells of light-headedness and dizziness every few days. The nurse provides which of the following interventions as an option to the patient? a) Bed rest and bathroom privileges only until delivery. b) Increase fluids and take more vitamins. c) The patient needs to discuss induction of labor with the physician. d) Decrease activity and rest more often.

d) Decrease activity and rest more often. Rationale: If the patient is developing symptoms associated with her heart condition, the first intervention is to monitor activity levels, decrease activity and treat the symptoms. At 32 weeks gestation, the suggestion to induce labor is not appropriate and without knowledge of the type of heart condition one would not recommend increase of fluids or vitamins. Total bed rest may be required if the symptoms do not resolve with decreased activity. Therefore options B, C, and D are incorrect,

A pregnant client with sickle cell anemia is at an increased risk for having a sickle cell crisis during pregnancy. Aggressive management for a client experiencing a sickle cell crisis with severe pain includes which measure? a) Acetaminophen (Tylenol) for pain b) Antihypertensive drugs c) Diuretic drugs d) I.V. fluids

d) I.V. fluids Rationale: A sickle cell crisis during pregnancy is usually managed by exchange transfusion, oxygen, and I.V. fluids. Antihypertensive drugs usually aren't necessary. Diuretics wouldn't be used unless fluid overload resulted. The client usually needs a stronger analgesic than acetaminophen to control the pain of a crisis.

When providing education to a teenage prenatal class, the nurse states that infants born to teenage mothers are more likely to have which of the following? a) Lower mortality rates b) Genetic problems c) Post-date delivery d) Low-birth weight

d) Low-birth weight Rationale: Infants born to adolescent mothers are more likely to have a low-birth weight and poor outcomes and higher mortality rates when compared to infants of older mothers. Infants born to teenage mothers are not more likely to have genetic problems; they are more likely to be born pre-term rather than post-date.

A woman develops gestational diabetes. Which of the following assessments should she make daily? a) Test her urine for protein with a chemical reagent strip. b) Measure her uterine height by hand-span distance. c) Measure her abdominal diameter with a tape measure. d) Measure serum for glucose level by a finger prick.

d) Measure serum for glucose level by a finger prick. Rationale: Assessing serum glucose reveals both hyperglycemia and hypoglycemia.

The nurse is assessing a woman with class III heart disease who is in for a prenatal visit. What would be the first recognizable sign that this patient is in heart failure? a) Elevated blood pressure b) Low blood pressure c) Audible wheezes d) Persistent rales in the bases of the lungs

d) Persistent rales in the bases of the lungs Rationale: The earliest warning sign of cardiac decompensation is persistent rales in the bases of the lungs.

A 40-year-old woman comes to the clinic complaining of having missed her period for two months. A pregnancy test is positive. What is she and her fetus at increased risk for? a) Post-term delivery b) Type I diabetes Mellituus c) Type II diabetes Mellitus d) Placental abnormalities

d) Placental abnormalities Rationale: A woman older than 35 years is more likely to conceive a child with chromosomal abnormalities, such as Down syndrome. She is also at higher risk for spontaneous abortion (miscarriage), preeclampsia-eclampsia, gestational diabetes, pre-term delivery, bleeding and placental abnormalities, and other intrapartum complications.

The nurse is providing education to women who had diabetes prior to pregnancy. The nurse is discussing pregnancy-related complications from diabetes. Which of the following is a potential complication? a) Post-term delivery b) Small for gestation age infant c) Hypotension of pregnancy d) Polyhydramnios

d) Polyhydramnios Rationale: Polyhydramnios is an increase, or excess, in amniotic fluid and is a pregnancy-related complication associated with diabetes in pregnancy. An infant who is small for gestational age is not associated with a mother who had diabetes prior to pregnancy. Other pregnancy-related complications associated with pregestational DM include hypertensive disorders, preterm delivery, and shoulder dystocia.

A woman is pregnant and has asthma. Her physician has told her to continue taking prednisone during pregnancy, but she is concerned the drug may be teratogenic. What advice would be best to give her regarding this? a) Prednisone is a teratogenic drug, but she may need it to control her asthma symptoms. b) You would recommend she omit the drug during pregnancy. c) She should half her dose during the first 3 months of pregnancy. d) Prednisone is considered safe in the doses prescribed by her physician.

d) Prednisone is considered safe in the doses prescribed by her physician. Rationale: Women should take no medication during pregnancy except that prescribed by their primary-care provider. Prednisone may be prescribed safely because, although it may be teratogenic in animal models, it does not appear to be teratogenic in humans.

Which of the following is recommended to prevent transmission of HIV to a newborn if the mother has AIDS? a) Avoid scalp electrodes for internal fetal monitoring. b) Admit infant to NICU after delivery. c) Perform amniotomy. d) Prepare for cesarean delivery.

d) Prepare for cesarean delivery. Rationale: When a patient is HIV positive, the method of delivery preferred is cesarean. This method has the lowest transmission rate for passage of the HIV infection to the infant. The nurse should educate the woman on the standard of care for delivery in an HIV or AIDS positive mother. Avoiding scalp electrodes for internal fetal monitoring, admitting the infant to NICU, and performing an amniotomy are not recommended methods for preventing transmission of HIV to a newborn.

A woman who has sickle cell anemia asks you if her infant will develop sickle cell disease. The nurse would base the answer on which of the following? a) Sickle cell anemia is not inherited; it occurs following a malaria infection. b) Sickle cell anemia is dominantly inherited. c) Sickle cell anemia has more than one polygenic inheritance pattern. d) Sickle cell anemia is recessively inherited.

d) Sickle cell anemia is recessively inherited. Rationale: Sickle cell anemia is an autosomal recessive disease requiring that the person have two genes for the disease, one from each parent. If one parent has the disease and the other is free of the disease and trait, the chances of the child inheriting the disease is zero. Options B, C, and D are incorrect as they give misinformation to the patient and are inappropriate answers for the nurse to give.

While the nurse is weighing a pregnant woman at a regularly scheduled OB visit, the patient complains of vaginal itching, a great deal of foamy yellow-green discharge, and pain during intercourse. She says this is her first pregnancy and she didn't know this was what happened. What can the nurse tell her? a) This is not normal for pregnancy but the doctor might test her for a simple yeast infection. If it is a yeast infection, it can be treated with a single-dose suppository that will not harm the fetus. Remind the patient that she should call immediately if she has any symptoms that don't seem normal to her. b) This is not normal for pregnancy; the doctor might test her for chlamydia. If it is chlamydia, she and her partner can be treated with a 7-day course of antibiotics. Remind the patient that she should call immediately if she has any symptoms that don't seem normal to her. c) This is not normal for pregnancy; the doctor might test her for gonorrhea. If it is gonorrhea, she and her partner will be treated with antibiotics; they might be treated with different medications because some antibiotics normally used to treat gonorrhea are damaging to the fetus. Remind the patient that she should call immediately if she has any symptoms that don't seem normal to her. d) This is not normal for pregnancy; the doctor might test her for trichomoniasis. If it is trichomoniasis, she can be treated with an oral dose of metronidazole. Remind the patient that she should call immediately if she has any symptoms that don't seem normal to her.

d) This is not normal for pregnancy; the doctor might test her for trichomoniasis. If it is trichomoniasis, she can be treated with an oral dose of metronidazole. Remind the patient that she should call immediately if she has any symptoms that don't seem normal to her. Rationale: Trichomoniasis is caused by a one-celled protozoa. The symptoms include large amounts of foamy, yellow-green vaginal discharge. Treatment is with metronidazole, her partner needs to be treated as well. A yeast infection presents with a cottage-cheese like discharge, so option A is incorrect. Chlamydia often has no symptoms. If the woman does experience symptoms, these may include vaginal discharge, abnormal vaginal bleeding, and abdominal or pelvic pain. Gonorrhea may have symptoms so mild that they go unnoticed in the woman. The woman who contracts gonorrhea may have vaginal bleeding during sexual intercourse, pain, and burning while urinating, and a yellow or bloody vaginal discharge.

When developing a plan of care for a pregnant woman who is HIV-positive, which of the following is essential? a) Helping her choose a newborn feeding method b) Preparing the woman for cesarean birth c) Educating her about family planning d) Using Standard Precautions

d) Using Standard Precautions Rationale: For the pregnant woman who is HIV-positive, nurses must always use Standard Precautions to reduce the risk of HIV transmission. Educating the woman about family planning methods is not as important as adhering to Standard Precautions. The decision about the mode of delivery is based on the woman's viral load, duration of ruptured membranes, progress of labor, and other clinical factors. Breast-feeding is contraindicated, so helping her choose a feeding method would be inappropriate.

A woman with no previous history of heart disease begins to have symptoms of myocardial failure a few weeks before the delivery of her first child. Findings include shortness of breath, chest pain, and edema, with her heart also showing enlargement. Which disease should the nurse suspect? a) pulmonary valve stenosis b) left sided heart failure c) mitral stenosis d) peripartal cardiomyopathy

d) peripartal cardiomyopathy Rationale: Peripartal cardiomyopathy can occur in pregnancy without any previous history of heart disease. Symptoms include shortness of breath, chest pain, and edema; also, the heart begins to increase. Treatment is with a diuretic, an antidysrhythmic agent, digitalis, low weight heparin, and bed rest.

fetal demise

death of the fetus

Oxytocin aids in stimulating prostiglandin synthesis through receptors in the _____

decidua

A nurse is caring for a client in labor who is delivering. Which of the following fetal responses should the nurse monitor for in the client's baby

decrease in circulation and perfusion to the fetus

____ is downward movement of the fetal head until it is within the pelvic inlet

descent

a nurse is caring for a pregnant client in labor in the health care facility. The nurse knows that which of the following marks the termination of the first stage of labor in the client

dilation of the cervix to 10cm

3 shunts during fetal life:

ductus venosus: connects umbilical vein to inferior vena cava ductus arteriosus: connects main pulmonary artery to aorta foramen ovale: opening between right and left atria

The thinning out process of the cervix during labor is termed what?

effacement

fetal stage

end of the eighth week until birth. pg. 336

embryonic stage

end of the second week through the eighth week. pg. 336

____occurs when the greatest transverse diameter of the head in vertex passes through the pelvic inlet

engagement

5 hormones of the placenta:

estrogen progesterone relaxin HPL HCG

A nurse is caring for a pregnant client who is in the active phase of labor. At what interval should the nurse monitor the client's vital signs?

every 30 minutes

The head emerges through extension under the symphasis pubis, along with the shoulders

extension

If the nitrazine test is inconclusive, an additional test, called the ________________ test, can be used to confirm rupture of membranes

fern

preembryonic stage

fertilization through the second week. pg. 336

_____ occurs as the vertex meets resistance from the cervix, walls of the pelvis, or pelvic floor

flexion

structure of umbilical cord:

formed by the amnion, and houses 1 large vein + 2 small arteries wrapped in wharton's jelly to prevent compression. @ term, average length is 22 in long + 1 in thick.

assessment of a fetus identifies the buttocks as the presenting part, with the legs extended upward. The nurse identifies this as which type of breech presentation? A) Frank B) Full C) Complete D) Footling

frank "I frankly dont care if I show my butt"

genotype

genes inherited from parents. Genetic makeup of an individual, usually in the form of DNA, is the internally coded inheritable information. pg.348

Vaginal birth is most favorable with ______ type of pelvis because the inlet is round and the outlet is roomy

gynecoid

- Green fluid may indicate that the fetus has passed meconium secondary to:

hypoxia, prolonged pregnancy, cord compression, intrauterine growth restriction, maternal hypertension, diabetes, or chorioamnionitis; however, it is considered a normal occurrence if the fetus is in a breech presentation.

During an admission assessment of a client in labor, the nurse observes that there is no vaginal bleeding yet. What nursing intervention is appropriate in the absence of vaginal bleeding? A. Monitor vital signs B. Assess amount of cervical dilation C. Obtain urine speicmen for urinalysis D. Monitor hydration status

if vaginal bleeding is absent during admission assessment, nurse should perform vaginal examination to assess amount of cervical dilation

Bishop score less than 6

indicates that cervical ripening method should be used before inducing labor

Amniotic fluid should be clear when the membranes rupture, either spontaneously or artificially through an amniotomy. - Cloudy or foul-smelling amniotic fluid indicates __________ .

infection

amnion:

inner membrane containing amniotic fluid. located in the ectoderm. umbilical cord is formed from here.

Braxton Hicks contractions

intermittent painless uterine contractions that occur with increasing frequency as the pregnancy progresses

Pelvic phase of the second stage of labor

is characterized by complete cervical dilation and effacement, with strong contractions every 2 to 3 minutes; the mother focuses on pushing.

how does newborn circulation work?

it's all a closed system now. unoxygenated blood enters heart through superior/inferior vena cava, enters RA & is pumped in RV. from RV, blood is pumped through pulmonary arteries into lungs, where it's oxygenated. blood travels back via pulmonary veins into LA, then is pumped in LV. from the LV, blood is pumped to all extremities and organs.

Turning the woman on her left side would be an appropriate intervention for a _______ deceleration pattern.

late

Contractions every 5 minutes with cervical dilation of 3 cm is typical of the _______ phase.

latent

FHR is assessed every 30 to 60 minutes during the _________ phase of labor.

latent

_____ occurs when the fetal presenting part begins to descend into the maternal pelvis

lightening

client reports increased respiratory capacity, decreased dyspnea, increased pelvic pressure, cramping, and low back pain, more frequent urination. Also, edema in lower extremities (a result of blood stasis of blood pooling) and increased vaginal discharge. What should the nurse suspect?

lightening when fetal presenting part begins to descend into pelvis ( may occur two weeks before labor)

perpendicular line

lines that form right angles

Trendelenburg position

lying on back with body tilted so that the head is lower than the feet

Of all of the synthetic opioids ___________ is the most commonly used opioid for the management of pain during labor.

meperidine

The elongated shape of the fetal skull at birth as a result of overlapping of the cranial bones is known as ______

molding

Respiratory depression Opioids given close to the time of birth can cause central nervous system depression, including respiratory depression, in the newborn, necessitating the administration of ____________

naloxone.

When the presenting part is above the ischial spines, it is noted as a ____________ station.

negative

A sudden increase in energy on the part of the expectant women 24-48 hours before the onset of labor is sometimes referred to as

nesting

this test evaluate fetal well being by monitoring fetal heart tracing

nonstress test

a nurse is caring for a client who is experiencing acute onset of dyspnea and hypotension. Physician suspect client has amniotic fluid embolism. What other signs would alert the nurse?

nurse should watch for cyanosis, pulmonary edema, hypotension, seizures, tachycardia, coagulation failure, hemorrhage

what is the blastocyst? also, how is it formed?

once the morula enters the uterus, uterine fluid enters it, turning it into a fluid-filled ball of cells. these cells will continue to divide in specialized cells that will turn into fetal structures. the fluid filled ball of cells is the blastocyst, which will form the embryo & the amnion.

Allele

one of two or more alternative forms of a gene that arise by mutation and are found at the same place on a chromosome.

identify the 3 layers of the blastocyst & what they form:

outer: ectoderm - CNS, special senses, skin, glands mid: mesoderm - skeleton, urinary, circulatoy systems + reproductive organs inner: endoderm - liver, pancreas, respiratory system, digestive system

how does blood circulate w/in a fetus?

oxygenated blood is arrives from placenta, half goes through capillaries in liver, while the other half enters inferior vena cava via ductus venosus. from the inferior vena cava, blood [oxygenated] travels to right atrium, then is shunted into the left atrium via foramen ovale, then to the left ventricle. from there, the majority of the blood is pushed to the parts that need it most. the brain & the heart, while a little bit is sent to the lungs to keep them nourished. the blood enters the descending aorta, where it makes its way back to the placenta, via the umbilical arteries.

The birth _____ is the route through which the fetus must travel to be birthed vaginally

passageway

The cervix, vagina, and pelvic floor muscles are the soft tissues of one of the 5 P's

passageway

Oxytocin is secreted by

posterior pituitary

Breech positions are associated with what factors?

prematurity, placenta previa, multiparity, abnormalities (fibroids), congenital anomalies such as hydrocephaly

Passage of the mucous plug

premonitory sign of labor

genetic counseling

process by which the patients or relatives at risk of an inherited disorder are advised of the consequences and nature of the disorder, the probability of developing or transmitting it, and the options open to them in management and family planning.

fertilization

process occurs in about an hour. When one spermatozoon penetrates the ovum's thick outer membrane. pg. 336

modified sims position

prolapsed cord

heterozygous

refers to a particular gene that has different alleles on both homologous chromosomes. pg. 348

homozygous

refers to a particular gene that has identical alleles on both homologous chromosomes. pg. 348

Fetal _______________ as evidenced by a fetal heart rate greater than 160 bpm accompanied by a decrease in variability and late decelerations is an ominous sign indicating the need for prompt intervention.

tachycardia

genomics

the branch of molecular biology concerned with the structure, function, evolution, and mapping of genomes.

mutation

the changing of the structure of a gene, resulting in a variant form that may be transmitted to subsequent generations, caused by the alteration of single base units in DNA, or the deletion, insertion, or rearrangement of larger sections of genes or chromosomes.

genome

the complete set of genes or genetic material present in a cell or organism.

what happens to the ductus venosus & the ductus arteriosus after birth?

the ductus venosus usually closes w/ inhibition of blood flow secondary to cutting of umbilical cord. the ductus arteriosus closes w/ the increased oxygenation of arterial blood levels. its closure prevents mixing of blood from aorta and pulmonary artery.

what is fertilization? also, when does it occur

the meeting of sperm & egg. it occurs around 2 weeks after the last normal menstrual period in a 28-day cycle.

karyotype

the number and visual appearance of the chromosomes in the cell nuclei of an organism.

trophoblast:

the outer layer of cells surrounding the blastocyst cavity. it develops into the chorion + helps form the placenta.

phenotype

the physical expression, or characteristics, of a trait.

implantation:

the process of attachment (of the blastocyst?) to the endometrium of the uterus. occurs 7-8 days after fertilization

mosaicism

the property or state of being composed of cells of two genetically different types.

Lightening

the sensation of the fetus moving from high in the abdomen to low in the birth canal

lightening

the sensation of the fetus moving from high in the abdomen to low in the birth canal

genetics

the study of heredity and the variation of inherited characteristics.

zona pellucida

the thick transparent membrane surrounding a mammalian ovum before implantation. pg. 337

A client experiencing contractions presents at a health care facility. Assessment conducted by the nurse reveals that the client has been experiencing Braxton Hick contractions. The nurse hast to educate the client on the usefulness of Braxton Hicks contractions. Which of the following is the role of Braxton Hicks contractions in aiding labor?

these contractions help in softening and ripening the cervix

nitrazine paper

to confirm ruptured membrane

A client in the 3rd stage of labor has experienced placental separation and expulsion. Why is it necessary for a nurse to massage the woman's uterus briefly until it is firm?

to constrict the uterine blood vessels

oligohydramnios:

too little amniotic fluid (<500 ml @ term). - associates w/: uteroplacental insufficiencies, fetal renal abnormalities, and higher risk for surgical birth (c-section) &/or low-birth weight baby.

polyhydramnios:

too much amniotic fluid ( >2000 ml @ term) - associated w/: maternal diabetes, neural tube defects, chromosomal deviations, malformations of CNS &/or GI tract that prevent normal swallowing of amniotic fluid by fetus. - risk for premature rupture of membranes d/t overdistention.

The __________ phase is characterized by strong contractions occurring every 1 to 2 minutes and cervical dilation from 8 to 10 cm.

transition

A nurse is caring for a client who is scheduled to undergo amnioinfusion. The nurse knows that the client will not be able to have this procedure if which condition is present? a. uterine hypertonicity b. active genital herpes c. BP 130/88 d. decrease urine output

uterine hypertonicity

VBAC

vaginal birth after cesarean section

monosomy

when a diploid organism has only one copy of one of its chromosomes instead of two.

____ station is designated when the presenting part is at the level of the maternal ischial spines

zero

Amnioinfusion indications

• Oligohydramnios • Umbilical compression - D/T lack of amniotic fluid • To reduce FHR variable deceleration • Dilute meconium stained amniotic fluid


संबंधित स्टडी सेट्स

OCE1001 Chapters 1-4 Combined (3)

View Set

ATI Exit Exam #2 version of comprehensive retake test

View Set

Alapfogalmak 1. tétel... vagy mi a fasz

View Set

Chapter 12: Estimating Cash Flows on Capital Budgeting Projects

View Set